SlideShare a Scribd company logo
1 of 83
Chuyªn ®Ò BDHSG To¸n THCS
§æi míi ph¬ng ph¸p d¹y häc to¸n theo híng tÝch cùc ho¸ ho¹t ®éng häc tËp
cña häc sinh
A. Mét vµi nÐt vÒ lý luËn d¹y häc
1.T tëng tÝch cùc ho¸ ho¹t ®éng häc tËp cña häc sinh
X· héi ph¸t triÓn vµ sù ®æi míi ®Êt níc ®ßi hái ph¶i n©ng cao chÊt lîng gi¸o
dôc nh»m ®µo t¹o nh÷ng con ngêi lao ®éng ®¶m b¶o môc tiªu hiÖn ®¹i ho¸ ®Êt n-
íc.
+ Theo Kharlamop.I.F. “Häc tËp lµ mét qu¸ tr×nh nhËn thøc tÝch cùc”
Theo tõ ®iÓn TiÕng ViÖt: TÝch cùc lµ mét tr¹ng th¸i tinh thÇn cã t¸c dông
kh¼ng ®Þnh vµ thóc ®Èy sù ph¸t triÓn . Trong ho¹t ®éng häc tËp nã diÔn ra ë
nhiÒu ph¬ng diÖn kh¸c nhau: Tri gi¸c tµi liÖu, th«ng hiÓu tµi liÖu, ghi nhí, luyÖn
tËp, vËn dông, kh¸i qu¸t... vµ ®îc thÓ hiÖn ë nhiÒu h×nh thøc ®a d¹ng, phong
phó.
§éng c¬ häc tËp lµ nguån t¹o ra tÝnh tÝch cùc trong ho¹t ®éng häc vµ khi ®·
h×nh thµnh l¹i cã gi¸ trÞ nh mét ®éng c¬ thóc giôc ho¹t ®éng, lµ thuéc tÝnh cña
nh©n c¸ch, cßn tÝnh tÝch cùc l¹i lµ mét tr¹ng th¸i tinh thÇn lµm nÒn cho ho¹t ®éng
diÔn ra cã hiÖu qu¶ vµ cã thuéc tÝnh thiªn vÒ c¶m xóc .
G.I. Sukina ®· chia tÝnh tÝch cùc ra lµm ba cÊp ®é
1. TÝnh tÝch cùc b¾t chíc, t¸i hiÖn: xuÊt hiÖn do t¸c ®éng kÝch thÝch bªn
ngoµi. Trong trêng hîp nµy ngêi häc thao t¸c trªn ®èi tîng, b¾t chíc theo mÉu
hoÆc m« h×nh cña GV, nh»m chuyÓn ®èi tîng tõ ngoµi vµo trong theo c¬ chÕ
“ho¹t ®éng bªn ngoµi bªn trong cã cïng cÊu tróc”. Nhê ®ã, kinh nghiÖm ho¹t ®éng
®îc tÝch luü th«ng qua kinh nghiÖm ngêi kh¸c.
2. TÝnh tÝch cùc t×m tßi: ®i liÒn víi qu¸ tr×nh h×nh thµnh kh¸i niÖm. Gi¶i
quyÕt c¸c t×nh huèng nhËn thøc, t×m ra c¸c ph¬ng thøc hµnh ®éng trªn c¬ së cã
tÝnh tù gi¸c, cã sù tham gia cña ®éng c¬, nhu cÇu, høng thó vµ ý chÝ cña HS.
Lo¹i nµy xuÊt hiÖn kh«ng chØ do yªu cÇu cña GV mµ cßn hoµn toµn tù ph¸t trong
qu¸ tr×nh nhËn thøc. Nã tån t¹i kh«ng chØ ë d¹ng tr¹ng th¸i, c¶m xóc mµ cßn ë
d¹ng thuéc tÝnh bÒn v÷ng cña ho¹t ®éng. ë møc ®é nµy tÝnh ®éc lËp cao h¬n
møc trªn, cho phÐp HS tiÕp nhËn nhiÖm vô vµ tù m×nh t×m ra ph¬ng tiÖn thùc
hiÖn.
1
Chuyªn ®Ò BDHSG To¸n THCS
3. TÝnh tÝch cùc s¸ng t¹o: thÓ hiÖn khi chñ thÓ nhËn thøc tù t×m tßi kiÕn
thøc míi, tù t×m kiÕm ra ph¬ng thøc hµnh ®éng riªng vµ trë thµnh phÈm chÊt bÒn
v÷ng cña c¸ nh©n. §©y lµ møc ®é biÓu hiÖn tÝnh tÝch cùc nhËn thøc cao nhÊt .
Nh vËy nãi vÒ tÝnh tÝch cùc nhËn thøc, ngêi ta thêng ®¸nh gi¸ vÒ møc ®é
nhËn thøc cña ngêi häc trong qu¸ tr×nh thùc hiÖn môc ®Ých d¹y häc.
Kharlamop I.F. viÕt: “TÝnh tÝch cùc trong ho¹t ®éng nhËn thøc lµ tr¹ng th¸i
ho¹t ®éng cña HS, ®îc ®Æc trng bëi kh¸t väng häc tËp, sù cè g¾ng trÝ tuÖ víi
nghÞ lùc cao trong qu¸ tr×nh n¾m v÷ng kiÕn thøc cho chÝnh m×nh”.
+ TÝch cùc ho¸ ho¹t ®éng häc tËp cña häc sinh: Tèi ®a ho¸ sù tham gia ho¹t
®éng cña ngêi häc víi ®Þnh híng chØ ®¹o lµ tù nhËn thøc, tù ph¸t triÓn, tù thùc
hiÖn, tù kiÓm tra vµ ®¸nh gi¸, qua ®ã h×nh thµnh vµ ph¸t triÓn t duy ®éc lËp vµ
s¸ng t¹o cña HS.
GS.TSKH NguyÔn B¸ Kim chØ râ 4 yªu cÇu ®Ó tÝch cùc ho¸ ho¹t ®éng häc
tËp cña HS:
- X¸c lËp vÞ trÝ chñ thÓ cña ngêi häc, ®¶m b¶o tÝnh tù gi¸c, tÝch cùc s¸ng
t¹o cña ho¹t ®éng häc tËp.
- D¹y häc ph¶i dùa trªn nghiªn cøu t¸c ®éng cña nh÷ng quan niÖm vµ kiÕn
thøc s½n cã cña ngêi häc, nh»m khai th¸c mÆt thuËn lîi, h¹n chÕ mÆt khã kh¨n,
nghiªn cøu nh÷ng chíng ng¹i hoÆc sai lÇm cã thÓ cã cña nh÷ng kiÕn thøc ®ã
trong qu¸ tr×nh häc tËp cña HS.
- D¹y häc kh«ng chØ nh»m môc ®Ých lµ tri thøc vµ kü n¨ng bé m«n, mµ
quan träng h¬n c¶ lµ viÖc häc, d¹y c¸ch häc cho HS.
- Qu¸ tr×nh d¹y häc ph¶i bao hµm c¶ viÖc d¹y c¸ch tù häc th«ng qua viÖc ®Ó
HS tù ho¹t ®éng nh»m ®¸p øng nhu cÇu cña b¶n th©n vµ cña x· héi .
Tãm l¹i: §Ó ph¸t huy ®îc tÝnh tÝch cùc trong ho¹t ®éng häc tËp cña HS cÇn
mét qu¸ tr×nh lµm cho ngêi häc trë thµnh chñ thÓ tÝch cùc trong ho¹t ®éng häc
tËp cña chÝnh hä.
2. §Þnh híng ®æi míi ph¬ng ph¸p d¹y häc
+ Ph¬ng ph¸p d¹y häc
- Ph¬ng ph¸p d¹y häc lµ con ®êng, lµ c¸ch thøc cña ho¹t ®éng ®Ó ®¹t môc
®Ých d¹y häc.
2
Chuyªn ®Ò BDHSG To¸n THCS
- Ph¬ng ph¸p d¹y häc lµ c¸ch thøc ho¹t ®éng vµ giao lu cña thÇy g©y nªn
nh÷ng ho¹t ®éng vµ giao lu cÇn thiÕt cña trß nh»m ®¹t môc ®Ých d¹y häc .
- Ho¹t ®éng cña thÇy g©y nªn ho¹t ®éng cña trß:
Ho¹t ®éng cña thÇy lµ t¸c ®éng ®iÒu khiÓn. Tuy nhiªn t¸c ®éng kh«ng chØ
gåm ho¹t ®éng mµ cßn cã sù øng xö cña thÇy gi¸o.
ThuËt ng÷ “d¹y häc” vèn ®îc dïng ®Ó ph¶n ¸nh ho¹t ®éng cña ngêi d¹y, thÕ
nhng ®èi tîng cña ho¹t ®éng d¹y häc lµ HS, HS võa lµ ®èi tîng cña ho¹t ®éng d¹y
l¹i võa lµ chñ thÓ cña ho¹t ®éng häc. V× vËy ph¬ng ph¸p d¹y häc võa bao hµm
c¸ch d¹y cña thÇy vµ c¸ch häc cña trß.
+ §æi míi ph¬ng ph¸p d¹y häc
NghÞ quyÕt Trung ¬ng II kho¸ VIII cña §¶ng Céng s¶n ViÖt Nam chØ râ “§æi
míi m¹nh mÏ ph¬ng ph¸p gi¸o dôc ®µo t¹o, kh¾c phôc lèi truyÒn thô mét chiÒu, rÌn
luyÖn thµnh nÕp t duy s¸ng t¹o cña ngêi häc...”
§µo t¹o con ngêi n¨ng ®éng s¸ng t¹o cã n¨ng lùc ph¸t hiÖn vÊn ®Ò vµ tù gi¶i
quyÕt vÊn ®Ò lµ mét trong nh÷ng nhiÖm vô träng t©m cña ngµnh gi¸o dôc.
M©u thuÉn gi÷a yªu cÇu ®µo t¹o con ngêi míi x©y dùng x· héi c«ng nghiÖp
ho¸ vµ thùc tr¹ng l¹c hËu cña ph¬ng ph¸p d¹y häc ®· lµm n¶y sinh thóc ®Èy c«ng
cuéc ®æi míi ph¬ng ph¸p d¹y häc ë tÊt c¶ c¸c cÊp häc trong ngµnh gi¸o dôc .
Ph¬ng ph¸p d¹y häc kh«ng ph¶i lµ b¶n th©n ho¹t ®éng vµ øng xö cña GV ë
b×nh diÖn xem xÐt riªng lÎ, cô thÓ mµ theo NguyÔn B¸ Kim:
“Ph¬ng ph¸p d¹y häc lµ h×nh ¶nh kh¸i qu¸t ho¸ nh÷ng ho¹t ®éng øng xö nµo
®ã cña GV. H×nh ¶nh nµy thêng ®îc h×nh thµnh do ph¶n ¸nh nh÷ng ho¹t ®éng
øng xö thµnh c«ng cña GV trong qu¸ tr×nh d¹y häc vµ ph¶n ¸nh nh÷ng thµnh tùu
cña khoa häc gi¸o dôc hoÆc nh÷ng khoa häc kh¸c th«ng qua khoa häc gi¸o dôc...
Ph¬ng ph¸p d¹y häc lµ ph¬ng tiÖn ®Ó ®¹t môc ®Ých d¹y häc” .
Trong nh÷ng n¨m gÇn ®©y t tëng d¹y häc chñ ®¹o ®îc ph¸t biÓu díi nhiÒu
h×nh thøc kh¸c nhau nh “lÊy HS lµm trung t©m”,“ph¸t huy tÝnh tÝch cùc”, “ph¬ng
ph¸p d¹y häc tÝch cùc”, “tÝch cùc ho¸ ho¹t ®éng häc tËp”, “Ho¹t ®éng ho¸ ngêi
häc” ....
§Þnh híng ®æi míi ph¬ng ph¸p d¹y häc hiÖn nay lµ tæ chøc cho häc sinh
häc tËp trong ho¹t ®éng vµ b»ng ho¹t ®éng tù gi¸c, tÝch cùc vµ s¸ng t¹o.
+ Lµm thÕ nµo ®Ó tÝch cùc ho¸ ho¹t ®éng häc tËp cña häc sinh ?
3
Chuyªn ®Ò BDHSG To¸n THCS
Nhµ t©m lÝ häc I.X.Iakimanxkai cho r»ng: Nhµ trêng cÇn trang bÞ cho HS
hai hÖ thèng tri thøc: 1. VÒ hiÖn thùc ®èi tîng; 2. VÒ néi dung c¸ch thøc thùc
hiÖn c¸c hµnh ®éng trÝ tuÖ, ®¶m b¶o viÖc n¾m v÷ng c¸c tri thøc khoa häc vÒ
hiÖn thùc ®èi tîng ®ã.
C¸c tri thøc lo¹i mét ®îc ph¶n ¸nh trong SGK, cßn c¸c tri thøc lo¹i hai ®îc h×nh
thµnh chñ yÕu ë HS b»ng con ®êng tù ph¸t. ë ®ã tri thøc lo¹i hai lµ c¸c thñ ph¸p
cña häc tËp nh: tri thøc logic (ph©n tÝch, so s¸nh, kh¸i qu¸t ho¸, ph©n lo¹i…); tri
thøc tæ chøc hîp lÝ c¸c qu¸ tr×nh nhËn thøc kh¸c nhau…
Lerner I.Ia. cßn thªm vµo ®ã hai hÖ thèng n÷a: Kinh nghiÖm ho¹t ®éng s¸ng
t¹o vµ kinh nghiÖm th¸i ®é t×nh c¶m.
C¸c nhµ lÝ luËn d¹y häc P.I. Pitcaxixti,B.I. C«r«tiaiev kh¼ng ®Þnh: t¬ng øng
víi hai lo¹i ho¹t ®éng nhËn thøc t¸i t¹o vµ t×m tßi, s¸ng t¹o cña HS th× cã hai lo¹i
th«ng tin t¸i hiÖn vµ dù ®o¸n. Th«ng tin t¸i hiÖn lµ nh÷ng tri thøc HS lÜnh héi ë
d¹ng cã s½n, th«ng qua viÖc ghi nhËn vµ t¸i hiÖn l¹i. Th«ng tin dù ®o¸n lµ c¸c tri
thøc häc tËp ®îc HS kh«i phôc l¹i b»ng c¸ch thiÕt kÕ, t×m kiÕm vµ kiÓm tra tÝnh
®óng ®¾n cña ®iÒu dù ®o¸n. Trong khi ho¹t ®éng t¸i hiÖn chØ cã mét ph¬ng ¸n
vµ viÖc thùc hiÖn nã chÝnh x¸c lu«n dÉn ®Õn kÕt qu¶, th× ho¹t ®éng t×m tßi
s¸ng t¹o l¹i dùa vµo nh÷ng th«ng tin Èn tµng, cha têng minh. HS kiÓm tra dù ®o¸n
trªn c¬ së t×m kiÕm vµ lùa chän ph¬ng ¸n cã kh¶ n¨ng nhÊt trong hÖ thèng kiÕn
thøc ®· cã cña m×nh.
Dùa vµo kÕt qu¶ nghiªn cøu cña P.I. Pitcaxixti, B.I. C«r«tiaiev cã hai c¸ch
chiÕm lÜnh kiÕn thøc:
1. T¸i hiÖn kiÕn thøc: ®Þnh híng ®Õn ho¹t ®éng t¸i t¹o, ®îc x©y dùng
trªn c¬ së HS lÜnh héi c¸c tiªu chuÈn, h×nh mÉu cã s½n.
2. T×m kiÕm kiÕn thøc: ®Þnh híng ®Õn ho¹t ®éng c¶i t¹o tÝch cùc, dÉn
®Õn viÖc “ph¸t minh” kiÕn thøc vµ kinh nghiÖm ho¹t ®éng .
Nh vËy, PPDH nµo ®¶m b¶o phèi hîp gi÷a c¸ch d¹y t¸i hiÖn kiÕn thøc vµ t×m
kiÕm kiÕn thøc, trong ®ã tËn dông c¬ héi vµ ®iÒu kiÖn ®Ó c¸ch d¹y t×m kiÕm
kiÕn thøc chiÕm u thÕ, ®ång thêi kÕt hîp hµi hoµ víi tÝnh s½n sµng häc tËp cña
HS, th× vÒ c¬ b¶n PPDH ®ã cã kh¶ n¨ng tÝch cùc ho¸ ho¹t ®éng häc tËp cña HS
3. Dạy luyện tập toán cho học sinh
4
Chuyªn ®Ò BDHSG To¸n THCS
Qu¸ tr×nh d¹y häc lµ mét qu¸ tr×nh t©m lý. Trong qu¸ tr×nh häc tËp HS ph¶i
c¶m gi¸c, tri gi¸c, vËn dông trÝ nhí, t×nh c¶m, ý chÝ...
Trong nh÷ng n¨m gÇn ®©y, t©m lý häc ®· chó ý vµo “d¹y häc ph¸t triÓn”: D¹y
häc ®i tríc sù ph¸t triÓn, ®iÒu ®ã cã nghÜa lµ d¹y häc ph¶i tiÕn hµnh trong ®iÒu
kiÖn dù kiÕn ®îc møc ®é ph¸t triÓn cña HS cao h¬n hiÖn t¹i. Quan ®iÓm d¹y häc
®i tríc sù ph¸t triÓn vµ kÐo theo sù ph¸t triÓn ®îc coi lµ quan ®iÓm khoa häc vµ
c¸ch m¹ng.
Theo L.X. Vygotski, d¹y häc ph¶i theo ®óng chøc n¨ng cña nã, ph¶i ®i tríc sù
ph¸t triÓn, nã sÏ thóc ®Èy, kÐo theo sù ph¸t triÓn ®i lªn. MÊu chèt cña d¹y häc
ph¸t triÓn lµ x¸c ®Þnh ®óng c¸c tr×nh ®é ph¸t triÓn cña häc sinh: Tr×nh ®é ph¸t
triÓn hiÖn thêi vµ kh¶ n¨ng ph¸t triÓn gÇn nhÊt. Møc ®é hiÖn t¹i ®îc biÓu hiÖn
qua qu¸ tr×nh HS ®éc lËp gi¶i quyÕt nhiÖm vô, kh«ng cÇn sù trî gióp tõ bªn
ngoµi. Cßn kh¶ n¨ng ph¸t triÓn gÇn nhÊt ®îc thÓ hiÖn trong t×nh huèng HS hoµn
thµnh nhiÖm vô khi cã sù hîp t¸c, gióp ®ì cña ngêi kh¸c. Tõ ®ã «ng ®a ra nguyªn
lý d¹y häc ph¶i t¸c ®éng vµo vïng ph¸t triÓn gÇn nhÊt, cã nghÜa lµ ph¬ng ph¸p d¹y
häc tu©n theo nguyªn t¾c t«n träng kinh nghiÖm ®· cã cña HS vµ t¨ng dÇn møc
®é khã kh¨n.
Nh vËy d¹y häc kh«ng bÞ ®éng chê sù ph¸t triÓn, mµ ngîc l¹i ph¸t triÓn c¸c
chøc n¨ng t©m lÝ. VÊn ®Ò ®éng c¬ häc tËp, høng thó nhËn thøc cã ý nghÜa rÊt
quan träng ®Õn hiÖu qu¶ cña qu¸ tr×nh d¹y häc.
§Ó ®¶m b¶o thµnh c«ng cña qu¸ tr×nh d¹y häc, thÇy gi¸o ph¶i ®Æc biÖt
chó ý tíi mÆt t©m lý trong qu¸ tr×nh d¹y häc.
- D¹y häc lµ mét qu¸ tr×nh x· héi, trong ®ã cã sù t¬ng t¸c gi÷a ngêi vµ ngêi,
gi÷a ngêi vµ x· héi. HiÓu ®îc tÝnh x· héi cña d¹y häc vµ ¶nh hëng cña x· héi ®èi
víi nhµ trêng sÏ gióp ngêi d¹y ®iÒu khiÓn ®îc qu¸ tr×nh d¹y häc.
Sù gièng nhau vµ kh¸c nhau vÒ yªu cÇu x· héi, vÒ sù ph¸t triÓn nh©n c¸ch
cña tõng ngêi ®ßi hái mét qu¸ tr×nh d¹y häc thèng nhÊt cïng víi biÖn ph¸p ph©n
ho¸, do vËy trong ho¹t ®éng cña m×nh ngêi thÇy cÇn quan t©m ®Õn kinh nghiÖm
sèng vµ ®iÒu kiÖn häc tËp thùc tÕ cña HS ®Ó x©y dùng kÕ ho¹ch vµ néi dung
d¹y häc thÝch hîp.
Tãm l¹i, c¨n cø vµo nhËn thøc vÒ qu¸ tr×nh d¹y häc trong giai ®o¹n hiÖn nay
(qu¸ tr×nh nhËn thøc, qu¸ tr×nh t©m lý vµ qu¸ tr×nh x· héi), hÖ thèng BT cÇn ph¶i
5
Chuyªn ®Ò BDHSG To¸n THCS
ph¶n ¸nh tÝch cùc vµ cã chän läc c¸c tri thøc, ph¬ng ph¸p, kü n¨ng ... liªn quan
chÆt chÏ ®Õn ho¹t ®éng to¸n häc, thóc ®Èy c¸c chøc n¨ng t©m lý, høng thó nhËn
thøc vµ chó ý ®Õn kinh nghiÖm sèng vµ ®iÒu kiÖn thùc tÕ cña häc sinh.
TiÕn tr×nh gi¶i bµi tËp to¸n
Gi¶i BTT lµ thùc hiÖn mét lo¹t c¸c ho¹t ®éng liªn tôc vµ kh¸ phøc t¹p v× BTT
lµ sù kÕt hîp ®a d¹ng nhiÒu kh¸i niÖm , quan hÖ to¸n häc... V× vËy ®Ó gi¶i ®ược
BTT ®ßi hái häc sinh n¾m ch¾c c¸c kh¸i niÖm, ®Þnh lý, quy t¾c... c¸c kiÕn thøc
trong mèi quan hÖ to¸n häc cña ch¬ng tr×nh ®· häc.
Theo V.M. Bra®ix¬ “BTT cã thÓ xem lµ ®· ®îc gi¶i chØ sau khi ®· t×m
®ược lêi gi¶i ®¶m b¶o c¸c ®iÒu kiÖn: Kh«ng sai sãt, cã lËp luËn khoa häc, mang
tÝnh toµn diÖn vµ tèi ưu”.
Theo Polya G. : “Gi¶i mét BTT chóng ta ph¶i lËp ®ược mét lưîc ®å x¸c ®Þnh
m¹nh l¹c nh÷ng thao t¸c (l«gÝc, to¸n häc hay thùc tiÔn) b¾t ®Çu tõ gi¶ thiÕt vµ
kÕt thóc b»ng kÕt luËn, dÉn d¾t tõ c¸c ®èi tượnng mµ ta cã trong tay ®Õn ®èi t-
ưîng ta muèn ®¹t tíi”.
+ Polya G. quan niÖm gi¶i mét BTT lµ mét qu¸ tr×nh t×m kiÕm nh÷ng ho¹t
®éng thÝch hîp ®Ó ®¹t kÕt qu¶. Theo «ng tiÕn tr×nh gi¶i mét BTT gåm 4 bước:
- HiÓu râ BTT (understanding the problem)
- X©y dùng chư¬ng tr×nh gi¶i (devising a plan)
- Thùc hiÖn chư¬ng tr×nh gi¶i (carrying out the plan)
- KiÓm tra lêi gi¶i t×m ®ược (looking back)
Bước 1: HiÓu râ bµi tËp to¸n
- X¸c ®Þnh ®èi tưîng vµ c¸c ®iÒu kiÖn vµ hÖ thèng hµnh ®éng, lµm râ c¸c
mèi quan hÖ ë gi¶ thiÕt, mèi quan hÖ gi÷a gi¶ thiÕt vµ kÕt luËn. X¸c ®Þnh ®ùîc
d¹ng BTT, xem xÐt cÊu tróc cña BTT tõ ®ã suy nghÜ hùíng gi¶i BTT ®ã.
Bíc 2: X©y dùng chư¬ng tr×nh gi¶i
- Tõ sù ph©n tÝch mèi quan hÖ gi÷a c¸c yÕu tè cña BTT, tõ suy nghÜ hùíng
gi¶i ë bùíc 1, HS t×m con ®ùêng cô thÓ, kh¶ n¨ng ®¹t ®îc môc ®Ých, ®Þnh hùíng
c¸c hµnh ®éng tiÕn tíi qu¸ tr×nh gi¶i BTT.
- Qu¸ tr×nh nµy kÕt hîp gi÷a logic h×nh thøc (viÖc v¹ch ra “cÊu tróc” cña kÕ
ho¹ch) vµ l«gic biÖn chøng (chØ ra tÝnh cô thÓ, tÝnh kh¶ thi vµ phù¬ng thøc thùc
hiÖn kÕ ho¹ch).
6
Chuyªn ®Ò BDHSG To¸n THCS
Bíc 3: Thùc hiÖn chù¬ng tr×nh gi¶i
- KÕ ho¹ch gi¶i vÉn cßn ë ý tùëng, HS ph¶i thùc hiÖn mét hÖ thèng hµnh
®éng phï hîp víi nh÷ng chi tiÕt cô thÓ cña BTT.
- Sö dông c¸c thao t¸c tư duy nh÷ng lËp luËn logic ®Ó thùc hiÖn kÕ ho¹ch.
- Cã thÓ gi¶i BTT theo nhiÒu c¸ch gi¶i kh¸c nhau, t×m ra c¸ch gi¶i tèi ưu.
- ë b ư íc nµy thao t¸c tư duy logÝc, ho¹t ®éng ng«n ng÷ ®ãng vai trß quan
träng.
Bíc 4: Kh¶o s¸t lêi gi¶i t×m ®ưîc
- C«ng viÖc ®ưîc tiÕn hµnh trong suèt qu¸ tr×nh gi¶i BTT, viÖc kiÓm tra
nh»m chÝnh x¸c ho¸ lêi gi¶i (c¸c bưíc suy luËn, c¸c kh©u tÝnh to¸n...).
- Qua kh¶o s¸t lêi gi¶i cßn rót ra ®îc kinh nghiÖm cho HS, gi¶i bµi BTT lµ
ph¬ng tiÖn häc tËp. Tõ kh¶o s¸t lêi gi¶i HS cã thÓ hîp thøc ho¸ BTT thµnh tri thøc
vµ kinh nghiÖm cña b¶n th©n.
* Gi¶i bµi tËp to¸n theo ®Þnh híng ang«rit vµ ¬ristic
“C¸c ang«rit tån t¹i dưíi nhiÒu h×nh thøc biÓu diÔn kh¸c nhau, ng«n ng÷ tù
nhiªn, ng«n ng÷ to¸n häc, s¬ ®å khèi, ng«n ng÷ phư¬ng tr×nh, lËp tr×nh..., ang«rit
(thuËt to¸n, thuËt gi¶i) lµ mét b¶n quy ®Þnh nh÷ng thao t¸c cÇn thùc hiÖn ®Ó gi¶i
mét BTT” .
“ThuËt to¸n ®ưîc hiÓu nh mét quy t¾c mµ tõ nh÷ng chØ dÉn râ rµng vµ
chÝnh x¸c ®Ó ngưêi (hay m¸y) thùc hiÖn mét lo¹t thao t¸c nh»m ®¹t ®îc môc ®Ých
®Æt ra hay gi¶i mét líp BTT nhÊt ®Þnh”.
R. §Ò C¸c ®· nghÜ ®Õn mét phư¬ng ph¸p toµn n¨ng ®Ó gi¶i mäi bµi tËp,
Leibnis th× ®a ra ý niÖm râ rµng vÒ mét ph¬ng ph¸p toµn mü ®Ó gi¶i to¸n. Polya
G. ®· rÊt ®Ò cao viÖc h×nh thµnh vµ ph¸t triÓn n¨ng lùc s¸ng t¹o qua gi¶i bµi tËp
to¸n nhng «ng ®· kh¼ng ®Þnh “T×m kiÕm mét phư¬ng ph¸p toµn n¨ng vµ toµn mü
ch¼ng mang l¹i kÕt qu¶ g× h¬n ®i t×m mét viªn ®¸ thÇn kú, ®Ó cã thÓ biÕn mäi
kim lo¹i thµnh vµng” . Như vËy kh«ng thÓ cã phư¬ng ph¸p ®Ó gi¶i tÊt c¶ c¸c BTT,
ngoµi nh÷ng d¹ng to¸n cã thÓ dïng phư¬ng ph¸p theo ®Þnh hưíng ang«rit cßn ph¶i
sö dông phư¬ng ph¸p ¬ristic.
ThuËt ng÷ ¬ristic cã nguån gèc Hy L¹p lµ “¬rªca” ®ưîc hiÓu lµ sù t×m tßi, t×m
®o¸n, s¸ng t¹o…
7
Chuyªn ®Ò BDHSG To¸n THCS
“ ¥ristic ®îc hiÓu lµ tæng thÓ nãi chung c¸c quy t¾c phư¬ng ph¸p kh¸i qu¸t tõ
kinh nghiÖm qu¸ khø ®ưîc dïng trong qu¸ tr×nh nghiªn cøu ph¸t hiÖn, s¸ng t¹o ra
c¸i míi”.
Gi¶i to¸n theo ®Þnh hưíng ¥ristÝc mang tÝnh chÊt “t×m ®o¸n” thưêng dïng ®Ó
gi¶i nh÷ng BTT mang tÝnh chÊt lµ mét vÊn ®Ò, t×m hiÓu vµ ph¸t hiÖn ra vÊn
®Ò, t×m c¸ch gi¶i quyÕt vÊn ®Ò ®ã lµ ho¹t ®éng to¸n häc cÇn thiÕt.
Cách thức học phương pháp tìm lời giải bài toán
Học phương pháp tìm lời giải không phải là học một thuật giải mà học những kinh
nghiệm giải toán mang tính chất tìm tòi, phát hiện. Do đó cách thức học phương pháp để tìm
lời giải bài toán yêu cầu:
- Thông qua việc giải những bài toán cụ thể, học sinh cần nắm được các bước tìm
lời giải bài toán và có ý thức vận dụng các bước đó trong quá trình giải toán.
- Cũng thông qua việc giải những bài toán cụ thể, giáo viên cần đặt ra cho học sinh
những câu hỏi gợi ý, tạo tình huống để các em tìm tòi, dự đoán, phát hiện và cuối cùng tìm
ra lời giải bài toán.
Như vậy, quá trình học sinh học phương pháp tìm lời giải bài toán là một quá trình
biến những tri thức, phương pháp tổng quát thành kinh nghiệm giải toán của bản thân
mình thông qua việc giải hàng loạt bài toán cụ thể. do đó đòi hỏi người giải toán phải có
một chặng đường lao động tích cực, có nhiều yếu tố sáng tạo.
Cã ý kiÕn cho r»ng “d¹y luyÖn tËp to¸n gièng nh luyÖn tËp qu©n sù”; ®èi víi
®èi tîng häc sinh kh¸, giái thÇy chØ híng dÉn c¸c thao t¸c vµ HS tù m×nh lµm ®îc;
®èi víi häc sinh trung b×nh thÇy lµm mÉu c¸c ®éng t¸c vµ häc sinh lµm theo ®îc;
®èi víi HS yÕu thÇy gi¸o ph¶i cho HS lµm tõng ®éng t¸c theo m×nh cho ®Õn lóc
HS tù lµm ®îc mµ kh«ng cã thÇy lµm mÉu ë phÝa tríc.
Qua thùc tÕ d¹y häc vµ c«ng t¸c qu¶n lý d¹y vµ häc, chóng t«i ®a ra mét sè
®Þnh híng cho tiÕt d¹y luyÖn tËp h×nh häc nh sau:
+ Ph©n lo¹i c¸c bµi tËp ë SGK, SBT (BTT còng cè kiÕn thøc cña bµi häc,
BTT «n kiÕn thøc cña bµi häc tríc, BT bæ sung lý thuyÕt, BT kh¾c s©u kiÕn
thøc).
+ C¨n cø vµo ®èi tîng HS cña líp gi¸o viªn gi¶ng d¹y ®Ó lùa chän mét trong
c¸c ý tëng :
- Víi ®èi tîng häc sinh cña líp lµ häc sinh häc trung b×nh vµ yÕu m«n to¸n:
. Thêng dïng c¸c BTT «n tËp kiÕn thøc ®Ó kiÓm tra nhanh ®Çu tiÕt luyÖn
tËp (nh÷ng bµi cha ®îc sö dông sau phÇn häc lý thuyÕt).
8
Chuyªn ®Ò BDHSG To¸n THCS
. Híng dÉn häc sinh gi¶i c¸c BTT bæ sung lý thuyÕt vµ BTT kh¾c s©u kiÕn
thøc (c¸c BT ë SGK vµ SBT ®îc lùa chän), qua viÖc sö dông c¸c BTT ®ã, gióp häc
sinh t×m ®îc quy tr×nh hoÆc ®Þnh híng gi¶i c¸c BTT cïng d¹ng.
- Víi ®èi tîng lµ häc sinh trung b×nh vµ trung b×nh kh¸:
.C¬ b¶n häc sinh ®· gi¶i ®îc c¸c BTT thÇy gi¸o ra vÒ nhµ chuÈn bÞ nªn nÕu
®Õn líp trong tiÕt d¹y luyÖn tËp thÇy gi¸o híng dÉn gi¶i c¸c BTT ®ã sÏ kh«ng t¹o
ra ®îc sù míi mÏ, dÉn ®Õn HS kh«ng høng thó trong häc tËp.
Trong trêng hîp nµy gi¸o viªn chän mét BTT t¬ng tù vµ thªm c¸c c©u hái
nh»m x©u chuçi c¸c BTT, HS ®· ®îc chuÈn bÞ. Thùc tÕ cho thÊy HS høng thó
trong häc tËp vµ tiÕt d¹y thµnh c«ng h¬n nhiÒu.
- §èi tîng lµ häc sinh kh¸, giái:
.ThÇy gi¸o chØ kiÓm tra nhanh c¸c BTT cã tÝnh chÊt còng cè kiÕn thøc.
Dïng BT ®iÓn h×nh luyÖn tËp víi c¸c ®Þnh híng kh¸c nhau nh»m t¹o ra nhiÒu
c¸ch gi¶i (nÕu cã thÓ), tõ BTT ®· cã t¹o ra BTT míi b»ng c¸c ho¹t ®éng t¬ng tù, t-
¬ng tù ho¸, kh¸i qu¸t ho¸, lËt ngîc vÊn ®Ò… t¹o thµnh mét sè BTT nh»m ph¸t triÓn
t duy s¸ng t¹o cho HS.
Lấy một số ví dụ về dạy luyện tập hình học cho học sinh giỏi.
RÌn luyÖn ho¹t ®éng to¸n häc cho häc sinh kh¸, giái
Theo NguyÔn B¸ Kim “d¹y häc ph©n ho¸ xuÊt ph¸t tõ sù biÖn chøng cña
thèng nhÊt vµ ph©n ho¸, tõ yªu cÇu thùc hiÖn tèt c¸c môc tiªu d¹y häc ®èi víi tÊt
c¶ mäi häc sinh, ®ång thêi ph¸t triÓn tèi ®a vµ tèi ưu nh÷ng kh¶ n¨ng c¸ nh©n.
ViÖc kÕt hîp gi÷a gi¸o dôc ®¹i trµ vµ gi¸o dôc mòi nhän gi÷a phæ cËp víi n©ng
cao trong d¹y häc to¸n phæ th«ng cÇn tiÕn hµnh theo c¸c tư tưởng chñ ®¹o sau:
- LÊy tr×nh ®é ph¸t triÓn chung cña HS trong líp lµm nÒn t¶ng…
- Sö dông biÖn ph¸p ph©n ho¸ ®a diÖn häc sinh yÕu kÐm lªn tr×nh ®é
chung…
- Cã nh÷ng néi dung bæ sung vµ biÖn ph¸p ph©n ho¸ gióp häc sinh kh¸ giái
®¹t ®îc nh÷ng yªu cÇu n©ng cao trªn c¬ së ®¹t ®ưîc nh÷ng yªu cÇu c¬ b¶n”.
Chóng ta quan t©m ®Õn ý tưëng thø ba: “Cã nh÷ng néi dung bæ sung vµ
biÖn ph¸p ph©n ho¸ gióp häc sinh kh¸, giái ®¹t ®ược nh÷ng yªu cÇu n©ng cao trªn
c¬ së ®¹t ®ược nh÷ng yªu cÇu c¬ b¶n”
9
Chuyªn ®Ò BDHSG To¸n THCS
Víi c¸ch d¹y häc ph©n ho¸ như ®· nªu lµ ph©n ho¸ néi t¹i (ph©n ho¸ trong)
trong mét líp häc. Thùc tÕ d¹y häc ë c¸c trêng THCS ®ang cßn phæ biÕn ph©n
ho¸ ngoµi, nghÜa lµ cã nh÷ng líp tr×nh ®é HS kh¸ h¬n, trong nh÷ng n¨m gÇn ®©y
c¸c trêng chuyªn, líp chän ë THCS kh«ng tån t¹i n÷a, nhưng víi tÝnh chÊt gi¸o dôc
cã tÝnh ®¹i chóng vµ phæ cËp nh hiÖn nay th× viÖc ph©n ho¸ ngoµi vÉn lµ phæ
biÕn (v× thuËn lîi h¬n trong qu¸ tr×nh d¹y häc, ë bËc THPT ph©n ban còng lµ
h×nh thøc ph©n ho¸ ngoµi).
C¨n cø vµo c¸c ho¹t ®éng to¸n häc liªn quan mËt thiÕt ®Õn néi dung m«n to¸n
ë trưêng phæ th«ng lµ: NhËn d¹ng vµ thÓ hiÖn; nh÷ng ho¹t ®éng to¸n häc phøc
hîp; nh÷ng ho¹t ®éng trÝ tuÖ phæ biÕn trong to¸n häc; nh÷ng ho¹t ®éng trÝ tuÖ
chung vµ nh÷ng ho¹t ®éng ng«n ng÷, tõ ®ã trong qu¸ tr×nh d¹y häc gi¸o viªn trong
mäi t×nh huèng dï tưêng minh hay Èn tµng còng ®Òu cã ý tưëng gãp phÇn rÌn
luyÖn ho¹t ®éng to¸n häc cho häc sinh.
10
Chuyªn ®Ò BDHSG To¸n THCS
b.C¸c chuyªn ®Ò båi dìng hsg to¸n thcs
Chuyªn ®Ò 1:
PhÇn I: Sè chÝnh ph¬ng
I- §Þnh nghÜa: Sè chÝnh ph¬ng lµ sè b»ng b×nh ph¬ng ®óng cña mét sè nguyªn.
II- tÝnh chÊt:
1- Sè chÝnh ph¬ng chØ cã thÓ cã ch÷ sè tËn cïng b»ng 0, 1, 4, 5, 6, 9; kh«ng
thÓ cã ch÷ tËn cïng b»ng 2, 3, 7, 8.
2- Khi ph©n tÝch ra thõa sè nguyªn tè, sè chÝnh ph¬ng chØ chøa c¸c thõa sè
nguyªn tè víi sè mò ch½n.
3- Sè chÝnh ph¬ng chØ cã thÓ cã mét trong hai d¹ng 4n hoÆc 4n+1. Kh«ng cã
sè chÝnh ph¬ng nµo cã d¹ng 4n + 2 hoÆc 4n + 3 (n ∈ N).
4- Sè chÝnh ph¬ng chØ cã thÓ cã mét trong hai d¹ng 3n hoÆc 3n +1. Kh«ng cã
sè chÝnh ph¬ng nµo cã d¹ng 3n + 2 ( n ∈ N ).
5- Sè chÝnh ph¬ng tËn cïng b»ng 1, 4 hoÆc 9 th× ch÷ sè hµng chôc lµ ch÷ sè
ch½n.
Sè chÝnh ph¬ng tËn cïng b»ng 5 th× ch÷ sè hµng chôc lµ 2.
Sè chÝnh ph¬ng tËn cïng b»ng 6 th× ch÷ sè hµng chôc lµ ch÷ sè lÎ.
6- Sè chÝnh ph¬ng chia hÕt cho 2 th× chia hÕt cho 4.
Sè chÝnh ph¬ng chia hÕt cho 3 th× chia hÕt cho 9
Sè chÝnh ph¬ng chia hÕt cho 5 th× chia hÕt cho 25
Sè chÝnh ph¬ng chia hÕt cho 8 th× chia hÕt cho 16.
III- Mét sè d¹ng bµi tËp vÒ sè chÝnh ph¬ng.
A- D¹ng 1: chøng minh mét sè lµ sè chÝnh ph¬ng.
Bµi 1: Chøng minh r»ng mäi sè nguyªn x, y th×:
A= (x + y)(x + 2y)(x + 3y)(x + 4y) + 4
y lµ sè chÝnh ph¬ng.
Gi¶i : Ta cã A = (x + y)(x + 2y)(x + 3y)(x + 4y) + 4
y
= ( 2 2 2 2 4
5 4 )( 5 6 )x xy y x xy y y+ + + + +
§Æt 2 2
5 5 ( )x xy y t t Z+ + = ∈ th×
A = ( 2 2 4 2 4 4 2 2 2 2
)( ) ( 5 5 )t y t y y t y y t x xy y− + + = − + = = + +
V× x, y, z ∈ Z nªn 2 2 2 2
, 5 , 5 5 5x Z xy Z y Z x xy y Z∈ ∈ ∈ ⇒ + + ∈
VËy A lµ sè chÝnh ph¬ng.
Bµi 2: Chøng minh tÝch cña 4 sè tù nhiªn liªn tiÕp céng 1 lu«n lµ sè chÝnh ph-
¬ng.
Gi¶i : Gäi 4 sè tù nhiªn, liªn tiÕp ®ã lµ n, n+1, n+2, n+3 (n ∈ Z). Ta cã:
n(n + 1)(n + 2)(n + 3) + 1 = n . ( n + 3)(n + 1)(n + 2) + 1
= ( 2 2
3 )( 3 2) 1 (*)n n n n+ + + +
§Æt 2
3 ( )n n t t N+ = ∈ th× (*) = t(t + 2) + 1 = t2
+ 2t + 1 = (t + 1)2
11
Chuyªn ®Ò BDHSG To¸n THCS
= (n2
+ 3n + 1)2
V× n ∈ N nªn n2
+ 3n + 1 ∈ N. VËy n(n + 1)(n + 2)(+ 3) + 1 lµ sè chÝnh ph¬ng.
Bµi 3: Cho S = 1.2.3 + 2.3.4 + 3.4.5 + ...+ k(k + 1)(k + 2)
Chøng minh r»ng 4S + 1 lµ sè chÝnh ph¬ng.
Gi¶i : Ta cã: k(k + 1)(k + 2) =
1
4
k (k + 1)(k + 2). 4=
1
4
k(k + 1)(k + 2). [ ]( 3) ( 1)k k+ − −
=
1
4
k(k + 1)(k + 2)(k + 3) -
1
4
k(k + 1)(k + 2)(k - 1)
=> 4S =1.2.3.4 - 0.1.2.3 + 2.3.4.5 - 1.2.3.4 + . . . + k(k + 1)(k + 2)(k + 3)
- k(k + 1)(k + 2)(k - 1) = k(k + 1)(k + 2)(k + 3)
=> 4S + 1 = k(k + 1)(k + 2)(k + 3) + 1
Theo kÕt qu¶ bµi 2 => k(k + 1)(k + 2)(k + 3) + 1 lµ sè chÝnh ph¬ng.
Bµi 4: Cho d·y sè 49; 4489; 444889; 44448889; . . .
- D·y sè trªn ®îc x©y dùng b»ng c¸ch thªm sè 48 vµo gi÷a c¸c ch÷ sè ®øng tríc vµ
®øng sau nã. Chøng minh r»ng tÊt c¶ c¸c sè cña d·y trªn ®Òu lµ sè chÝnh ph¬ng.
Ta cã 44 ...488...89 = 44...488...8 + 1 = 44...4 . 10n
+ 8 . 11 ... 1 + 1
n ch÷ sè 4 n - 1 ch÷ sè 8 n ch÷ sè 4 n ch÷ sè 8 n ch÷ sè 4 n ch÷ sè 1
= 4.
10 1 10 1
.10 8. 1
9 9
n n
n− −
+ +
=
2 2
4.10 4.10 8.10 8 9 4.10 4.10 1
9 9
n n n n n
− + − + + +
=
=
2
2.10 1
3
n
 +
 ÷
 
Ta thÊy 2.10n
+ 1 = 200...01 cã tæng c¸c ch÷ sè chia hÕt cho 3 nªn nã chia hÕt
cho 3
n - 1 ch÷ sè 0
=>
2
2.10 1
3
n
 +
 ÷
 
∈ Z hay c¸c sè cã d¹ng 44 ... 488 ... 89 lµ sè chÝnh ph¬ng.
C¸c bµi t¬ng tù:
Chøng minh r»ng sè sau ®©y lµ sè chÝnh ph¬ng.
A = 11 ... 1 + 44 ... 4 + 1
2n ch÷ sè 1 n ch÷ sè 4
B = 11 ... 1 + 11 . . .1 + 66 . . . 6 + 8
2n ch÷ sè 1 n+1 ch÷ sè 1 n ch÷ sè 6
C= 44 . . . 4 + 22 . . . 2 + 88 . . . 8 + 7
2n ch÷ sè 4 n+1 ch÷ sè 2 n ch÷ sè 8
D = 22499 . . .9100 . . . 09
12
Chuyªn ®Ò BDHSG To¸n THCS
n-2 ch÷ sè 9 n ch÷ sè 0
E = 11 . . .155 . . . 56
n ch÷ sè 1 n-1 ch÷ sè 5
KÕt qu¶: A=
2 2 2
10 2 10 8 2.10 7
; ;
3 3 3
n n n
B C
     + + +
= = ÷  ÷  ÷
     
D = (15.10n
- 3)2
E =
2
3
210





 +n
Bµi 5: Chøng minh r»ng tæng c¸c b×nh ph¬ng cña 5 sè tù nhiªn liªn tiÕp kh«ng
thÓ lµ mét sè chÝnh ph¬ng.
Gäi 5 sè tù nhiªn liªn tiÕp ®ã lµ n - 2, n - 1, n +1, n + 2 ( n ∈ N, n >2).
Ta cã (n - 2)2
+ ( n - 1)2
+ n2
+ (n + 1)2
+ (n + 2)2
= 5 . (n2
+ 2)
V× n2
kh«ng thÓ tËn cïng bëi 3 hoÆc 8 do ®ã n2
+ 2 kh«ng thÓ chia hÕt cho 5
=> 5. (n2
+ 2) kh«ng lµ sè chÝnh ph¬ng hay A kh«ng lµ sè chÝnh ph¬ng.
Bµi 6: Chøng minh r»ng sè cã d¹ng n6
- n4
+ 2n3
+ 2n2
trong ®ã n ∈ N vµ n >1
kh«ng ph¶i lµ sè chÝnh ph¬ng.
n6
- n 4
+ 2n3
+ 2n2
= n2
. (n4
- n2
+ 2n +2) = n2
. [n2
(n-1)(n+1) +2(n+1)]
= n2
[(n+1)(n3
- n2
+ 2)] = n2
(n + 1) . [(n3
+ 1) - (n2
- 1)]
= n2
(n + 1)2
. (n2
- 2n + 2)
Víi n∈N, n > 1 th× n2
- 2n + 2 = ( n -1)2
+ 1 > ( n - 1)2
Vµ n2
- 2n + 2 = n2
- 2(n - 1) < n2
VËy (n - 1)2
< n2
- 2n + 2 < n2
=> n2
- 2n + 2 kh«ng ph¶i lµ mét sè chÝnh ph¬ng.
Bµi 7: Cho 5 sè chÝnh ph¬ng bÊt kú cã ch÷ sè hµng chôc kh¸c nhau cßn ch÷ sè
hµng ®¬n vÞ ®Òu lµ 6. Chøng minh r»ng tæng c¸c ch÷ sè hµng chôc cña 5 sè
chÝnh ph¬ng ®ã lµ mét sè chÝnh ph¬ng.
Ta biÕt mét sè chÝnh ph¬ng cã ch÷ sè hµng ®¬n vÞ lµ 6 th× ch÷ sè hµng chôc
cña nã lµ sè lÎ. V× vËy ch÷ sè hµng chôc cña 5 sè chÝnh ph¬ng ®ã lµ 1,3,5,7,9
khi ®ã tæng cña chóng b»ng 1 + 3 + 5 + 7 + 9 = 25 = 52
lµ sè chÝnh ph¬ng.
Bµi 8: Chøng minh r»ng tæng b×nh ph¬ng cña 2 sè lÎ bÊt kú kh«ng ph¶i lµ sè
chÝnh ph¬ng.
a vµ b lÎ nªn a = 2k + 1, b= 2m + 1 (Víi k, m ∈ N).
=> a2
+ b2
= (2k + 1)2
+ ( 2m + 1)2
= 4k2
+ 4k + 1 + 4m2
+ 4m + 1
= 4 (k2
+ k + m2
+ m) + 2
=> a2
+ b2
kh«ng thÓ lµ sè chÝnh ph¬ng.
Bµi 9: Chøng minh r»ng nÕu p lµ tÝch cña n (víi n > 1) sè nguyªn tè ®Çu tiªn
th× p - 1 vµ p + 1 kh«ng thÓ lµ c¸c sè chÝnh ph¬ng.
V× p lµ tÝch cña n sè nguyªn tè ®Çu tiªn nªn pM2 vµ p kh«ng thÓ chia hÕt cho 4
(1)
a- Gi¶ sö p + 1 lµ sè chÝnh ph¬ng. §Æt p + 1 = m2
( m ∈ N).
V× p ch½n nªn p + 1 lÎ => m2
lÎ => m lÎ.
§Æt m = 2k + 1 (k ∈ N). Ta cã m2
= 4k2
+ 4k + 1 => p + 1 = 4k2
+ 4k + 1
13
Chuyªn ®Ò BDHSG To¸n THCS
=> p = 4k2
+ 4k = 4k (k + 1) M 4 m©u thuÉn víi (1).
=> p + 1 kh«ng ph¶i lµ sè chÝnh ph¬ng.
b- p = 2.3.5... lµ sè chia hÕt cho 3 => p - 1 cã d¹ng 3k + 2.
=> p - 1 kh«ng lµ sè chÝnh ph¬ng.
VËy nÕu p lµ tÝch n (n >1) sè nguyªn tè ®Çu tiªn th× p - 1 vµ p + 1 kh«ng lµ sè
chÝnh ph¬ng.
Bµi 10: Gi¶ sö N = 1.3.5.7 . . . 2007. 2011
Chøng minh r»ng trong 3 sè nguyªn liªn tiÕp 2N - 1, 2N vµ 2N + 1 kh«ng cã sè
nµo lµ sè chÝnh ph¬ng.
a- 2N - 1 = 2.1.3.5.7 . . . 2011 - 1
Cã 2N M 3 => 2N - 1 = 3k + 2 (k ∈ N)
=> 2N - 1 kh«ng lµ sè chÝnh ph¬ng.
b- 2N = 2.1.3.5.7 . . . 2011 => 2N ch½n.
=> N lÎ => N kh«ng chia hÕt cho 2 vµ 2N M 2 nhng 2N kh«ng chia hÕt cho 4.
2N ch½n nªn 2N kh«ng chia cho 4 d 1 hoÆc d 3 => 2N kh«ng lµ sè chÝnh ph¬ng.
c- 2N + 1 = 2.1.3.5.7 . . . 2011 + 1
2N + 1 lÎ nªn 2N + 1 kh«ng chia hÕt cho 4
2N kh«ng chia hÕt cho 4 nªn 2N + 1 kh«ng chia cho 4 d 1.
=> 2N + 1 kh«ng lµ sè chÝnh ph¬ng.
Bµi 11: Cho a = 11 . . . 1 ; b = 100 . . . 05
2010 ch÷ sè 1 2009 ch÷ sè 0
Chøng minh 1ab + lµ sè tù nhiªn.
Gi¶i: b = 100 . . . 05 = 100 . . . 0 - 1 + 6 = 99 . . . 9 + 6 = 9a + 6
2009 ch÷ sè 0 2010 ch÷ sè 0 2010 ch÷ sè 9
⇒ ab + 1 = a(9a + 6) + 1 = 9a2
+ 6a + 1 = (3a + 1)2
⇒ Naaab ∈+=+=+ 13)13(1 2
B. d¹ng 2: t×m gi¸ trÞ cña biÕn ®Ó biÓu thøc lµ sè chÝnh ph¬ng
Bµi 1: T×m sè tù nhiªn n sao cho c¸c sè sau lµ sè chÝnh ph¬ng
a) n2
+ 2n + 12 b) n(n + 3)
c) 13n + 3 d) n2
+ n + 1589
Gi¶i:
a) V× n2
+ 2n + 12 lµ sè chÝnh ph¬ng nªn ®Æt n2
+ 2n + 12 = k2
(k ∈ N)
⇒ (n2
+ 2n + 1) + 11 = k2 ⇔ k2
– (n + 1)2
= 11 ⇔ (k + n + 1)(k – n - 1) = 11
NhËn xÐt thÊy k + n + 1 > k - n - 1 vµ chóng lµ nh÷ng sè nguyªn d¬ng, nªn ta cã thÓ
viÕt (k + n + 1) (k - n - 1) = 11.1 ⇔ k + n + 1 = 11 ⇔ k = 6
k - n – 1 = 1 n = 4
b) ®Æt n(n + 3) = a2
(n ∈ N) ⇒ n2
+ 3n = a2 ⇔ 4n2
+ 12n = 4a2
⇔ (4n2
+ 12n + 9) – 9 = 4a2
⇔ (2n + 3)2
– 4a2
= 9
⇔ (2n + 3 + 2a)(2n + 3 – 2a) = 9
14
Chuyªn ®Ò BDHSG To¸n THCS
NhËn xÐt thÊy 2n + 3 + 2a > 2n + 3 – 2a vµ chóng lµ nh÷ng sè nguyªn d¬ng, nªn ta cã
thÓ viÕt (2n + 3 + 2a)(2n + 3 – 2a) = 9.1 ⇔ 2n + 3 + 2a = 9 ⇔ n = 1
2n + 3 – 2a = 1 a = 2
c) §Æt 13n + 3 = y2
(y ∈ N) ⇒ 13(n - 1) = y2
– 16
⇔ 13(n - 1) = (y + 4)(y – 4)
⇒(y + 4)(y – 4) M 13 mµ 13 lµ sè nguyªn tè nªn y + 4 M 13 hoÆc y – 4 M 13
⇒ y = 13k ± 4 (víi k ∈ N)
⇒ 13(n - 1) = (13k ± 4)2
– 16 = 13k.(13k ± 8)
⇒13k2
± 8k + 1
VËy n = 13k2
± 8k + 1 (víi k ∈ N) th× 13n + 3 lµ sè chÝnh ph¬ng
d) §Æt n2
+ n + 1589 = m2
(m ∈ N) ⇒ (4n2
+ 1)2
+ 6355 = 4m2
⇔ (2m + 2n + 1) (2m – 2n – 1) = 6355
NhËn xÐt thÊy 2m + 2n + 1 > 2m – 2n – 1 > 0 vµ chóng lµ nh÷ng sè lÎ, nªn ta cã thÓ
viÕt (2m + 2n + 1) (2m – 2n – 1) = 6355.1 = 1271.5 = 205.31 = 155.41
Suy ra n cã thÓ cã c¸c gi¸ trÞ sau : 1588 ; 316 ; 43 ; 28
Bµi t¬ng tù :
T×m a ®Ó c¸c sè sau lµ nh÷ng sè chÝnh ph¬ng
a) a2
+ a + 43
b) a2
+ 81
c) a2
+ 31a + 1984
KÕt qu¶: a) 2; 42; 13
b) 0; 12; 40
c) 12 ; 33 ; 48 ; 97 ; 176 ; 332 ; 565 ; 1728
Bµi 2 : T×m sè tù nhiªn n ≥ 1 sao cho tæng 1! + 2! + 3! + … + n! lµ mét sè chÝnh
ph¬ng.
Víi n = 1 th× 1! = 1 = 12
lµ sè chÝnh ph¬ng
Víi n = 2 th× 1! + 2! = 3 kh«ng lµ sè chÝnh ph¬ng
Víi n = 3 th× 1! + 2! + 3! = 1 + 1.2 + 1.2.3 = 9 = 33
lµ sè chÝnh ph¬ng
Víi n ≥ 4 ta cã 1! + 2! + 3! + 4! = 1 + 1.2 + 1.2.3 + 1.2.3.4 = 33 cßn 5!; 6!; …; n! ®Òu
tËn cïng bëi 0 do ®ã 1! + 2! + 3! + … n! cã tËn cïng bëi ch÷ sè 3 nªn nã kh«ng
ph¶i lµ sè chÝnh ph¬ng.
VËy cã 2 sè tù nhiªn n tho¶ m·n ®Ò bµi lµ n = 1; n = 3
Bµi 3: Cã hay kh«ng sè tù nhiªn n ®Ó 2010 + n2
lµ sè chÝnh ph¬ng.
Gi¶ sö 2010 + n2
lµ sè chÝnh ph¬ng th× 2010 + n2
= m2
(m N∈ )
Tõ ®ã suy ra m2
- n2
= 2010 ⇔ (m + n) (m – n) = 2010
Nh vËy trong 2 sè m vµ n ph¶i cã Ýt nhÊt 1 sè ch½n (1)
MÆt kh¸c m + n + m – n = 2m ⇒ 2 sè m + n vµ m – n cïng tÝnh ch½n lÎ (2)
Tõ (1) vµ (2) ⇒ m + n vµ m – n lµ 2 sè ch½n.
⇒ (m + n) (m – n) M 4 nhng 2006 kh«ng chia hÕt cho 4
⇒ §iÒu gi¶ sö sai.
VËy kh«ng tån t¹i sè tù nhiªn n ®Ó 2006 + n2
lµ sè chÝnh ph¬ng.
Bµi 4: BiÕt x N∈ vµ x > 2. T×m x sao cho )1()2()1(.)1( −−=−− xxxxxxxx
§¼ng thøc ®· cho ®îc viÕt l¹i nh sau: )1()2()1(
2
−−=− xxxxxx
15
Chuyªn ®Ò BDHSG To¸n THCS
Do vÕ tr¸i lµ mét sè chÝnh ph¬ng nªn vÕ ph¶i còng lµ mét sè chÝnh ph¬ng.
Mét sè chÝnh ph¬ng chØ cã thÓ tËn cïng bëi mét trong c¸c ch÷ sè 0; 1; 4; 5; 6; 9
nªn x chØ cã thÓ tËn cïng bëi mét trong c¸c ch÷ sè 1; 2; 5; 6; 7; 0 (1)
Do x lµ ch÷ sè nªn x ≤ 9, kÕt hîp víi ®iÒu kiÖn ®Ò bµi ta cã x N∈ vµ 2 < x ≤ 9
(2)
Tõ (1) vµ (2) ⇒ x chØ cã thÓ nhËn mét trong c¸c gi¸ trÞ 5; 6; 7
B»ng phÐp thö ta thÊy chØ cã x = 7 tho¶ m·n ®Ò bµi, khi ®ã 762
= 5776
Bµi 5: T×m sè tù nhiªn n cã 2 ch÷ sè biÕt r»ng 2n + 1 vµ 3n + 1 ®Òu lµ c¸c sè
chÝnh ph¬ng.
Ta cã 10 ≤ n ≤ 99 nªn 21 ≤ 2n + 1 ≤ 199. T×m sè chÝnh ph¬ng lÎ trong kho¶ng
trªn ta ®îc 2n + 1 b»ng 25; 49; 81; 121; 169 t¬ng øng víi sè n b»ng 12; 24; 40; 60; 84
Sè 3n + 1 b»ng 37; 73; 121; 181; 253. ChØ cã 121 lµ sè chÝnh ph¬ng.
VËy n = 40
Bµi 6: Chøng minh r»ng nÕu n lµ sè tù nhiªn sao cho n + 1 vµ 2n + 1 ®Òu lµ c¸c sè
chÝnh ph¬ng th× n lµ béi sè cña 24
V× n + 1 vµ 2n + 1 lµ c¸c sè chÝnh ph¬ng nªn ®Æt n + 1 = k2
, 2n + 1 = m2
(k, m N∈ )
Ta cã m lµ sè lÎ ⇒ m = 2a + 1 ⇒ m2
= 4a(a + 1) + 1
Mµ )1(2
2
)1(4
2
12
+=
+
=
−
= aa
aam
n
⇒ n ch½n ⇒ n + 1 lÎ ⇒ k lÎ ⇒ ®Æt k = 2b + 1 (víi b N∈ ) ⇒ k2
= 4b(b+1) + 1
⇒ n = 4b(b+1) ⇒ n M 8 (1)
Ta cã: k2
+ m2
= 3n + 2 ≡ 2 (mod3)
MÆt kh¸c k2
chia cho 3 d 0 hoÆc 1, m2
chia cho 3 d 0 hoÆc 1
Nªn ®Ó k2
+ m2
≡ 2 (mod3) th× k2
≡ 1 (mod3)
m2
≡ 1 (mod3)
⇒ m2
– k2
M 3 hay (2n + 1) – (n + 1) M 3 ⇒ n M 3 (2)
Mµ (8; 3) = 1 (3)
Tõ (1), (2), (3) ⇒ n M 24
Bµi 7: T×m tÊt c¶ c¸c sè tù nhiªn n sao cho sè 28
+ 211
+ 2n
lµ sè chÝnh ph¬ng
Gi¶ sö 28
+ 211
+ 2n
= a2
(a ∈ N) th×
2n
= a2
– 482
= (a + 48) (a – 48)
2p
. 2q
= (a + 48) (a – 48) víi p, q ∈ N ; p + q = n vµ p > q
⇒ a + 48 = 2p ⇒ 2p
2q
= 96 ⇔ 2q
(2p-q
– 1) = 25
.3
a – 48 = 2q
⇒ q = 5 vµ p – q = 2 ⇒ p = 7
⇒ n = 5 + 7 = 12
Thö l¹i ta cã: 28
+ 211
+ 2n
= 802
C.d¹ng 3 : T×m sè chÝnh ph¬ng
Bµi 1 : Cho A lµ sè chÝnh ph¬ng gåm 4 ch÷ sè. NÕu ta thªm vµo mçi ch÷ sè cña
A mét ®¬n vÞ th× ta ®îc sè chÝnh ph¬ng B. H·y t×m c¸c sè A vµ B.
Gäi A = 2
kabcd = . NÕu thªm vµo mçi ch÷ sè cña A mét ®¬n vÞ th× ta cã sè
16
Chuyªn ®Ò BDHSG To¸n THCS
B = 2
)1)(1)(1)(1( mdcba =++++ víi k, m ∈ N vµ 32 < k < m < 100
a, b, c, d = 9;1
⇒ Ta cã: A = 2
kabcd =
B = 2
1111 mabcd =+ . §óng khi céng kh«ng cã nhí
⇒ m2
– k2
= 1111 ⇔ (m - k)(m + k) = 1111 (*)
NhËn xÐt thÊy tÝch (m – k)(m + k) > 0 nªn m – k vµ m + k lµ 2 sè nguyªn d¬ng.
Vµ m – k < m + k < 200 nªn (*) cã thÓ viÕt (m – k) (m + k) = 11.101
Do ®ã: m – k = 11 ⇔ m = 56 ⇔ A = 2025
m + k = 101 n = 45 B = 3136
Bµi 2: T×m mét sè chÝnh ph¬ng gåm 4 ch÷ sè biÕt r»ng sè gåm 2 ch÷ sè ®Çu lín
h¬n sè gåm 2 ch÷ sè sau mét ®¬n vÞ.
§Æt 2
kabcd = ta cã 1=−cdab vµ k ∈ N, 32 ≤ k < 100
Suy ra : 101cd = k2
– 100 = (k – 10)(k + 10) ⇒ k + 10 M 101 hoÆc k – 10 M 101
Mµ (k – 10; 101) = 1 ⇒ k + 10 M 101
V× 32 ≤ k < 100 nªn 42 ≤ k + 10 < 110 ⇒ k + 10 = 101 ⇒ k = 91
⇒ abcd = 912
= 8281
Bµi 3: T×m sè chÝnh ph¬ng cã 4 ch÷ sè biÕt r»ng 2 ch÷ sè ®Çu gièng nhau, 2
ch÷ sè cuèi gièng nhau.
Gäi sè chÝnh ph¬ng ph¶i t×m lµ: aabb = n2
víi a, b ∈ N, 1 ≤ a ≤ 9; 0 ≤ b ≤ 9
Ta cã: n2
= aabb = 11. ba0 = 11.(100a + b) = 11.(99a + a + b) (1)
NhËn xÐt thÊy aabb M 11 ⇒ a + b M 11
Mµ 1 ≤ a ≤ 9; 0 ≤ b ≤ 9 nªn 1 ≤ a + b ≤ 18 ⇒ a + b = 11
Thay a + b = 11 vµo (1) ®îc n2
= 112
(9a + 1) do ®ã 9a + 1 lµ sè chÝnh ph¬ng
B»ng phÐp thö víi a = 1; 2;…; 9 ta thÊy chØ cã a = 7 tho¶ m·n ⇒ b = 4
Sè cÇn t×m lµ: 7744
Bµi 4: T×m mét sè cã 4 ch÷ sè võa lµ sè chÝnh ph¬ng võa lµ mét lËp ph¬ng.
Gäi sè chÝnh ph¬ng ®ã lµ abcd . V× abcd võa lµ sè chÝnh ph¬ng võa lµ mét lËp
ph¬ng nªn ®Æt abcd = x2
= y3
víi x, y ∈ N
V× y3
= x2
nªn y còng lµ mét sè chÝnh ph¬ng.
Ta cã : 1000 ≤ abcd ≤ 9999 ⇒ 10 ≤ y ≤ 21 vµ y chÝnh ph¬ng
⇒ y = 16 ⇒ abcd = 4096
Bµi 5 : T×m mét sè chÝnh ph¬ng gåm 4 ch÷ sè sao cho ch÷ sè cuèi lµ sè nguyªn
tè, c¨n bËc hai cña sè ®ã cã tæng c¸c ch÷ sè lµ mét sè chÝnh ph¬ng.
Gäi sè ph¶i t×m lµ abcd víi a, b, c, d nguyªn vµ 1 ≤ a ≤ 9; 0 ≤ b, c, d ≤ 9
abcd chÝnh ph¬ng ⇒ d { }9,6,5,4,1,0∈
d nguyªn tè ⇒ d = 5
§Æt abcd = k2
< 10000 ⇒ 32 ≤ k < 100
k lµ mét sè cã hai ch÷ sè mµ k2
cã tËn cïng b»ng 5 ⇒ k tËn cïng b»ng 5
Tæng c¸c ch÷ sè cña k lµ mét sè chÝnh ph¬ng ⇒ k = 45
⇒ abcd = 2025
17
Chuyªn ®Ò BDHSG To¸n THCS
VËy sè ph¶i t×m lµ: 2025
Bµi 6: T×m sè tù nhiªn cã hai ch÷ sè biÕt r»ng hiÖu c¸c b×nh ph¬ng cña sè ®ã vµ
viÕt sè bë hai ch÷ sè cña sè ®ã nhng theo thø tù ngîc l¹i lµ mét sè chÝnh ph¬ng
Gäi sè tù nhiªn cã hai ch÷ sèph¶i t×m lµ ab (a, b ∈ N, 1 ≤ a, b ≤ 9)
Sè viÕt theo thø tù ngîc l¹i ba
Ta cã ab
2
- ba
2
= (10a + b)2
– (10b + a)2
= 99 (a2
– b2
) M 11 ⇒ a2
– b2
M 11
Hay (a - b) (a + b) M 11
V× 0 < a – b ≤ 8, 2 ≤ a + b ≤ 18 nªn a + b M 11 ⇒ a + b = 11
Khi ®ã: ab
2
- ba
2
= 32
. 112
. (a – b)
§Ó ab
2
- ba
2
lµ sè chÝnh ph¬ng th× a – b ph¶i lµ sè chÝnh ph¬ng do ®ã a – b =
1 hoÆc a – b = 4
NÕu a – b = 1 kÕt hîp víi a + b = 11 ⇒ a = 6, b = 5 , ab = 65
Khi ®ã 652
– 562
= 1089 = 332
NÕu a – b = 4 kÕt hîp víi a + b = 11 ⇒ a = 7,5 lo¹i
VËy sè ph¶i t×m lµ 65
Bµi 7: Cho mét sè chÝnh ph¬ng cã 4 ch÷ sè. NÕu thªm 3 vµo mçi ch÷ sè ®ã ta
còng ®îc mét sè chÝnh ph¬ng. T×m sè chÝnh ph¬ng ban ®Çu.
(KÕt qu¶: 1156)
Bµi 8: T×m sè cã 2 ch÷ sè mµ b×nh ph¬ng cña sè Êy b»ng lËp ph¬ng cña tæng
c¸c ch÷ sè cña nã.
Gäi sè ph¶i t×m lµ ab víi a, b ∈ N, 1 ≤ a ≤ 9; 0 ≤ b ≤ 9
Theo gi¶ thiÕt ta cã: ab = (a + b)3
⇔ (10a +b)2
= (a + b)3
⇒ ab lµ mét lËp ph¬ng vµ a + b lµ mét sè chÝnh ph¬ng
§Æt ab = t3
(t ∈ N), a + b = 12
(1 ∈ N)
V× 10 ≤ ab ≤ 99 ⇒ ab = 27 hoÆc ab = 64
NÕu ab = 27 ⇒ a + b = 9 lµ sè chÝnh ph¬ng
NÕu ab = 64 ⇒ a + b = 10 kh«ng lµ sè chÝnh ph¬ng ⇒ lo¹i
VËy sè cÇn t×m lµ ab = 27
Bµi 9 : T×m 3 sè lÎ liªn tiÕp mµ tæng b×nh ph¬ng lµ mét sè cã 4 ch÷ sè gièng
nhau.
Gäi 3 sè lÎ liªn tiÕp ®ã lµ 2n - 1 ; 2n + 1 ; 2n + 3 (n ∈ N)
Ta cã : A = (2n – 1)2
+ (2n + 1)2
+ (2n +3)2
= 12n2
+ 12n + 11
Theo ®Ò bµi ta ®Æt 12n2
+ 12n + 11 = aaaa = 1111 . a víi a lÎ vµ 1 ≤ a ≤ 9
⇒ 12n(n + 1) = 11(101a – 1)
⇒ 101a – 1 M 3⇒ 2a – 1 M 3
V× 1 ≤ a ≤ 9 nªn 1 ≤ 2a – 1 ≤17 vµ 2a – 1 lÎ nªn 2a – 1 { }15;9;3∈
⇒ a { }8;5;2∈
V× a lÎ ⇒ a = 5 ⇒ n = 21
18
Chuyªn ®Ò BDHSG To¸n THCS
3 sè cÇn t×m lµ: 41; 43; 45
Bµi 10 : T×m sè cã 2 ch÷ sè sao cho tÝch cña sè ®ã víi tæng c¸c ch÷ sè cña nã
b»ng tæng lËp ph¬ng c¸c ch÷ sè cña sè ®ã.
ab (a + b) = a3
+ b3
⇔ 10a + b = a2
– ab + b2
= (a + b)2
– 3ab
⇔ 3a (3 + b) = (a + b) (a + b – 1)
a + b vµ a + b – 1 nguyªn tè cïng nhau do ®ã
a + b = 3a hoÆc a + b – 1 = 3a
a + b – 1 = 3 + b a + b = 3 + b
⇒ a = 4, b = 8 hoÆc a = 3, b = 7
VËy ab = 48 hoÆc ab = 37
Chuyªn ®Ò 2:
ph¬ng tr×nh nghiÖm nguyªn
1. T×m nghiÖm nguyªn cña Ph¬ng tr×nh vµ hÖ ph¬ng tr×nh bËc nhÊt hai Èn
Tuú tõng bµi cô thÓ mµ lµm c¸c c¸ch kh¸c nhau.
VD1: T×m nghiÖm nguyªn cña ph¬ng tr×nh: 2x + 3y = 11 (1)
C¸ch 1: Ph¬ng ph¸p tæng qu¸t:
Ta cã: 2x + 3y = 11
2
1
5
2
311 −
−−=
−
=⇔
y
y
y
x
§Ó ph¬ng tr×nh cã nghiÖm nguyªn 2
1−
⇔
y
nguyªn
§Æt Zt
y
∈=
−
2
1
⇒ y = 2t + 1
x = -3t + 4
C¸ch 2 : Dïng tÝnh chÊt chia hÕt
V× 11 lÎ ⇒ 2x + 3y lu«n lµ sè lÎ mµ 2x lu«n lµ sè ch½n ⇒ 3y lÎ ⇒ y lÎ
Do ®ã : y = 2t + 1 víi Zt ∈
x = -3t + 4
C¸ch 3 : Ta nh©n thÊy ph¬ng tr×nh cã mét cÆp nghiÖm nguyªn ®Æc biÖt lµ
x0 = 4 ; y0 = 1
ThËt vËy : 2 . 4 + 3.1 = 11 (2)
Trõ (1) cho (2) vÕ theo vÕ ta cã :
2(x - 4) + 3(y - 1) = 0
⇔ 2(x -4) = -3(y -1) (3)
Tõ (3) ⇒ 3(y - 1) M 2 mµ (2 ; 3) = 1 ⇒ y - 1 M 2
⇔ y = 2t + 1 víi Zt ∈
Thay y = 2t + 1 vµo (3) ta cã : x = -3t + 4
19
Chuyªn ®Ò BDHSG To¸n THCS
NhËn xÐt : Víi c¸ch gi¶i nµy ta ph¶i mß ra mét cÆp nghiÖm nguyªn (x0, y0) cña
ph¬ng tr×nh ax + by = c ; c¸ch nµy sÏ gÆp khã kh¨n nÕu hÖ sè a, b, c qu¸ lín.
C¸c bµi tËp t¬ng tù : T×m nghiÖm nguyªn cña ph¬ng tr×nh.
a) 3x + 5y = 10
b) 4x + 5y = 65
c) 5x + 7y = 112
VD2 : HÖ ph¬ng tr×nh.
T×m nghiÖm nguyªn d¬ng cña hÖ ph¬ng tr×nh sau :
3x + y + z = 14 (1)
5x + 3y + z = 28 (2)
Gi¶i : Tõ hÖ ®· cho ta cã : 2(x + y) = 14 vËy x = 7 - y (*)
Thay (*) vµo (1) ta ®îc z = 14 - y - 3x = 2y -7
V× x > 0 nªn 7 - y > 0 ⇒ y < 7 mµ z > 0 nªn 2y - 7 > 0 ⇒ y > 2
7
VËy 2
7
< y < 7 vµ Zy ∈ { }6;5;4∈⇒ y
Gi¶i tiÕp hÖ ®· cho cã 3 nghiÖm (3; 4; 1); (2; 5; 3); (1; 6; 5)
Bµi tËp t¬ng tù:
a) T×m nghiÖm nguyªn cña hÖ
2x -5y = 5
2y - 3z = 1
b) Tr¨m tr©u ¨n tr¨m bã cá – tr©u ®øng ¨n n¨m, tr©u n»m ¨n ba, tr©u giµ 3 con 1
bã. T×m sè tr©u mçi lo¹i.
c) T×m sè nguyªn d¬ng nhá nhÊt chia cho 1000 d 1 vµ chia cho 761 d 8.
2. T×m nghiÖm nguyªn cña ph¬ng tr×nh, hÖ ph¬ng tr×nh bËc cao.
Ph¬ng ph¸p 1 : Dïng dÊu hiÖu chia hÕt ®Ó gi¶i ph¬ng tr×nh.
VD1: a) T×m cÆp sè nguyªn (x ; y) tho¶ m·n ph¬ng tr×nh
6x2
+ 5y2
= 74 (1)
C¸ch 1 : Ta cã : 6 (x2
- 4) = 5 (10 - y2
) (2)
Tõ (2) ⇒ 6(x2
- 4) M 5 vµ (6 ; 5) = 1 ⇒ x2
- 4 M 5
⇒ x2
= 5t + 4 víi Nt ∈
Thay x2
- 4 = 5t vµo (2) ta cã : y2
= 10 – 6t
V× x2
> 0 vµ y2
> 0 ⇒ 5t + 4 > 0
10 - 6t > 0
⇒
3
5
5
4
<<− t víi Nt ∈
⇒ t = 0 hoÆc t = 1
Víi t = 0 ⇒ y2
= 10 (lo¹i)
Víi t = 1 ⇒ x2
= 9 ⇔ x = 3±
y2
= 4 y = 2±
VËy c¸c cÆp nghiÖm nguyªn lµ :........................
20
Chuyªn ®Ò BDHSG To¸n THCS
C¸ch 2 : Tõ (1) ta cã x2
+ 1 M 5
0 < x2
≤ 12 ⇒ x2
= 4 hoÆc x2
= 9
Víi x2
= 4 ⇒ y2
= 10 (lo¹i)
Víi x2
= 9 ⇒ y2
= 4 (tho¶ m·n)
VËy.....................
C¸ch 3 : Ta cã :
(1) ⇒ y2
ch½n
0 < y2
≤ 14 ⇒ y2
= 4 ⇒ x2
= 9
VËy...............
VD2 : Chøng minh r»ng ph¬ng tr×nh sau kh«ng cã nghiÖm nguyªn
a) x5
+ 29x = 10(3y + 1)
b) 7x
= 2y
- 3z
- 1
Gi¶i : x5
- x + 30x = 10(3y+1)
VP M 30 cßn VT M 30 ⇒ ph¬ng tr×nh v« nghiÖm
Ph¬ng ph¸p 2: Ph©n tÝch mét vÕ thµnh tÝch, mét vÕ thµnh h»ng sè nguyªn
VD1: T×m nghiÖm nguyªn cña ph¬ng tr×nh:
a) xy + 3x - 5y = -3
b) 2x2
- 2xy + x - y + 15 = 0
c) x2
+ x = y2
- 19
Gi¶i : a) C¸ch 1: x(y + 3) – 5(y + 3) = -18
⇔ (x – 5) (y + 3) = -18...
C¸ch 2 : 3
18
5
3
35
+
−=
+
−
=
yy
y
x
b) T¬ng tù.
c) 4x2
+ 4x = 4y2
- 76
⇔ (2x + 1)2
- (2y)2
= -75...
Ph¬ng ph¸p 3 : Sö dông tÝnh ch½n lÎ (®Æc biÖt cña chia hÕt)
VD2 : T×m nghiÖm nguyªn.
x3
- 2y3
- 4z3
= 0
Gi¶i : ⇔ x3
= 2(y3
+ 2z3
)
VP M 2 ⇒ x3
M 2 ⇒ x M 2 ®Æt x = 2k
8k3
= 2(y3
+ 2z3
) ⇔ 4k3
= y3
+ 2z3
⇒ y3
= 4k3
- 2z3
= 2(2k3
- z3
)
⇒ y ch½n. §Æt y = 2t ta cã :
8t3
= 2(2k3
- z3
) ⇒ 4t3
= 2k3
- z3
⇒ z3
= 2k3
- 4t3 ⇒ z ch½n ⇒ z = 2m
⇒ 8m3
= 2(k3
- 2t3
) ⇒ ......k ch½n.......
Ph¬ng ph¸p 4 : Ph¬ng ph¸p sö dông tÝnh chÊt cña sè chÝnh ph¬ng
VD1 : T×m nghiÖm nguyªn cña.
a) x2
- 4xy + 5y2
= 169
21
Chuyªn ®Ò BDHSG To¸n THCS
b) x2
- 6xy + 13y2
= 100
Gi¶i :
a) (x - 2y)2
+ y2
= 169 = 0 + 169 = 25 + 144...
b) (x – 3y)2
+ (2y)2
= 100 = 0 + 100 = 36 + 64 = ...
Ph¬ng ph¸p 5 : Ph¬ng ph¸p c«ng thøc nghiÖm ph¬ng tr×nh bËc 2
VD1 : T×m nghiÖm nguyªn cña ph¬ng tr×nh.
a) 2x2
-2xy + x + y + 15 = 0
b) 5(x2
+ xy + y2
) = 7(x+2y) (®Ò thi häc sinh giái tØnh 2009 – 2010)
c) x(x + 1) = y (y + 1) (y2
+ 2)
Ph¬ng ph¸p 6 : Ph¬ng ph¸p ®Æt Èn phô
VD: T×m nghiÖm nguyªn cña ph¬ng tr×nh:
6
7
32
22
22
12
2
2
2
2
=
++
++
+
++
++
xx
xx
xx
xx
(1)
§Æt y = x2
+ 2x + 2 (y ∈ Z)
(1) 6
7
1
1
=
+
+
−
⇔
y
y
y
y
⇔ 5y2
– 7y – 6 = 0
5
3
1 −=y (lo¹i) ; y2 = 2 (tho¶ m·n) ⇒ x1 = 0; x2 = -2
C¸c bµi tËp t¬ng tù:
a) x3
+ (x + 1)3
+ (x + 2)3
= (x + 3)3
b) 12
1
)1(
1
)2(
1
2
=
+
−
+ xxx
* Mét sè ph¬ng ph¸p kh¸c.
VD1 : T×m nghiÖm nguyªn cña ph¬ng tr×nh :
2x2
+ 4x = 19 -3y2
Gi¶i : ⇔ 4x2
+ 8x + 4 = 42 - 6y2
(2x + 2)2
= 6 (7 - y2
)
V× (2x + 2)2
≥ 0 ⇒ 7 - y2
≥ 0 ⇒ 72
≤y
Mµ y Z∈ ⇒ y = 0 ; 1± ; 2± Tõ ®©y ta t×m ®îc gi¸ trÞ t¬ng øng cña x
3. Mét sè bµi to¸n liªn quan tíi h×nh häc.
a) Cho tam gi¸c cã ®é dµi cña 3 ®êng cao lµ nh÷ng sè nguyªn d¬ng vµ ®êng trßn
néi tiÕp tam gi¸c ®ã cã b¸n kÝnh b»ng 1(®.v.®.d). Chøng minh tam gi¸c ®ã lµ tam
gi¸c ®Òu
Gi¶i: Gäi ®é dµi c¸c c¹nh vµ c¸c ®êng cao t¬ng øng theo thø tù lµ a; b; c vµ x; y; z. R
lµ b¸n kÝnh ®êng trßn néi tiÕp.
Ta cã R = 1⇒ x; y; z > 2 vµ gi¶ sö x ≥ y ≥ z > 2
Ta cã : ax = by = cz = (a + b+ c).1 (=2S)
Suy ra: a
cba
x
++
= ; c
cba
z
b
cba
y
++
=
++
= ;
cba
a
x ++
=⇒
1
; cba
b
y ++
=
1
; cba
c
z ++
=
1
⇒ 1
111
=++
zyx mµ x ≥ y ≥ z > 2
22
Chuyªn ®Ò BDHSG To¸n THCS
⇒
xz
11
≥ vµ yz
11
≥ nªn zzyx
3111
≤++
⇒
z
3
1 ≤ ⇒ 3≤z ⇒ z = 3
T¬ng tù ta cã: x = 3; y = 3 ⇒ tam gi¸c ®ã lµ tam gi¸c ®Òu
b) T×m tÊt c¶ c¸c h×nh ch÷ nhËt víi ®é dµi c¸c c¹nh lµ c¸c sè nguyªn d¬ng cã thÓ
c¾t thµnh 13 h×nh vu«ng b»ng nhau sao cho mçi c¹nh cña h×nh vu«ng lµ sè
nguyªn d¬ng kh«ng lín h¬n 4 (®.v.®.d)
Gi¶i : Gäi c¸c c¹nh h×nh ch÷ nhËt cÇn t×m lµ a vµ b, c¹nh h×nh vu«ng lµ c. Tõ gi¶
thiÕt h×nh ch÷ nhËt c¾t thµnh 13 h×nh vu«ng nªn ph¶i cã:
ab = 13c2
(1) víi 0 < c ≤ 4 (2)
Tõ (1) suy ra a hoÆc b chia hÕt cho 13. V× vai trß a, b nh nhau ta cã thÓ gi¶ gi¶
sö a chia hÕt cho 13, tøc lµ a = 13d
Thay vµo (1) ta ®îc : 13db = 13c2
Hay db = c2
Ta h·y xÐt c¸c trêng hîp cã thÓ cã cña c.
Víi c = 1, chØ cã thÓ: d = 1, b = 1, suy ra a = 13
Víi c = 2, chØ cã thÓ: d = 1, b = 4, suy ra a = 13
d = 2, b = 2, suy ra a = 26
d = 4, b = 1, suy ra a = 52
Víi c = 3, chØ cã thÓ: d = 1, b = 9, suy ra a = 13
d = 3, b = 3, suy ra a = 39
d = 9, b = 1, suy ra a = 117
Víi c = 4, chØ cã thÓ: d = 1, b = 16, suy ra a = 13
d = 2, b = 8, suy ra a = 26
d = 4, b = 4, suy ra a = 52
d = 8, b = 2, suy ra a = 104
d = 16, b = 1, suy ra a = 208
Víi 12 nghiÖm cña ph¬ng tr×nh (1) chØ cã 4 trêng hîp tho¶ m·n bµi to¸n. Bµi to¸n
cã 4 nghiÖm. Ta t×m ®îc 4 h×nh ch÷ nhËt tho¶ m·n ®Ò bµi:
(a = 13, b = 1); (a = 26, b = 2); (a = 39, b = 3); (a = 52, b = 4)
Chuyªn ®Ò 3:
Gi¶i ph¬ng tr×nh v« tû vµ hÖ ph¬ng tr×nh
(Dµnh cho båi dìng häc sinh giái tØnh)
I. Gi¶i ph¬ng tr×nh v« tû
* C¸c ph¬ng ph¸p
1. Luü thõa khö c¨n
23
Chuyªn ®Ò BDHSG To¸n THCS
2. §Æt Èn phô
3. Dïng bÊt ®¼ng thøc
4. XÐt kho¶ng
II. ¸p dông c¸c ph¬ng ph¸p
A. Phương pháp luỹ thừa khử căn
1. Gi¶i c¸c ph¬ng tr×nh
a) )1(2321 =−+− xx
§iÒu kiÖn: x≤
2
3
Víi 2
3
≥x PT (1) 43522321 2
=+−+−+−⇔ xxxx
xxx 383522 2
−=+−⇔





≤
−+=+−
⇔
3
8
)2(48964)352(4 22
x
xxxx
PT (2) 052282
=+−⇔ xx



=
=
⇔
)(26
)(2
Kotmx
tmx
VËy PT ®· cho cã nghiÖm x=2
b) )1()1()1(3 22
−+=+− xxxx
§K: 1≥x
Víi 1≥x PT (1) 112)1(3 22
−+−+=+−⇔ xxxxxx
12442 2
−=+−⇔ xxxx 1222
−=+−⇔ xxxx
Do 1≥x nªn 2 vÕ cña PT nµy kh«ng ©m v× vËy PT nµy
232324
48444 xxxxxxx −=+−−+−⇔
04895 234
=+−+−⇔ xxxx
0)1()2( 22
=+−−⇔ xxx



=+−
=−
⇔
01
02
2
xx
x
2=⇔ x ™
c) 1222 33
−=−−− xx (1)
Gi¶i:
Pt (1) ⇔ ( ) 1222
3
33
−=−−− xx
24
Chuyªn ®Ò BDHSG To¸n THCS
1)22()2(.()22)(2(3222 333 −=−−−−−−+−−⇔ xxxxxx
3 2
46231 +−=−⇒ xxx
)462(27331 232
+−=−+−⇔ xxxxx
010715951 23
=+−+⇔ xxx
0)10752)(1( 2
=−+−⇔ xxx



=+−
=
⇔
010752
1
2
xx
x





−−=
+−=
=
⇔
78326
78326
1
x
x
x
B. Ph¬ng ph¸p ®Æt Èn phô
(2) Gi¶i c¸c ph¬ng tr×nh:
a) 3123
=++− xx
Gi¶i:
§K: 1−≥x
§Æt ax =−3
2 ; bx =+1 ( 0≥b )
Ta cã hÖ PT



=+
−=−
3
323
ba
ba
Suy ra 06623
=−+− aaa 0)6)(1( 2
=+−⇔ aa
)/(31 mTxa =⇒=⇔
VËy ph¬ng tr×nh nghiÖm 3=x
b. )1(552
=+− xx
§K: 5−≥x
§Æt : yx =+5 ( )0≥y ta cã hÖ ph¬ng tr×nh




=−
=−
5
5
2
2
xy
yx
0)()( 22
=−+−⇒ yxyx



=++
=
⇔
01yx
yx
+)



=−−
≥
⇔=+⇒=
05
0
5 2
xx
x
xxyx
25
Chuyªn ®Ò BDHSG To¸n THCS





±
=
≥
⇔
2
211
0
x
x
2
211+
=⇔ x (Ko T/m)
+) 01 =++ yx 015 =+++⇒ xx
51 +−=+⇔ xx )1(5 +−=+⇔ xx
⇔



+=++
≤+
(*)512
01
2
xxx
x
PT (*) 042
=−+ xx






−−
=
+−
=
⇔
2
171
2
171
x
x
(ko t/m)
VËy PT v« nghiÖm
c) 6
2
4
).2(5)4)(2( =
+
+
++++
x
x
xxx
§K: 0
2
4
≥
+
+
x
x
§Æt )2)(4()2.(
2
4 2
++=⇒=+
+
+
xxaax
x
x
Ta cã PT: 0652
=−+ aa



−=
=
6
1
a
a
+) 01861 2
=−++⇒= xxa
0762
=++⇒ xx





−−=
+−
=
23
)(
1
23
x
tmx
+) 036866 2
=−++⇒−= xxa
02862
=−+⇒ xx




−−=
+−=
)(373
373
tmx
x
VËy pt cã 2 nghiÖm
373;23 −−+−=x
C. ¸p dông bÊt ®¼ng thøc
26
Chuyªn ®Ò BDHSG To¸n THCS
(3) Gi¶i c¸c ph¬ng tr×nh
a) 45224252642 =−−−+−++ xxxx (1)
§K: 2
5
≥x
Víi §k: 2
5
≥x PT (1)
⇔ 4152352 =−−++− xx
Ta cã:
4152352 ≥−−++− xx
§¼ng thøc xÈy ra





≥
≤−−+−
⇔
2
5
0)152)(352(
x
xx
⇔ 3
2
5
≤≤ x
VËy nghiÖm cña PT ®· cho lµ 3
2
5
≤≤ x
b) )1(271064 2
+−=−+− xxxx
Gi¶i
§K 64 ≤≤ x
Trªn TX§ )64)(11(64 22
xxxx −+−+≤−+−
⇔ 264 ≤−+− xx
L¹i cã
22)5(2710 22
≥+−=+− xxx
xxxx −+−≥+−⇒ 6427102
§¼ng thøc xÈy ra
5
64
5
64
=⇔





≤≤
=
−=−
⇔ x
x
x
xx
VËy PT (1) cã nghiÖm lµ x=5
c) Gi¶i ph¬ng tr×nh
211 22
+−=+−+−+ xxxxxx
Gi¶i
27
Chuyªn ®Ò BDHSG To¸n THCS
§K:




≥++−
≥−+
01
01
2
2
xx
xx
¸p dông B§T c« si cho c¸c sè kh«ng ©m ta cã
⇒






+++−
≤++−
+−+
≤−+
2
11
1).1(
2
11
1).1(
2
2
2
2
xx
xx
xx
xx
111 22
+≤+−+−+ xxxxx
Ta cã
122
+≥+− xxx (V× 0)1( 2
≥−x )
211 222
+−≤+−+−+⇒ xxxxxx
§¼ng thøc xÈy ra 1=⇔ x
VËy pt cã nghiÖm lµ x=1
D. XÐt kho¶ng
(4) Gi¶i c¸c PT
a) )1(353448 22
++−=+ xxx
Gi¶i
TX§: x∀
PT(1) 343548 22
−=+−+⇔ xxx
34
3548
13
22
−=
+++
⇔ x
xx
ThÊy 1=x lµ nghiÖm cña PT (1)
+) 1335481 22
>+++⇒> xxx
⇒





>−
<
+++
⇒
134
1
3548
13
22
x
xx PT v« nghiÖm
+) 1
4
3
<≤ x
133548 22
<+++⇒ xx
28
Chuyªn ®Ò BDHSG To¸n THCS
⇒





<−
>
+++⇒
134
1
3548
13
22
x
xx PT v« nghiÖm
VËy ph¬ng tr×nh cã nghiÖm duy nhÊt lµ x=1
b) 1235 3 46
=−−− xx (1)
Gi¶i
Ta cã:
1>x th× 1; 64
>xx
1<x th× 1; 64
<xx
+) XÐt 1>x 123;45 46
>−<−⇒ xx
3 46
235 −−−⇒ xx
 PT (1) v« nghiÖm
Xet 1<x tương tự ta suy ra phương trình vô nghiệm
ThÊy x= 1 hoÆc x= -1 lµ nghiÖm cña PT (1)
Bµi tËp:
Gi¶i c¸c PT
(1) a) )(15)2(2 32
Bxx +=+
(b) )(917.17 22
Bxxxx =−+−+
(2) xxx +=− 3.3 (A)
(3) 83124 22
++=++ xxx (D)
(4) )(13626 2
Cxxxx +−=++−
(5) )(231034 Axx −=−−
(6) 08645.27 56105
=+− xx (C)
III. Gi¶i hÖ ph¬ng tr×nh
* C¸c ph¬ng ph¸p:
1. Ph¬ng ph¸p thÕ
2. C«ng thøc trõ, nh©n, chia c¸c vÕ
3. §Æt Èn phô
4. Dïng bÊt ®¼ng thøc.
IV. ¸p dông c¸c ph¬ng ph¸p.
A. Ph¬ng ph¸p thÕ.
1. Gi¶i c¸c hÖ pg¬ng tr×nh
29
Chuyªn ®Ò BDHSG To¸n THCS
a)



=+
=+
2947
113
yx
yx
Gi¶i
HÖ ®· cho t¬ng ®¬ng víi



=−+
−=
29)311(47
311
xx
xy



=
−=
⇔
155
311
x
xy



=
=
2
3
y
x
VËy hÖ ®· cho cã nghiÖm lµ: (x;y) = (3;2)
b)




=−++
=+−
027624
065
2
22
yxyx
xyx
Gi¶i
HÖ ®· cho t¬ng ®¬ng víi



=−++
=−−
027624
0)3)(2(
2
yxyx
yxyx










=−+
=



=−+
=
⇔
027642
3
027620
2
2
2
yy
yx
yy
yx
30
Chuyªn ®Ò BDHSG To¸n THCS






























−−
=
+
=
=














−−
=
−
=
=
⇔
14
1271
14
1271
3
20
5493
20
3549
2
y
y
yx
y
y
yx







+−
=
+−
=
20
5493
10
5493
y
x
HoÆc







−−
=
−−
=
20
5493
10
5493
y
x







+−
=
+−
=
14
1271
14
12733
y
x
HoÆc







−−
=
−−
=
14
1271
14
12733
y
x
c)




=++
++=++
2004200320032003
222
3zyx
zxyzxyzyx
Gi¶i:




=++
++=++
)2(3
)1(
2004200320032003
222
zyx
zxyzxyzyx
Ta cã:
PT (1) 0222222 222
=−−−++⇔ zxyzxyzyx
0)()()( 222
=−+−+−⇔ xzzyyx
zyx ==⇔
ThÕ vµo (2) ta cã: 20042003
33 =x
20032003
3=x
31
Chuyªn ®Ò BDHSG To¸n THCS
3=x
Do ®ã x= y=z = 3
VËy nghiÖm cña hÖ ®· cho lµ:
(x;y;z) = (3;3;3)
B. Ph¬ng ph¸p céng, trõ, nh©n, chia c¸c vÕ
(2) Gi¶i c¸c hÖ ph¬ng tr×nh
a)




=−
=+
226
2235
yx
yx
Gi¶i:
HÖ ®· cho t¬ng ®¬ng víi:




=−
=+
226
4265
yx
yx




=+
=
⇔
2235
666
yx
x






−
=
=
2
1
6
1
y
x
b)




+=
+=
12
12
3
3
xy
yx
Gi¶i:
HÖ ®· cho t¬ng ®¬ng víi




=−+−
+=
0)(2
12
33
3
yxyx
yx




=+++−
+=
⇔
0)2)((
12
22
3
yxyxyx
yx



=
+=
⇔
yx
yx 123
(do 0222
>+++ yxyx )
32
Chuyªn ®Ò BDHSG To¸n THCS



=
=−−
⇔
yx
xx 0123



=
=−−+
⇔
yx
xxx 0)1)(1( 2











=









+
=
−
=
−=
⇔
yx
x
x
x
2
51
2
51
1



−=
−=
⇔
1
1
y
x
hoÆc







−
=
−
=
2
51
2
51
y
x
hoÆc







+
=
+
=
2
51
2
51
y
x
c)





=++
=++
=++
9
4
1(
xzxz
zyzy
yxyx
trong ®ã 0,, zyx
Gi¶i
HÖ ®¸ cho t¬ng ®¬ng víi





=++
=++
=++
10)1)(1(
5)1)(1(
2)1)(1(
xz
zy
yx
[ ]







=++
=++
=++
=+++
⇔
10)1)(1(
5)1)(1(
2)1)(1(
100)1)(1)(1(
2
xz
zy
yx
zyx







=++
=++
=++
=+++
⇔
10)1)(1(
5)1)(1(
2)1)(1(
10)1)(1)(1(
xz
zy
yx
zyx
33
(Do x,y,z>0)
Chuyªn ®Ò BDHSG To¸n THCS





=+
=+
=+
⇔
11
21
51
y
x
z





=
=
=
⇔
4
0
1
z
y
x
VËy hÖ ®· cho cã nghiÖm lµ
(x;y;z)=(1;0;4)
C. Ph¬ng ph¸p ®Æt Èn phô
(3). Gi¶i c¸c hÖ ph¬ng tr×nh
a)




++=+
++=+
6
5
2233
22
xyyxyx
yyxx
§Æt:
x-y=a; x+y =b
HÖ ®· cho trë thµnh



=
=+
)2(6
)1(5
2
ba
aab
Tõ PT (2) ta suy ra 0≠a
Do ®ã: 2
6
a
b =
ThÕ vµo (1) ta ®îc:
5
6
=+ a
a
0652
=+−⇔ aa (V× 0≠a )
0)3)(2( =−−⇔ aa



=
=
⇔
3
2
a
a
+) 2
3
2 =⇒= ba
34
Chuyªn ®Ò BDHSG To¸n THCS
Hay






−
=
=
⇔




=−
=+
4
1
4
7
2
2
3
y
x
yx
yx
+) 3
2
3 =⇒= ba
Hay






−
=
=
⇔




=−
=+
6
7
6
11
3
3
2
y
x
yx
yx
Tãm l¹i hÖ ph¬ng tr×nh ®· cho cã nghiÖm lµ:
(x;y) = 




 −





 −
6
7
;
6
11
;
4
1
;
4
7
b)



=++
=++
5
173333
yxyx
yyxx
Gi¶i:
§Æt x+y = a; xy=b
HÖ ®· cho trë thµnh



=+
=−+
5
17333
ba
abba



=+−
−=
⇔
065
5
2
bb
ba



=−−
−=
⇔
0)3)(2(
5
bb
ba



=
=
⇔
2
3
b
a
HoÆc



=
=
3
2
b
a
35
Chuyªn ®Ò BDHSG To¸n THCS
+)



=
=
2
3
b
a
Ta cã hÖ ph¬ng tr×nh



=
=+
2
3
xy
yx



=+−
−=
⇔
023
3
2
yy
yx



=−−
−=
⇔
0)2)(1(
3
yy
yx
⇔



=
=
1
2
y
x
HoÆc



=
=
2
1
y
x
+)



=
=
3
2
b
a
Ta cã hÖ ph¬ng tr×nh



=
=+
3
2
xy
yx



=+−
−=
⇔
032
2
2
yy
yx
(V« nghiÖm)
HÖ nµy v« nghiÖm
VËy nghiÖm cña hÖ ®· cho lµ:
(x;y) = (1;2); (2;1)
c)
( )




−=+
−=+
78)(
2156
22
224
yxxy
yxyx
Gi¶i
HÖ ®· cho t¬ng ®¬ng víi




−=+
−=+++
78
21544
33
4334
xyyx
yxyyxx
⇔




−=+
−=+++
16770215215
167707831231278
33
4334
xyyx
yxyyxx
⇔




−=+
=+++
78
)1(078979778
33
4334
xyyx
yxyyxx
36
Chuyªn ®Ò BDHSG To¸n THCS
§Æt y
x
t = PT (1) trë thµnh
078979778 34
=+++ ttt
0)131213)(32)(23( 2
=+−++⇔ tttt






−
=
−
=
⇔
2
3
3
2
t
t
+) yxt
3
2
3
2 −
=⇒
−
=
ThÕ vµo (2) ta ®îc 78
27
26 4
=y
814
=⇔ y
3=⇔ y HoÆc 3−=y
Suy ra:



=
−=
3
2
y
x
HoÆc



−=
=
3
2
y
x
+) yxt
2
3
2
3 −
=⇒
−
=
ThÕ vµo (2) ta ®îc 78
8
39 4
=y
164
=⇔ y
2=⇔ y HoÆc 2−=y
Suy ra:



=
−=
2
3
y
x
HoÆc



−=
=
2
3
y
x
Tãm l¹i hÖ ®· cho cã nghiÖm lµ:
(x;y) = (-2;3); (2;-3); (-3;2) ; (3;-2)
D. ¸p dông bÊt ®¼ng thøc
(4) Gi¶i c¸c hÖ ph¬ng tr×nh
a)



=++
=++
xyzzyx
zyx
444
1
Gi¶i:
NhËn xÐt: Tõ B§T 0)()()( 222
≥−+−+− accbba
37
Chuyªn ®Ò BDHSG To¸n THCS
Ta suy ra: (*)222
cabcabcba ++≥++
¸p dông liªn tiÕp B§T (*) ta ®îc
222222444
xzzyyxzyx ++≥++ )( zyxxyz ++≥
⇔ xyzzyx ≥++ 444
§¼ng thøc xÈy ra khi: 3
1
=== zyx
VËy hÖ ®· cho cã nghiÖm lµ: 





=
3
1
;
3
1
;
3
1
);;( zyx
b)




−=−+
−=−+
yxx
yxx
62432
332
4
24
Gi¶i:
§K: 320 ≤≤ x
HÖ ®· cho t¬ng ®¬ng víi




−=−+
+−=−++−+
332
216)32()32(
24
244
yxx
yyxxxx
Theo bÊt ®¼ng thøc BunhiaCèp xki ta cã
64)32)(11()32( 222
=−++≤−+ xxxx
832 ≤−+⇒ xx
( ) [ ] 256)32(232
24
44
≤−+≤−+ xxxx
⇒ 43244
≤−+ xx
Suy ra 12)32()32( 44
≤−++−+ xxxx
MÆt kh¸c ( ) 12123216
22
≥+−=+− yyy
§¼ng thøc xÈy ra khi x= 16 vµ y=3 (t/m)
VËy hÖ ®· cã nghiÖm lµ (x;y) = (16;3)
Chuyªn ®Ò 4:
BẤT ĐẲNG THỨC VÀ GIÁ TRỊ LỚN NHẤT, GIÁ TRỊ NHỎ NHẤT
A - CÁC PHƯƠNG PHÁP CHỨNG MINH BẤT ĐẲNG THỨC
1) Phương pháp đổi tương đương
• Để chứng minh: A B≥
Ta biến đổi 1 1 n nA B A B A B≥ ⇔ ≥ ≥ ≥K (đây là bất đẳng thức đúng)
38
Chuyªn ®Ò BDHSG To¸n THCS
Hoặc từ bất đẳng thức đứng n nA B≥ , ta biến đổi
1 1 1 1n n n nA B A B A B A B− −≥ ⇔ ≥ ≥ ≥ ⇔ ≥K
Ví dụ 1.1
( ) ( )
22 2
2 2 2
: ) 2 (1)
b) a (1)
CMR a a b a b
b c ab bc ca
+ ≥ +
+ + ≥ + +
Giải
( ) ( ) ( )
( )
22 2
2 2
2
) 1 2 0
2 0
0 (2)
a a b a b
a b ab
a b
⇔ + − + ≥
⇔ + − ≥
⇔ − ≥
Do bất đẳng thức (2) đúng nên bất đẳng thức (1) được chứng minh.
b)
( ) ( ) ( )
( ) ( ) ( )
( ) ( ) ( )
2 2 2
2 2 2 2 2 2
2 2 2
) 1 2 2 0
2 2 2 0
0 (2)
b a b c ab bc ca
a ab b b bc c c ca a
a b b c c a
⇔ + + − + + ≥
⇔ − + + − + + − + ≥
⇔ − + − + − ≥
Bất đẳng thức (2) đúng suy ra điều phải chứng minh.
Ví dụ 1.2
CMR
( ) ( )( )
( ) ( ) ( )
4 4 3 3
4 4 4 3 3 3
) 2 (1)
) 3 (1)
a a b a b a b
b a b c a b c a b c
+ ≥ + +
+ + ≥ + + + +
Giải
( ) ( )
( ) ( )
( ) ( )
( )( ) ( ) ( ) ( )
4 4 4 3 3 4
4 3 3 3
3 3
23 3 2 2
) 1 2 2 0
0
0
0 0 2
a a b a a b ab b
a a b ab b
a a b b a b
a b a b a b a ab b
⇔ + − + + + ≥
⇔ − − − ≥
⇔ − − − ≥
⇔ − − ≥ ⇔ − + + ≥
Do bất đẳng thức (2) đúng suy ra điều phải chứng minh.
( ) ( )
( ) ( ) ( )
( ) ( ) ( ) ( ) ( ) ( )
4 4 4 4 3 3 4 3 3 3 3 4
4 4 3 3 4 4 3 3 4 4 3 3
2 2 22 2 2 2 2 2
) 1 3 3 3 0
0
0
b a b c a a b a c b ab b c ac bc c
a b a b ab b c b c bc a c a c ac
a b a ab b b c b bc c a c a ac c
⇔ + + − + + + + + + + + ≥
⇔ + − − + + − − + + − − ≥
⇔ − + + + − + + + − + + ≥
Ví dụ 1.3
39
Chuyªn ®Ò BDHSG To¸n THCS
( )( ) ( ) ( )
( ) ( ) ( )
22 2 2 2
2 22 2 2 2
: a) ax 1
b) 1
CMR a b x b by
a b c d a c b d
+ + ≥ +
+ + + ≥ + + +
Giải
( )
( )
2 2 2 2 2 2 2 2 2 2 2 2
2 2 2 2
2
) 1 2
2 0
0
a a x a y b x b y a x abxy b y
a y abxy b y
ay bx
⇔ + + + ≥ + +
⇔ − + ≥
⇔ − ≥
( ) ( )( ) ( ) ( )
( )( )
2 22 2 2 2 2 2 2 2
2 2 2 2
) 1 2
(2)
b a b c d a b c d a c b d
a b c d ac bd
⇔ + + + + + + ≥ + + +
⇔ + + ≥ +
Nếu ac + bd < 0 thì (2) đúng
Nếu 0ac bd+ ≥ thì
( ) ( )( ) ( )
( )
22 2 2 2
2 2 2 2 2 2 2 2 2 2 2 2
2
2
2
0
a b c d ac bd
a c a d b c b d a c abcd b d
ad bc
⇔ + + ≥ +
⇔ + + + ≥ − +
⇔ − ≥
Suy ra đpcm.
Ví dụ 1.4
Cho a, b, c > 0, chứng minh rằng: ( )
2 2 2
1
a b c
a b c
b c a
+ ≥ + +
Giải
( )
( ) ( ) ( )
( ) ( ) ( )
3 3 3 2 2 2
2 2 2 2 2 2
2 2 2
1 0
2 2 2 0
0
a c b a c b a bc b ac c ab
ac a ab b ab b bc c bc c ca a
ac a b ab b c bc c a
⇔ + + − − − ≥
⇔ − + + − + + − + ≥
⇔ − + − + − ≥
⇒ đpcm.
Ví dụ 1.5
Cho a, b, c > 0. CMR: ( ) ( ) ( )2 2 2
3a b c a b a c b c b a c abc+ − + + − + + − ≤ (1)
Giải
( ) ( ) ( ) ( )
( ) ( ) ( )
( )( ) ( ) ( ) ( ) ( )
( )( ) ( )( )
( ) ( ) ( )( )
2 2 2
2 2 2
2 2
2
/ , 0
1 3 0
0
0
0
0
G s a b c
abc a b c a b a c b c b a c
a a ab ac bc b b bc ba ac c c ac bc ab
a a b a c b b c b a c c a c b
a b a ac b bc c a c b c
a b a b c c a c b c
≥ >
⇔ − + − + + − + + − ≥
⇔ − − + + − − + + − − + ≥
⇔ − − + − − + − − ≥
⇔ − − − + + − − ≥
⇔ − + − + − − ≥
Suy ra ĐPCM.
40
Chuyªn ®Ò BDHSG To¸n THCS
2) Phương pháp biến đổi đồng nhất
Để chứng minh BĐT: A ≥ B. Ta biến đổi biểu thức A – B thành tổng các biểu thức có giá
trị không âm.
Ví dụ 2.1
Chứng minh rằng:
( )
( )
2 2 2 2
2 2 2
) 1
) 4 4 4 4 8 1
a a b c d ab ac ad
b a b c ab ac bc
+ + + ≥ + +
+ + ≥ − +
Giải
2 2 2 2
2 2 2 2
2 2 2
2 2 2 2
) Ta có
=
4 4 4 4
0
2 2 2 4
a a b c d ab ac ad
a a a a
ab b ac c ad d
a a a a
b c d
+ + + − − −
     
− + + − + + − + + ÷  ÷  ÷
     
     
= − + − + − + ≥ ÷  ÷  ÷
     
( ) ( )
( ) ( )
( )
2 2 2
2 2 2
2 2
2
) Ta có: 4 4 4 4 8
4 4 4 4 8
2 4 4 2
2 2 0
b a b c ab ac bc
a ab b c ac bc
a b c c a b
a b c
+ + − + −
= − + + + −
= − + + −
= − + ≥
Ví dụ 2.2
Chứng minh rằng:
a) ( ) ( )
33 3
4 a b a b+ ≥ + với a, b > 0
b) ( ) ( ) ( ) ( )
3 3 33 3 3
8 a b c a b b c c a+ + ≥ + + + + + với a, b, c > 0
c) ( )
3 3 3 3
24a b c a b c abc+ + ≥ + + + với a, b, c ≥0
Giải
( ) ( ) ( ) ( ) ( )
( ) ( )
3 23 3 2 2
2
) Ta có:4 4
3 0
a a b a b a b a ab b a b
a b a b
 + − + = + − + − +
 
= + − ≥
( ) ( ) ( ) ( )
( ) ( ) ( ) ( ) ( ) ( )
( ) ( ) ( ) ( ) ( ) ( )
3 3 33 3 3
3 3 33 3 3 3 3 3
2 2 2
) Ta có:8
4 4 4
3 4 3 0
b a b c a b b c c a
a b a b a c c a b c b c
a b a b a c a c b c b c
+ + − + − + − +
     = + − + + + − + + + − +
     
= + − + + − + + − ≥
( )
( ) ( ) ( )
( ) ( ) ( )
( ) ( ) ( )
3 3 3 3
3 2 2 3 3 3 3
3 2 2 3 2 2 2 3 3
2 2 2 2 2 2
2 2 2
) Ta có: 24
3 3 24
3 3 3 6 3 3 24
3 2 3 2 3 2
3 3 3 0
c a b c a b c abc
a b a b c a b c c a b c abc
a a b ab b a c abc ac bc a b abc
a b c b abc a c b c abc b a c a abc
b a c c a b a b c
+ + − − − −
= + + + + + + − − − −
= + + + + + + + − − −
= + − + + − + + −
= − + − + − ≥
41
Chuyªn ®Ò BDHSG To¸n THCS
Ví dụ 2.3
Với a, b, c > 0. Chứng minh rằng:
1 1 4
)
1 1 1 9
)
3
)
2
a
a b a b
b
a b c a b c
a b c
c
b c c a a b
+ ≥
+
+ + ≥
+ +
+ + ≥
+ + +
Giải
( ) ( )
( )
( ) ( ) ( )
2 2
2 2 2
41 1 4
) Ta có: 0
1 1 1
)Ta có: 9 2 2 2
0
a b ab a b
a
a b a b a b a b
a b b c a c
b a b c
a b c b a c b c a
a b b c a c
ab bc ac
+ − −
+ − = = ≥
+ + +
       
+ + + + − = + − + + − + + − ÷  ÷  ÷  ÷
       
− − −
= + + ≥
( ) ( )
( )
( ) ( )
( )
( ) ( )
( )
( ) ( ) ( )
( )
( ) ( )
( )
( ) ( )
( )
( )
2 2 2
3 1 1 1
)Ta có:
2 2 2 2
2 2 2
1 1 1 1 1 1 1 1 1
2 2 2
1
2
a b c a b c
c
b c c a a b b c c a a b
a b a c b a b c c a c b
b c c a a b
a b b c c a
b c c a a c a b a b b c
a b b c a c
a c b c a c a b a b b
     
+ + − = − + − + − ÷  ÷  ÷
+ + + + + +     
− + − − + − − + −
= + +
+ + +
     
= − − + − − + − − ÷  ÷  ÷
+ + + + + +     
− − −
= + +
+ + + + + +( )
( ) ( ) ( ) ( ) ( ) ( )
0
Cách 2
3 1 1
Ta có: 1 3
2 2 2
1
6
2
1
2 2 2
2
c
a b c a b c
b c c a a b b c c a a b
a b a c a b b c a c b c
b c a c a b
a b b c b c a c a c a b
b c a b a c b c a b a c
 
≥ 
  
     
+ + − = + + + + + − ÷  ÷  ÷
+ + + + + +     
+ + + + + + + + + 
= + + − 
+ + + 
 + + + + + +      
= + − + + − + + − ÷  ÷  ÷ + + + + + +      
0≥
42
Chuyªn ®Ò BDHSG To¸n THCS
Ví dụ 2.4
) Cho a, b 0. CMR: a + b 2a ab≥ ≥ (Bất đẳng thức Cô – si)
3
) Cho a, b, c 0. CMR: a + b + c 3b abc≥ ≥ (Bất đẳng thức Cô – si)
) Cho a b c và .c x y z CMR≥ ≥ ≤ ≤
( ) ( ) ( )3 axa b c x y z by cz+ + + + ≥ + + (Bất đẳng thức Trê bư sếp)
Giải
a) Ta có: ( )
2
2 0a b ab a b+ − = − ≥
b) ( ) ( ) ( ) ( )
2 2 2
3 3 3 3 3 3 3 3 3 31
3 0
2
a b c abc a b c a b c b a c + + − = + + − + − + − ≥
  
c)
( ) ( ) ( )
( )( ) ( ) ( ) ( ) ( )
3 ax
0
a b c x y z by cz
y x a b z x b c x z c a
+ + + + − + +
= − − + − − + − − ≥
Ví dụ 2.5
Cho a, b, c > 0. Chứng minh:
( )2 2 2
)
) 3
bc ac ab
a a b c
a b c
bc ac ab
b a b c
a b c
+ + ≥ + +
+ + ≥ + +
Giải
( )
( ) ( ) ( )
( )
( ) ( ) ( )
2 2 2
2
2 2 2
2 2 2
2 2 22 2 2 2 2 2
2 2 2 2 2 2
1 1 1
) 0
2 2 2
) 3
1
0
2
a b a c b cbc ac ab
a a b c c b a
a b c ab ac bc
bc ac ab
b a b c
a b c
c a b
a b c b a c
a b c b a c
− − −
+ + − + + = + + ≥
 
+ + − + + ÷
 
 
= − + − + − ≥ 
 
Ví dụ 2.6
Chứng minh rằng
2 2
1 1 2
)
1 1 1
a
a b ab
+ ≥
+ + +
nếu ab ≥ 0
2 2
1 1 2
)
1 1 1
b
a b ab
+ ≤
+ + +
nếu a2
+ b2
< 2
2 2
1 1 2
)
1 1 1
c
a b ab
+ ≥
− − −
nếu -1 < a, b < 1
( ) ( )
2 2
1 1 1
)
11 1
d
aba b
+ ≥
++ +
nếu a, b > 0
Giải
43
Chuyªn ®Ò BDHSG To¸n THCS
( ) ( )
( )( )( )
2 2 2 2
2
2 2
1 1 2 1 1 1 1
) = +
1 1 1 1 1 1 1
1
0
1 1 1
a
a b ab a ab b ab
a b ab
a b ab
   
+ − − − ÷  ÷
+ + + + + + +   
− −
≥
+ + +
( ) ( )
( )( )( )
22 2
2 2
1 0 1
) 1 0
1 02 1 1 1
ab a b aba b
b ab
ab a b ab
+ ≥ − −+
≤ = ⇒ ⇒ ≤
− ≤ + + +
( ) ( )
( )( )( )
2 2 2 2
2 2 2
22 2
1 1 2 1 1 1 1
) = +
1 1 1 1 1 1 1
1
0
1 1 1
c
a b ab a ab b ab
a b a b
a b ab
   
+ − − − ÷  ÷
− − − − − − −   
− −
≥
− − −
( ) ( )
( ) ( )
( ) ( ) ( )
2 2
2 2 2 2
11 1 1
) = 0
11 1 1 1 1
ab a b ab
d
aba b a b ab
− + −
+ − ≥
++ + + + +
3) Phương pháp sử dung tính chất của bất đẳng thức
Cơ sở của phương pháp này là các tính chất của bất đẳng thức và một số bất đẳng thức cơ
bản như:
( )
2
) ,
1 1
) và a.b > 0
0
)
0
) 0
1 1 4
) , 0
a a b b c a c
b a b
a b
a b
c ac bd
c d
d a b
e a b
a b a b
≥ ≥ ⇒ ≥
≥ ⇒ ≤
≥ ≥
⇒ ≥
≥ ≥
− ≥
> ⇒ + ≥
+
Ví dụ 3.1
Cho a + b > 1 . Chứng minh:
4 4 1
8
a b+ >
Giải
( )
( )
( )
2
2 2 2
22 2
4 4
1
0
2 2
1
2 8
a b
a b a b
a b
a b
+
− ≥ ⇒ + ≥ >
+
⇒ + ≥ >
Ví dụ 3.2
Với a, b, c > 0. CMR
44
Chuyªn ®Ò BDHSG To¸n THCS
3 3 3
3 3 3
2 2 2
)
)
a b c
a ab ac bc
b c a
a b c
b a b c
b c a
+ + ≥ + +
+ + ≥ + +
Giải
( )
( ) ( ) ( )
( )
( ) ( ) ( )
2
2 2
3 3 3
2 2 2 2 2 2
3
2
3 3 3
2 2 2
) Ta có: , 0
) Ta có: 3 3 , 0
3 2 3 2 3 2
x
a x xy y x y
y
a b c
a ab b b bc c c ac a ab ac bc
b c a
x
b x y x y
y
a b c
a b b c c a a b c
b c a
≥ + − >
⇒ + + ≥ + − + + − + + − = + +
≥ − >
⇒ + + ≥ − + − + − = + +
Ví dụ 3.3
Cho a, b, c > 0. CMR:
2 2 2
)
)
2
bc ac ab
a a b c
a b c
a b c a b c
b
b c c a a b
+ + ≥ + +
+ +
+ + ≥
+ + +
Giải
a) dễ dàng chứng minh
2
bc ac
c
a b
+ ≥ ⇒
đpcm
b) dễ dàng chứng minh
2
4
a b c
a
b c
+
+ ≥ ⇒
+
đpcm
Ví dụ 3.4
1 1 1 1 1 1
) cho x, y, z >0. t/m: 4. : 1
2 2 2
a CMR
x y z x y z x y z x y z
+ + = + + ≤
+ + + + + +
b) Cho a, b, c là độ dài ba cạnh của một tam giác. Chứng minh
1 1 1 1 1 1
a b c a c b b c a a b c
+ + ≥ + +
+ − + − + −
c) Cho a, b, c > 0 thỏa mãn: abc = ab + bc + ca. Chứng minh:
1 1 1 3
2 3 2 3 2 3 16a b c b c a c a b
+ + ≤
+ + + + + +
Giải
45
Chuyªn ®Ò BDHSG To¸n THCS
( ) ( )
1 1 4
) Ta có: , 0
1 1 1 1 1 1 2 1 1
2 4 16
a a b
a b a b
x y z x y x zx x y x z x y z
> ⇒ + ≥
+
   
⇒ = ≤ + ≤ + + ÷  ÷
+ + + + + + +   
Tương tự:
1 1 1 2 1 1 1 1 1 2
;
2 16 2 16x y z x y z x y z x y z
   
≤ + + ≤ + + ÷  ÷
+ + + +   
1 1 1 4 1 1 1
1
2 2 2 16x y z x y z x y z x y z
 
+ + ≤ + + = ÷
+ + + + + +  
1 1 4 2
)
2
1 1 2
;
1 1 2
1 1 1 1 1 1
b
a b c a b c a a
a b c b c a b
a b c b c a c
a b c a b c b c a a b c
+ ≥ =
+ − − +
+ ≥
+ − + −
+ ≥
− + + −
⇒ + + ≥ + +
+ − − + + −
( ) ( ) ( )
1 1 1 1 1 1 1 3
)
2 3 2 4 2 16 32 32
1 3 1 1 1 1 3 1
tt: ;
3 2 32 16 32 2 3 32 32 16
1 1 1 6 1 1 1 3
2 3 3 2 2 3 32 16
c
a b c a c b c a c b c a b c
a b c a b c a b c a b c
a b c a b c a b c a b c
 
= ≤ + ≤ + + 
+ + + + + + + 
≤ + + ≤ + +
+ + + +
 
⇒ + + ≤ + + = ÷
+ + + + + +  
Ví dụ 3.5
Cho a, b, c > 0. Chứng minh rằng:
2 2 2
) 2
)
1 1 1
a b c
a
a b b c c a
a b c a b c
b
b c c a a b a b c
+ + <
+ + +
+ + ≥ + +
+ + + + + +
Giải
a) áp dụng BĐT: 0, 0. ta có:
y y t
x y t
x x t
+
> > > <
+
Ta có: ; ;
2
a a c b b a c c b
a b a b c b c b c a c a a b c
a b c
a b b c c a
+ + +
< < <
+ + + + + + + + +
⇒ + + <
+ + +
46
Chuyªn ®Ò BDHSG To¸n THCS
b)
2
2
2 2 2
1
Ta có: 1 2
1 2
3
1 1 1 2
3
mà
2
a
a a
a
a b c
a b c
a b c
b c c a a b
+ ≥ ⇒ ≤
+
⇒ + + ≤
+ + +
+ + ≥
+ + +
suy ra điều phải chứng minh.
4)Phương pháp sử dung bất đẳng thức Co-si
1 2 1 2 1 2*) Cho , ,..., 0, ta có: ... . ...n
n n na a a a a a n a a a≥ + + + ≥
Dấu “=” xảy ra khi 1 2 ... 0na a a= = = ≥
Ví dụ 4.1
Cho a, b > 0 thỏa mãn ab = 1. CMR: ( ) ( )2 2 4
1 8a b a b
a b
+ + + + ≥
+
Giải
Áp dụng BĐT Cosi ta có
( )
( )( ) ( )
( )
2 2
2 2
2 2
2 2
4 4
2 4
4 4
1 2 1
4
2 4 2 2 8
a b ab
a b ab
a b a b
a b a b
a b a b a b
a b a b
a b a b
a b
+ ≥ =
+ ≥ =
+ + ≥ + =
+ +
⇒ + + + + ≥ + + +
+ +
 
= + + + + + ≥ + + = ÷
+ 
Ví dụ 4.2
Chứng minh rằng:
( )
2
)
2 4
a b a b
a a b b a
+ +
+ ≥ + với a, b 0≥
) 2
a b c
b
b c c a a b
+ + >
+ + +
với a,b,c > 0
Giải
( )
( )
2
2
1 1
a) Ta có:
2 4 2 2 2
1 1 1
mà ,
4 4 2
2 4
a b a b a b
a b ab a b
a a b b a b a b
a b a b
a b b a
+ + +    
+ = + + ≥ + + ÷  ÷
   
 
+ ≥ + ≥ ⇒ + + ≥ + ÷
 
+ +
⇒ + ≥ +
47
Chuyªn ®Ò BDHSG To¸n THCS
1
) 1 1
2
tt: ;
b c b c a b c
b
a a a
a a
b c a b c
b b c c
a c a b c a b a b c
+ + + + 
× ≤ + = ÷
 
⇒ ≥
+ + +
≥ ≥
+ + + + + +
Cộng vế với vế ta được: 2
a b c
b c c a a b
+ + ≥
+ + +
Dấu “=” xảy ra khi 0
a b c
b a c a b c
c a b
= +

= + ⇒ + + =
 = +
vô lí.
Vậy dấu “=” không xảy ra.
Ví dụ 4.3
Cho a, b, c > 0. Chứng minh rằng:
2 2 2 3 3 3
2 2 2 2 2 2
3 3 3
2 2 2
2 2 2 2 2 2
)
2
1 1 1
) 3; ( 1)
2
a b c a b c
a
b c c a a b abc
a b c
b a b c
a b b c c a abc
+ +
+ + ≤
+ + +
+ +
+ + ≤ + + + =
+ + +
Giải
2 2 2 2 2 2
2 2 2 2 2 2
2 2 2 2 2 2 3 3 3
2 2 2 2 2 2
) Ta có: ; ;
2 2 2
2 2 2 2
a a b b c c
a
b c bc a c ac a b ab
a b c a b c a b c
b c a c a b bc ac ab abc
≤ ≤ ≤
+ + +
+ +
⇒ + + ≤ + + =
+ + +
( )2 2 2
2 2 2 2 2 2 2 2 2 2 2 2
2 2 2 3 3 3
2 2 2 2 2 2
1 1 1 1 1 1
) Ta có:
3 3
2
b a b c
a b b c c a a b b c c a
c a b a b c
a b b c c a abc
 
+ + = + + + + ÷
+ + + + + + 
+ +
= + + + ≤ +
+ + +
Ví dụ 4.4
Cho a, b, c > 0. Chứng minh rằng
3 3 3 3 3 3
1 1 1 1
a b abc b c abc c a abc abc
+ + ≤
+ + + + + +
Giải
( )
( )
2 2 3 3
3 3
Ta có: 2 0a b ab a b ab a b
abc abc c
a b abc ab a b abc a b c
+ ≥ ⇒ + ≥ + >
⇒ ≤ =
+ + + + + +
3 3 3 3
tt: ;
abc a abc b
b c abc a b c c a abc a b c
≤ ≤
+ + + + + + + +
Cộng vế với vế suy ra điều phải chứng minh
Ví dụ 4.5
Cho a, b, c > 0 thỏa mãn a2
+b2
+ c2
= 3. Chứng minh rằng
48
Chuyªn ®Ò BDHSG To¸n THCS
3 (1)
ab bc ca
c a b
+ + ≥
Gải
( ) ( ) ( )
2 2 2 2 2 2
2 2 2 2 2 2
2 2 2
2 2 2 2 2 2
2 2 2
2 2 2
1 2 3
mà :
a b b c c a
a b c a b c
c a b
a b b c c a ab bc ab ac bc ca
a b c
c a b c a c b a b
⇔ + + + + + ≥ + +
+ + ≥ × + × + × = + +
Suy ra điều phải chứng minh.
Ví dụ 4.6
Cho x, y, z > 0 thỏa mãn xyz = 1. Chứng minh
( ) ( ) ( )( ) ( )( )
3 3 3
5 5 5 5 5 5
3
)
1 1 1 1 1 1 4
) 1
x y z
a
y z z x x y
xy yz zx
b
x xy y y yz z z zx x
+ + ≥
+ + + + + +
+ + ≤
+ + + + + +
Giải
a) Áp dụng bất đẳng thức cô si ta có:
( ) ( )
3
1 1 3
1 1 8 8 4
x y z x
y z
+ +
+ + ≥
+ +
Tương tự suy ra VT
333 3 3
2 4 2 4 4
xyzx y z+ +
≥ − ≥ − =
( )
( ) ( )
2 2 5 5 2 2
5 5 2 2
5 5 5 5
5 5 5 5 5 5
) Ta có: 2 0
1
1
tt: ;
1
b x y xy x y x y x y
xy xy z
x xy y xy x y x y xy x y x y z
yz x zx y
y yz z x y z z xz x x y z
xy yz zx
x xy y y yz z z xz x
+ ≥ ⇒ + ≥ + >
≤ = =
+ + + + + + + +
≤ ≤
+ + + + + + + +
⇒ + + ≤
+ + + + + +
Ví dụ 4.7
Cho x, y, z > 0. Chứng minh
3 3 3
x y z
x y z
yz zx xy
+ + ≥ + +
Giải
Áp dụng bất đẳng thức Cosi ta có:
3 3 3
3 3 3
3 ; 3 ; 3
x y z
z y x z x y x y z
yz zx xy
x y z
x y z
yz zx xy
+ + ≥ + + ≥ + + ≥
⇒ + + ≥ + +
5)Phương pháp sử dung bất đẳng thức Bunhiacopski
49
Chuyªn ®Ò BDHSG To¸n THCS
*) ( )( ) ( )
22 2 2 2
a b x y xa by+ + ≥ + dấu “=” xảy ra khi
a kx
b ky
=

=
*) ( )( ) ( )
22 2 2 2 2 2
a b c x y z xa by cz+ + + + ≥ + + dấu “=” xảy ra khi
a kx
b ky
c kz
=

=
 =
Tổng quát:
( )( ) ( )
22 2 2 2 2 2
1 2 1 2 1 1 2 2... ... ...n n n xa a a x x x a x a x a x+ + + + + + ≥ + + + dấu “=” xảy ra khi ai = kxi
Ví dụ 5.1
Cho a, b > 0. Chứng minh
( )
22 2
1 1 4
)
)
a
a b a b
m nn m
b
a b a b
+ ≥
+
+
+ ≥
+
Giải
a) Áp dụng bất đẳng thức Bunhiacopski ta có:
( )
2
1 1 1 1
. . 4
1 1 4
a b a b
a b a b
a b a b
  
+ + ≥ + = ÷  ÷
   
⇒ + ≥
+
( ) ( )
( )
22 2
2
22 2
) . .
n m n m
b a b a b n m
a b a b
n mn m
a b a b
   
+ + ≥ + = + ÷  ÷
  
+
⇒ + ≥
+
Tổng quát:
• Cho 0, 1.ib i n> = thì
( )
22 2 2
1 21 2
1 2 1 2
...
...
...
nn
n n
a a aa a a
b b b b b b
+ + +
+ + + ≥
+ + +
(1)
• Với 0i ia c > với 1.i n= thì
( )
2
1 21 2
1 2 1 1 2 2
...
...
...
nn
n n n
a a aa a a
c c c a c a c a c
+ + +
+ + + ≥
+ + +
(2)
Thật vậy:
( ) ( )
( )
2
2 2 2
21 2 1 2
1 2 1 2 1 2
1 2 1 2
22 2 2
1 21 2
1 2 1 2
... ... . . ... . ...
...
...
...
n n
n n n
n n
nn
n n
a a a a a a
b b b b b b a a a
b b b b b b
a a aa a a
b b b b b b
  
+ + + + + + ≥ + + + = + + + ÷ ÷  ÷   
+ + +
⇒ + + + ≥
+ + +
đặt aici = bi > 0 thay vào (1) được (2)
Ví dụ 5.2
Cho a,b,c là các số thực dương. Chứng minh
50
Chuyªn ®Ò BDHSG To¸n THCS
2 2 2 2 2 2
3 3 3 3 3 3 2 2 2
2 2 2
) . )
2
) )
2
a b c a b c a b c
a a b c b
b c a b c c a a b
a b c a b c a b c
c a b c d
b c a b c c a a b
+ +
+ + ≥ + + + + ≥
+ + +
+ +
+ + ≥ + + + + ≥
+ + +
Giải
( )
( )
( )
( )
22 2 2
22 2 2
22 2 23 3 3 4 4 4
2 2 2
3 3 3 4 4 4 2 2 2
) Ta có:
)
2 2
)
)
2
a b ca b c
a a b c
b c a a b c
a b ca b c a b c
b
b c c a a b a b c
a b ca b c a b c
c a b c
b c a ab bc ca ab bc ca
a b c a b c a b c
d
b c a c a b ab ac ab bc ac bc
+ +
+ + ≥ = + +
+ +
+ + + +
+ + ≥ =
+ + + + +
+ +
+ + = + + ≥ ≥ + +
+ +
+ +
+ + = + + ≥
+ + + + + +
Ví dụ 5.3
Cho a, b, c > 0. Chứng minh:
25 16
8
a b c
b c c a a b
+ + >
+ + +
Giải
( ) ( )
( )
( )
2
Ta có: 25 1 16 1 1 42
5 4 125 16 1
42 8
2
a b c
VT
b c c a a b
a b c a b c
b c c a a b a b c
   
= + + + + + − = ÷  ÷
+ + +   
+ + 
+ + + + ≥ + + − = ÷
+ + + + + 
Dấu “=” xảy ra khi 0
5 4 1
b c a c a b
a
+ + +
= + ⇒ = vô lí suy ra điều phải chứng minh.
Ví dụ 5.4
Cho x, y, z > 0. Chứng minh:
2 2 2
2 2 2
x y z x y z
y z x y z x
+ + ≥ + +
Giải
Áp dụng bất đẳng thức Bunhiacopki ta có
22 2 2
2 2 2
1 1
3 3
x y z x y z x y z x y z x y z
y z x y z x y z x y z x y z x
    
+ + ≥ + + ≥ + + + + ≥ + + ÷  ÷ ÷
    
B – CÁC PHƯƠNG PHÁP TÌM GIÁ TRỊ LỚN NHẤT, GIÁ TRỊ NHỎ NHẤT
Cho biểu thức f(x,y…)
• Ta nói M là giá trị lớn nhất của f(x,y…) kí hiệu maxf(x,y…) = M, nếu hai điều kiện
sau được thỏa mãn:
- Với mọi x,y… để f(x,y…) xá định thì f(x,y…) ≤ M
- Tồn tại x0, y0… sao cho f(x0,y0…) = M
51
Chuyªn ®Ò BDHSG To¸n THCS
• Ta nói m là giá trị nhỏ nhất của f(x,y…) kí hiệu minf(x,y…) = m, nếu hai điều kiện
sau được thỏa mãn:
- Với mọi x,y… để f(x,y…) xá định thì f(x,y…) ≥ m
- Tồn tại x0, y0… sao cho f(x0,y0…) = m
I) TÌM GTLN, GTNN CỦA ĐA THỨC BẬC HAI
1) Đa thức bậc hai một biến
Ví dụ 1.1
a) Tìm GTNN của A = 3x2
– 4x + 1
b) Tìm GTLN của B = - 5x2
+ 6x – 2
c) Tìm GTNN của C = (x – 2)2
+ (x – 3)2
d) Cho tam thức bậc hai P = ax2
+ bx + c
Tìm GTNN của P nếu a > 0
Tìm GTNN của P nếu a > 0
Giải
a) A =
2
2 4 4 1 2 1 1
3 3
3 9 3 3 3 3
x x x
   
− + − = − − ≥ − ÷  ÷
   
. Vậy minA=
1 2
khi
3 3
x− =
b) B =
2
2 6 9 1 3 1 1
5 5
5 25 5 5 5 5
x x x
   
− − + − = − − − ≤ − ÷  ÷
   
. Vậy maxB =
1 3
khi
5 5
x− =
c) C =
2
2 2 2 25 1 5 1 1
4 4 6 9 2 5 2
4 2 2 2 2
x x x x x x x
   
− + + − + = − + + = − + ≥ ÷  ÷
   
.
Vậy maxC =
1 5
khi
2 2
x =
d) Ta có: P =
22 2 2
2
4 4 2 4
b b b b b
a x x c a a c
a a a a a
   
+ + + − = + + − ÷  ÷
  
Nếu a > 0 thì P ≥
2
4
b
c
a
− . Vậy minP =
2
4
b
c
a
− khi
2
b
x
a
= −
Nếu a < 0 thì P ≤
2
4
b
c
a
− . Vậy maxP =
2
4
b
c
a
− khi
2
b
x
a
= −
Ví dụ 1.2
a) Tìm GTNN của M = x2
– 3x + 1 với 2x ≥
b) Tìm GTLN của N = x2
– 5x + 1 với 3 8x− ≤ ≤
Giải
a) M = ( ) ( )1 2 1 1x x− − − ≥ − . Vậy minM = -1 khi x = 2
b) N = ( ) ( )3 8 25 25x x+ − + ≤ . Vậy maxN = 25 khi x = -3, x = 8
2. Đa thức bậc hai hai biến
a) Đa thức bậc hai hai biến có điều kiện
Ví dụ 2a.1
a) Cho x + y = 1. Tìm GTLN của P = 3xy – 4
b) Cho x – 2y = 2. Tìm GTNN của Q = x2
+ 2y2
– x + 3y
Giải
a) ( )
2
1 13 13
1 1 3 1 4 3
2 4 4
x y x y P y y y
− 
+ = ⇒ = − ⇒ = − − = − − − ≤ ÷
 
52
Chuyªn ®Ò BDHSG To¸n THCS
Vậy maxP =
13 1
khi
4 2
x
−
=
2 2
2 2
b) 2 2 2 2 4 8 4 2 2 2 3
3 9 11 11
6 9 2 6
2 16 8 8
x y x y Q y y y y y
y y y y
− = ⇒ = + ⇒ = + + + − − +
 
= + + = + + − ≥ − ÷
 
Vậy minQ =
11 3
khi
8 4
x− = −
Ví dụ 2a.2
Tìm GTLN của của P = xy vói x, y thỏa mãn
a) 6, 4x y y+ = ≥
b) ,
2
S
x y S y a+ = ≥ >
a) ( ) ( )( )6 8 2 4 8P y y y y= − = − − − ≤ . Vậy maxP = 8 khi x = 2, y = 4
b) ( ) ( ) ( ) ( ) ( )Q S y y S a a y a y a S S a a= − = − − − + − ≤ − .
Vậy maxQ = (S – a)a khi x = S – a, y = a
b) Đa thức bậc hai hai biến
Cho đa thức: P(x,y) = ax2
+ bxy + cy2
+ dx + ey + h (1), với a,b,c ≠ 0
Ta thường đưa P(x, y) về dạng
P(x, y) = mF2
(x, y) + nG2
(y) + k (2)
P(x, y) = mH2
(x, y) + nG2
(x) + k (3)
Trong đó G(y), H(x) là hai biểu thức bậc nhất một ẩn, H(x, y) là biểu thức bậc nhất hai
ẩn.
Chẳng hạn nếu ta biến đổi (1) về (2) với a, (4ac – b2
) ≠ 0
( ) ( )
( ) ( ) ( )
2 2 2
2 2 2 2 2 2 2
22
2 2 2
2 2
4 ( , ) 4 4 4 4 4 4
4 4 4 2 4 2 2 4
22
2 4 4
4 4
aP x y a x abxy acy adx aey ah
a x b y d abxy adx bdy ac b ae bd y ah d
ae bdae bd
ax by d ac b y ahd
ac b ac b
= + + + + +
= + + + + + + − + − + −
−− 
= + + + − + + − ÷
− − 
(Tương tự nhân hai vế của (1) với 4c để chuyển về (3))
Ví dụ 3.1
a) Tìm GTNN của P = x2
+ y2
+ xy + x + y
b) Tìm GTLN của Q = -5x2
– 2xy – 2y2
+ 14x + 10y – 1
Giải
( )
2 2 2 2 2
2
2
a) 4 4 4 4 4 4 4 1 4 4 2 3 2 1
1 4 4
= 2 1 3
3 3 3
P x y xy x y x y xy x y y y
x y y
= + + + + = + + + + + + + −
 
+ + + + − ≥ − ÷
 
Vậy minP =
4 1
khi
3 3
x y− = = −
( ) ( )
( ) ( )
2 22 2
2 2
b) 5 25 10 10 70 50 5 = 5 7 3 6 80
1 9
5 7 2 16 16
5 5
Q x xy y x y x y y
Q x y y
− = + + − − + − − + − −
⇒ = − − − − − + ≤
Vậy maxQ = 16 khi x = 1, y = 2
Ví dụ 3.2
53
Chuyªn ®Ò BDHSG To¸n THCS
Tìm cặp số (x, y) với y nhỏ nhất thỏa mãn: x2
+ 5y2
+ 2y – 4xy – 3 = 0 (*)
Giải
( ) ( ) ( ) ( )( )
2 2 2
(*) 2 1 4 1 4 3 1 0
3 1
x y y y y y
y
⇔ − + + = ⇒ + ≤ ⇒ + − ≤
⇒ − ≤ ≤
Vậy miny = -3 khi x = -6. Vậy ccawpj số (x, y) = (-6; -3)
Ví dụ 3.3
Cho x, y liên hệ với nhau bởi hệ thức x2
+ 2xy + 7(x + y) + 7y2
+ 10 = 0 (**).
Hãy tìm GTLN, GTNN của S = x + y + 1.
Giải
( )
( ) ( )
( ) ( ) ( )
2 2
2 22
** 4 8 28 28 4 40 0
2 2 7 4 9 2 2 7 9
5 0
5 2 0 vì 2 5
2 0
4 1
x xy x y y
x y y x y
x y
x y x y x y x y
x y
S
⇔ + + + + + =
⇔ + + + = ⇒ + + ≤
+ + ≥
⇔ + + + + ≤ ⇔ + + ≤ + +
+ + ≤
⇔ − ≤ ≤ −
Vậy minS = -4 khi x = -5, y = 0. maxS = -1 khi x = -2, y = 0.
II. PHƯƠNG PHÁP MIỀN GIÁ TRỊ
Ví dụ 1
Tìm GTLN, GTNN của A =
2
2
4 2 3
1
x x
x
+ +
+
Giải
Biểu thức A nhận giá trị a khi phương trình sau đây cóa nghiệm
a =
2
2
4 2 3
1
x x
x
+ +
+
( ) ( )2
1 4 2 3 0 1a x x a⇔ − − + − =
Nếu a = 1 thì phương trình (1) có nghiệm x =
2
4
−
Nếu a ≠ 1 thì phương trình (1) có nghiệm khi –a2
+ 4a +5 0 1 5.a≥ ⇔ − ≤ ≤
Vậy minA = -1 khi 2x = −
maxA = 5 khi
2
2
x =
Ví dụ 2
Tìm GTLN, GTNN của biểu thức B = 2 2
2 1
7
x y
x y
+ +
+ +
Giải
Biểu thức B nhận giá trị b khi phương trình sau có nghiệm
b = 2 2
2 1
7
x y
x y
+ +
+ +
( )2 2
2 7 1 0 2bx x by y b⇔ − + − + − =
Trong đó x là ẩn, y là tham số và b là tham số có điều kiện
Nếu b = 0 2 1 0x y⇒ + + =
54
Chuyªn ®Ò BDHSG To¸n THCS
Nếu b ≠ để (2) có nghiệm x khi 1 – 4b(by2
– 2y + 7b -1) 0≥ (3)
Coi (3) là bất phương trình ẩn y. BPT này xảy ra với mọi giá trị của y khi
16b2
+ 4b2
(-28b2
+ 4b + 1) 0≥
2 5 1
28 4 5 0
14 2
b b b⇔ − + + ≥ ⇔ − ≤ ≤
Vậy minB =
5 7 14
khi x = - ,
14 5 5
y− = −
maxB =
1
khi x = 1, y = 2
2
III. PHƯƠNG PHÁP SỬ DỤNG BẤT ĐẲNG THỨC
1) Sử dụng bất đẳng thức Cô-si
Ví dụ 1.1
Tìm GTLN, GTNN của A = 3 5 7 3x x− + − với
5 7
3 3
x≤ ≤
Giải
( ) ( ) ( )( )2
3 5 7 3 2 3 5 7 3 2 2 3 5 7 3A x x x x x x= − + − + − − = + − −
Vậy A2
2 2.A≥ ⇒ ≥ Vậy minA =
5 7
2 khi ,
3 3
x y= =
2
4 2 max 2 khi 2A A A x≤ ⇒ ≤ ⇒ = =
(Biểu thức được cho dưới dạng tổng hai căn thức. Hai biểu thức lấy căn có tổng là
hằng số)
Ví dụ 1.2
Cho x, y > 0 thỏa mãn x + y 6≥ . Hãy tìm GTNN của P =
6 8
3 2x y
x y
+ + +
Giải
Ta có: ( )
3 3 6 8 3 3 6 8
.6 2 . 2 . 9 6 4 19
2 2 2 2 2 2
x y x y
P x y
x y x y
= + + + + + ≥ + + = + + =
Vậy minP = 19 khi x = 2, y = 4.
Ví dụ 1.3
Tìm GTLN của biểu thức M =
2 3x y y x
xy
− + −
với 3; 2x y≥ ≥
Giải
To có:
23 yx
M
x y
−−
= +
Theo bất đẳng thức Cô – si ta có:
( ) ( )
23 3 2
3 3 ; 2 2
2 6 2 4
yx x y
x y
x y
−−
− ≤ ⇒ ≤ − ≤ ⇒ ≤
55
Chuyªn ®Ò BDHSG To¸n THCS
3 2 3 2
max khi 6; 4
6 4 6 4
M M x y⇒ ≤ + ⇒ = + = =
Ví dụ 1.4
Cho x, y, z > 0 thỏa mãn: ( )
1 1 1
2 1
1 1 1x y z
+ + ≥
+ + +
. Tìm TGLN của P = xyz
Giải
( )
( ) ( )
1 1 1
1 1 1 2
1 1 1 1 1 1 1
y z z
x y z y z y z
   
⇒ ≥ − + + = + ≥ ÷ ÷
+ + + + + + +  
Tương tự:
( )( ) ( ) ( )
1 1
2 ; 2
1 1 1 1 1 1
xz xy
y x z z x y
≥ ≥
+ + + + + +
Nhân vế với vế của ba BĐT trên
1 1 1
max khi
8 8 2
P xyz P x y z⇒ = ≤ ⇒ = = = =
Ví dụ 1.5
Cho 0 < x < 1, Tìm GTNN của Q =
3 4
1 x x
+
−
Giải
( ) ( )
( )
( ) ( )
( )
2
2 2
4 1 4 13 3
Ta có: 7 2 . 7 2 3
1 1
4 13
minP = 2 3 khi 3 1
1
x xx x
P
x x x x
xx
x
x x
− −
= + + ≥ + = +
− −
−
⇒ + = ⇒ = −
−
(Đặt P =
( )4 1-3
1-
b xax
c
x x
+ + đồng nhất hệ số suy ra a = b = 1; c = 7)
Ví dụ 1.6
Cho x, y, z, t > 0. Tìm GTNN của biểu thức.
M
x t t y y z z x
t y y z z x x t
− − − −
= + + +
+ + + +
Giải
Áp dụng bất đẳng thức Cô-si ta có:
1 1 4
a b a b
+ ≥
+
với a, b > 0.
( ) ( )
( ) ( )
Ta có: 4 4 1 1 1 1 4
1 1 1 1
= 4
4 4
x t t y y z z x
M M
t y y z z x x t
x y t z y x z t
x y z t
t y y z z x x t t y z x y z x t
x y z t
x y z t x y z t
   − − − −   
= + − = + + + + + + + − ÷  ÷ ÷  ÷
+ + + +      
   + + + +
+ + + − = + + + + + ÷  ÷
+ + + + + + + +   
+ +
≥ + −
+ + + + + +
4 0
minM = 0 khi x = y và z t
=
⇒ =
2. Sử dụng BĐT Bunhiacopski (BCS)
Ví dụ 2.1
Cho x, y, z thỏa mãn: xy + yz + zx =1. Tìm GTNN của biểu thức
A = x4
+ y4
+ z4
Giải
56
Chuyªn ®Ò BDHSG To¸n THCS
Áp dụng BĐT BCS ta có
( ) ( )( ) ( )
( ) ( )( )
22 2 2 2 2 2 2 2 2 2
22 2 2 4 4 4
1
1 1 1 1
1 1 3
minP = khi x = y = z =
3 3 3
xy yz zx x y z x y z x y z
x y z x y z
P
= + + ≤ + + + + = + +
⇒ ≤ + + ≤ + + + +
⇒ ≥ ⇒
Ví dụ 2.2
Tìm GTNN của P =
4 9 16a b c
b c a c a b a b c
+ +
+ − + − + −
trong đó a, b, c là độ dài ba cạnh
của một tam giác.
Giải
( )
( ) ( ) ( )
2
1 1 1 29
4 9 16
2 2 2 2
4 9 16 29
=
2 2
2 3 4 29
.
2 2
81 29 81 29
. 26
2 2 2 2
minP = 26 khi
7 6 5
a b c
P
b c a c a b a b c
a b c
b c a c a b a b c
a b c
b c a c a b a b c
a b c
a b c
a b c
     
= + + + + + − ÷  ÷  ÷
+ − + − + −     
+ +  
+ + − ÷
+ − + − + − 
+ ++ +
≥ −
+ − + + − + + −
+ +
= − = − =
+ +
⇒ = =
Ví dụ 2.3
Tìm giá trị nhỏ nhất của Q = + +
trong đó a, b, c > 0 thỏa mãn a + b + c = 1
Giải
( )
( ) ( ) ( )
( )
( )
2 2
Ta có: 1
2 2 2 2 2 2 3
1
minQ = 1 khi
3
a b c a b ca b c
Q
b c c a a b a b c b c a c a b ab bc ca
a b c
+ + + +
= + + ≥ = ≥
+ + + + + + + + + +
⇒ = = =
Ví dụ 2.4
Cho a, b, c > 0 thỏa mãn a + b + c = 1. Tìm GTNN của
2 2 2
1 1 1 1
P
a b c ab bc ca
= + + +
+ +
Giải
57
A
M
I OC
E
F
N
B
M'
Chuyªn ®Ò BDHSG To¸n THCS
( )
( )
( ) ( )
2 2 2
2 2 2
2
2 2
1 1 1 9
1 9
1 4 7
2
1 2 9 21
9 30
1
minP = 30 khi a = b = c =
3
ab bc ca ab bc ca
P
a b c ab bc ca
a b c ab bc ca ab bc ca
ab bc caa b c a b c
+ + ≥
+ +
⇒ ≥ +
+ + + +
 
= + + ÷
+ + + + + + 
+
≥ + ≥ + =
+ ++ + + +
⇒
Chuyªn ®Ò 5:
:
: Tø gi¸c néi tiÕp
I -c¸c dÊu hiÖu nhËn biÕt tø gi¸c néi tiÕp
1- Tæng hai gãc ®èi b»ng 1800
2- Hai gãc liªn tiÕp cïng nh×n mét c¹nh díi hai gãc b»ng nhau.
3- NÕu hai c¹nh ®èi diÖn cu¶ gi¸c ABCD c¾t nhau t¹i M tháa m·n:
MA.MB =MC.MD ; hoÆc hai ®êng chÐo c¾t nhau t¹i O tháa m·n
OA.OC = OB.OD th× ABCD lµ tø gi¸c néi tiÕp
4- Sö dông ®Þnh lý Pt«lªmª
II- C¸c vÝ dô
VÝ dô1: Cho ®êng trßn t©m O vµ mét ®iÓm C ë ngoµi ®êng trßn ®ã. Tõ C kÎ hai
tiÕp tuyÕn CE ; CF ( E vµ F lµ c¸c tiÕp ®iÓm) vµ c¸t tuyÕn CMN ( N n»m gi÷a C
vµ M ) tíi ®êng trßn.§êng th¼ng CO c¾t ®êng trßn t¹i hai ®iÓm A vµ B. Gäi I lµ
giao ®iÓm cña AB víi EF. Chøng minh r»ng:
a, Bèn ®iÓm O, I, M, N cïng thuéc mét ®êng trßn
b, =
Gi¶i
a, Do CE lµ tiÕp tuyÕn cña (O) nªn:
= (Cïng ch¾n )
 ∆CEM ~ ∆CNE .
 =
 CM.CN =CE2
MÆt Kh¸c , do CE; CF lµ c¸c tiÕp tuyÕn cña (O) nªn
58
O
O'
A
B
C
C'
B'
Chuyªn ®Ò BDHSG To¸n THCS
AB⊥ EF t¹i I v× vËy trong tam gi¸c vu«ng CEO ®êng cao EI ta cã:
CE2
= CI.CO
Tõ (1) vµ (2) suy ra CM.CN = CI.CO => =
 ∆CMI ~ ∆CON
 =
 Tø gi¸c OIMN néi tiÕp 
b KÐo dµi NI c¾t ®êng trßn t¹i M’.
Do tø gi¸c IONM néi tiÕp nªn :
= = s®
=> = . Do ®ã:
= = 
VÝ Dô 2
Cho tam gi¸c ABC cã = 450
; BC =a néi tiÕp trong ®êng trßn t©m O; c¸c ®êng
cao BB’ vµ CC’. Gäi O’ lµ ®iÓm ®èi xøng cña O qua ®êng th¼ng B’C’.
a. Chøng minh r»ng A; B’; C’; O’ cïng thuéc mét ®êng trßn
b. TÝnh B’C’ theo a.
Lêi gi¶i
a. Do O lµ t©m ®êng trßn ngo¹i tiÕp tam gi¸c ABC nªn
= 2 =900
Tõ ®ã suy ra c¸c ®iÓm O; B’; C’
Cïng thuéc ®êng trßn ®êng kÝnh BC.XÐt
tø gi¸c néi tiÕp CC’OB’ cã :
= 1800
-
= 1800
- ( 900
- ) =1350
.
Mµ O’ ®èi xøng víi O qua B’C’ nªn:
= = 1350
=1800
-
Hay tø gi¸c AC’O’B’ néi tiÕp.
b. Do = 450
nªn ∆BB’A vu«ng c©n t¹i B’
V× vËy B’ n»m trªn ®êng trung trùc cña ®o¹n AB hay B’O ⊥ AB
 C’OB’C lµ h×nh thang c©n nªn B’C’ =OC
MÆt kh¸c ∆BOC vu«ng c©n nªn: B’C’ =OC =
2
2
2
2 aBC
=
III bµi tËp ¸p dông
Bài tập 1:
Cho tứ giác ABCD nội tiế đường tròn đường kính AD. Hai đường chéo AC và BD cắt
nhau tại E. Vẽ EF vuông góc với AD. Chứng minh:
59
Chuyªn ®Ò BDHSG To¸n THCS
a/ Tứ giác EBEF, tứ giác DCEF nội tiếp.
b/ CA là phân giác của ·BCF
c/ Gọi M là trung điểm của DE. Chứng minh tứ giác BCMF nội tiếp.
Bài tập 2:
Tứ giác ABCD nội tiếp đường tròn đường kính AD. Hai đường chéo AC, BD cắt nhau
tại E. Hình chiếu vuông góc của E trên AD là F. Đường thẳng CF cắt đường tròn tại điểm
thứ hai là M. Giao điểm của BD và CF là N. Chứng minh:
a/ CEFD là tứ giác nội tiếp
b/ Tia FA là phân giác của góc BFM
c/ BE.DN = EN.BD.
Bài tập 3:
Cho tam giác ABC vuông ở A và một điểm D nằm giữa A và B. Đường tròn đường
kính BD cắt BC tại E. Các đường thẳng CD, AE lần lượt cắt đường tròn tại các điểm thứ
hai F, G. Chứng minh:
a/ Tam giác ABC đồng dạng với tam giác EBD
b/ Tứ giác ADEC và AFBC nội tiếp được một đường tròn
c/ AC song song với FG
d/ Các đường thẳng AC, DE, BF đồng quy.
Bài tập 4:
Cho tam giác ABC có 0ˆ 90A = ; AB > AC, và một điểm M nằm trên đoạn AC ( M không
trùng với A và C ). Gọi N và D lần lượt là giao điểm thứ hai của BC và MB với đường
tròn đường kính MC; gọi S là giao điểm thứ hai giữa AD với đường tròn đường kính MC;
T là giao điểm của MN và AB. Chứng minh:
a/ Bốn điểm A, M, N, B cùng thuộc một đường tròn
b/ CM là phân giác của góc BCS.
c/
TA TC
TD TB
=
Bài tập 5:
Cho đường tròn (O) và điểm A nằm ngoài đường tròn. Qua A dựng hai tiếp tuyến AM
và AN với đường tròn ( M, N là các tiếp điểm ) và một cact tuyến bất kỳ cắt đường tròn tại
P, Q. Gọi L là trung điểm của PQ.
a/ Chứng minh 5 điểm: O, L, M, A, N cùng thuộc một đường tròn
b/ Chứng minh LA là phân giác của góc MLN
c/ Gọi I là giao điểm của MN và LA. Chứng minh: MA2
= AI. AL
d/ Gọi K là giao điểm của ML với (O). Chứng minh rằng: KN // AQ
e/ Chứng minh tam giác KLN cân.
60
Chuyªn ®Ò BDHSG To¸n THCS
Bài tập 6:
Cho đường tròn (O;R) tiếp xúc với đường thẳng d tại A. Trên d lấy điểm H không trùng
với điểm A và AH < R. Qua H kẻ đường thẳng vuông góc với d, đường thẳng này cắt
đường tròn tại hai điểm E và B ( E nằm giữa B và H )
a/ Chứng minh: góc ABE bằng góc EAH và tam giác AHB đồng dạng với
tam giác EAH.
b/ Lấy điểm C trên d sao cho H lá trung điểm của đoạn AC, đường thẳng CE
cắt AB tại K. Chứng minh: AHEK là tứ giác nội tiếp
c/ Xác định vị trí của điểm H để AB = R 3
Bài tập 7:
Từ điểm P nằm ngoài đường tròn (O), kẻ hai tiếp tuyến PM và PN với đường tròn (O) (
M, N là tiếp điểm ). Đường thẳng đi qua điểm P cắt đường tròn (O) tại hai điểm E và F.
Đường thẳng qua O song song với MP cắt PN tại Q. Gọi H là trung điểm của đoạn EF.
Chứng minh:
a/ Tứ giác PMON nội tiếp đường tròn
b/ Các điểm P, N, O, H cùng nằm trên một đường tròn
c/ Tam giác PQO cân
d/ MP2
= PE. PF
e/ =
Bài tập 8:
Cho tam giác ABC có ba góc nhọn nội tiếp đường tròn (O). Các đường cao AD, BE, CF
cắt nhau tại H và cắt đường tròn (O) lần lượt tại M, N, P.
Chứng minh rằng:
a/ Các tứ giác AEHF, BFHD nội tiếp.
b/ Bốn điểm B, C, E, F cùng nằm trên một đường tròn.
c/ AE. AC = AH. AD và AD. BC = BE. AC
d/ H và M đối xứng nhau qua BC
e/ Xác định tâm đường tròn nội tiếp tam giác DEF.
Bài tập 9:
Cho tam giác ABC không cân, đường cao AH, nội tiếp trong đường tròn tâm O. Gọi E,
F thứ tự là hình chiếu của B, C lên đường kính AD của đường tròn (O) và M, N thứ tự là
trung điểm của BC, AB. Chứng minh:
a/ Bốn điểm A, B, H, E cùng nằm trên một đường tròn tâm N và HE // CD.
b/ M là tâm đường tròn ngoại tiếp tam giác HEF.
61
Chuyªn ®Ò BDHSG To¸n THCS
Bài tập 10:
Cho đường tròn (O) và điểm A ở bên ngoài đường tròn. Vẽ các tiếp tuyến AB, AC và
cát tuyến ADE với đường tròn ( B và C là các tiếp điểm ). Gọi H là trung điểm của DE.
a/ CMR: A, B,H, O, C cùng thuộc một đường tròn. Xác định tâm của đường
tròn này.
b/ Chứng minh: HA là tia phân giác .
c/ Gọi I là giao điểm của BC và DE. Chứng minh: AB2
= AI.AH
d/ BH cắt (O) tại K. Chứng minh: AE // CK.
Bài tập 11:
Từ một điểm S ở ngoài đường tròn (O) vẽ hai tiếp tuyến SA, SB và cát tuyến SCD của
đường tròn đó.
a/ Gọi E là trung điểm của dây CD. Chứng minh 5 điểm S, A, E, O, B cùng thuộc
một đường tròn.
b/ Nếu SA = AO thì SAOB là hình gì? Tại sao?.
c/ CMR: AC.BD = BC.DA =
.
2
AB CD
Bài tập 12:
Trên đường thẳng d lấy 3 điểm A, B, C theo thứ tự đó. Trên nửa mặt phẳng bờ d kẻ hai
tia Ax, By cùng vuông góc với d. Trên tia Ax lấy I. Tia vuông góc với CI tại C cắt By tại
K. Đường tròn đường kính IC cắt IK tại P.
a/ Chứng minh tứ giác CBPK nội tiếp được đường tròn
b/ Chứng minh: AI. BK = AC. CB
c/ Giả sử A, B, I cố định hãy xác định vị trí điểm C sao cho diện tích hình thang
vuông ABKI lớn nhất.
Bài tập 13:
Cho tam giác đều ABC nội tiếp đường tròn (O). M là điểm di động trên cung nhỏ BC.
Trên đoạn thẳng MA lấy điểm D sao cho MD = MC.
a/ Chứng minh: VDMC đều
b/ Chứng minh: MB + MC = MA
c/ Chứng minh tứ giác ADOC nội tiếp được.
d/ Khi M di động trên cung nhỏ BC thì D di động trên đường cố định nào?.
Bài tập 14:
62
Boi duong hsg
Boi duong hsg
Boi duong hsg
Boi duong hsg
Boi duong hsg
Boi duong hsg
Boi duong hsg
Boi duong hsg
Boi duong hsg
Boi duong hsg
Boi duong hsg
Boi duong hsg
Boi duong hsg
Boi duong hsg
Boi duong hsg
Boi duong hsg
Boi duong hsg
Boi duong hsg
Boi duong hsg
Boi duong hsg
Boi duong hsg

More Related Content

What's hot

Lich trieu hien chuong loai chi nxb giao duc-tap 1
Lich trieu hien chuong loai chi nxb giao duc-tap 1Lich trieu hien chuong loai chi nxb giao duc-tap 1
Lich trieu hien chuong loai chi nxb giao duc-tap 1Đỗ Đạt
 
Lich trieu hien chuong loai chi nxb giao duc-tap 1
Lich trieu hien chuong loai chi nxb giao duc-tap 1Lich trieu hien chuong loai chi nxb giao duc-tap 1
Lich trieu hien chuong loai chi nxb giao duc-tap 1Nguyen Hoang Le
 
Tailieu.vncty.com boi-duong-tu-duy-sang-tao-qua-giai-bt-hinh-hoc
Tailieu.vncty.com   boi-duong-tu-duy-sang-tao-qua-giai-bt-hinh-hocTailieu.vncty.com   boi-duong-tu-duy-sang-tao-qua-giai-bt-hinh-hoc
Tailieu.vncty.com boi-duong-tu-duy-sang-tao-qua-giai-bt-hinh-hocTrần Đức Anh
 
đề Tài ứng dụng công nghệ thông tin trong giảng dạy phân môn thường thức mĩ ...
đề Tài ứng dụng công nghệ thông tin trong giảng dạy phân môn thường thức mĩ ...đề Tài ứng dụng công nghệ thông tin trong giảng dạy phân môn thường thức mĩ ...
đề Tài ứng dụng công nghệ thông tin trong giảng dạy phân môn thường thức mĩ ...nataliej4
 
Tiểu Luận Tiềm Năng Và Thực Trạng Phát Triển Du Lịch Văn Hoá Ở Hà Nội
Tiểu Luận Tiềm Năng Và Thực Trạng Phát Triển Du Lịch Văn Hoá Ở Hà Nội Tiểu Luận Tiềm Năng Và Thực Trạng Phát Triển Du Lịch Văn Hoá Ở Hà Nội
Tiểu Luận Tiềm Năng Và Thực Trạng Phát Triển Du Lịch Văn Hoá Ở Hà Nội nataliej4
 
Lien bp su dung do dung khi day tre ke chuyen trong hd lqvh
Lien   bp su dung do dung khi day tre ke chuyen trong hd lqvhLien   bp su dung do dung khi day tre ke chuyen trong hd lqvh
Lien bp su dung do dung khi day tre ke chuyen trong hd lqvhtieuhocanthai
 
Bai 15 quyen va nghia vu hoc tap
Bai 15  quyen va nghia vu hoc tapBai 15  quyen va nghia vu hoc tap
Bai 15 quyen va nghia vu hoc tapHoa Phượng
 
Luan van ve hop dong chuyen nhuong quyen su dung dat
Luan van ve hop dong chuyen nhuong quyen su dung datLuan van ve hop dong chuyen nhuong quyen su dung dat
Luan van ve hop dong chuyen nhuong quyen su dung datHung Nguyen
 

What's hot (10)

Lich trieu hien chuong loai chi nxb giao duc-tap 1
Lich trieu hien chuong loai chi nxb giao duc-tap 1Lich trieu hien chuong loai chi nxb giao duc-tap 1
Lich trieu hien chuong loai chi nxb giao duc-tap 1
 
Lich trieu hien chuong loai chi nxb giao duc-tap 1
Lich trieu hien chuong loai chi nxb giao duc-tap 1Lich trieu hien chuong loai chi nxb giao duc-tap 1
Lich trieu hien chuong loai chi nxb giao duc-tap 1
 
Tailieu.vncty.com boi-duong-tu-duy-sang-tao-qua-giai-bt-hinh-hoc
Tailieu.vncty.com   boi-duong-tu-duy-sang-tao-qua-giai-bt-hinh-hocTailieu.vncty.com   boi-duong-tu-duy-sang-tao-qua-giai-bt-hinh-hoc
Tailieu.vncty.com boi-duong-tu-duy-sang-tao-qua-giai-bt-hinh-hoc
 
đề Tài ứng dụng công nghệ thông tin trong giảng dạy phân môn thường thức mĩ ...
đề Tài ứng dụng công nghệ thông tin trong giảng dạy phân môn thường thức mĩ ...đề Tài ứng dụng công nghệ thông tin trong giảng dạy phân môn thường thức mĩ ...
đề Tài ứng dụng công nghệ thông tin trong giảng dạy phân môn thường thức mĩ ...
 
T005
T005T005
T005
 
Tiểu Luận Tiềm Năng Và Thực Trạng Phát Triển Du Lịch Văn Hoá Ở Hà Nội
Tiểu Luận Tiềm Năng Và Thực Trạng Phát Triển Du Lịch Văn Hoá Ở Hà Nội Tiểu Luận Tiềm Năng Và Thực Trạng Phát Triển Du Lịch Văn Hoá Ở Hà Nội
Tiểu Luận Tiềm Năng Và Thực Trạng Phát Triển Du Lịch Văn Hoá Ở Hà Nội
 
Lien bp su dung do dung khi day tre ke chuyen trong hd lqvh
Lien   bp su dung do dung khi day tre ke chuyen trong hd lqvhLien   bp su dung do dung khi day tre ke chuyen trong hd lqvh
Lien bp su dung do dung khi day tre ke chuyen trong hd lqvh
 
Bai 15 quyen va nghia vu hoc tap
Bai 15  quyen va nghia vu hoc tapBai 15  quyen va nghia vu hoc tap
Bai 15 quyen va nghia vu hoc tap
 
Luan van ve hop dong chuyen nhuong quyen su dung dat
Luan van ve hop dong chuyen nhuong quyen su dung datLuan van ve hop dong chuyen nhuong quyen su dung dat
Luan van ve hop dong chuyen nhuong quyen su dung dat
 
Nguyenly ketoan
Nguyenly ketoanNguyenly ketoan
Nguyenly ketoan
 

Similar to Boi duong hsg

Tailieu.vncty.com boi-duong-tu-duy-sang-tao-qua-giai-bt-hinh-hoc
Tailieu.vncty.com   boi-duong-tu-duy-sang-tao-qua-giai-bt-hinh-hocTailieu.vncty.com   boi-duong-tu-duy-sang-tao-qua-giai-bt-hinh-hoc
Tailieu.vncty.com boi-duong-tu-duy-sang-tao-qua-giai-bt-hinh-hocTrần Đức Anh
 
Tailieu.vncty.com phat-trien-tu-duy-ham-qua-day-phuong-trinh
Tailieu.vncty.com   phat-trien-tu-duy-ham-qua-day-phuong-trinhTailieu.vncty.com   phat-trien-tu-duy-ham-qua-day-phuong-trinh
Tailieu.vncty.com phat-trien-tu-duy-ham-qua-day-phuong-trinhTrần Đức Anh
 
Sáng kiến kinh nghiệm : Hướng dẫn học sinh lớp 8 giải các bài toán cực trị tr...
Sáng kiến kinh nghiệm : Hướng dẫn học sinh lớp 8 giải các bài toán cực trị tr...Sáng kiến kinh nghiệm : Hướng dẫn học sinh lớp 8 giải các bài toán cực trị tr...
Sáng kiến kinh nghiệm : Hướng dẫn học sinh lớp 8 giải các bài toán cực trị tr...Học Tập Long An
 
Tailieu.vncty.com phat-trien-tu-duy-thuat-giai-qua-day-pt
Tailieu.vncty.com   phat-trien-tu-duy-thuat-giai-qua-day-ptTailieu.vncty.com   phat-trien-tu-duy-thuat-giai-qua-day-pt
Tailieu.vncty.com phat-trien-tu-duy-thuat-giai-qua-day-ptTrần Đức Anh
 
Tailieu.vncty.com phan tich bien dong dan so, lao dong va viec lam ở huyen ...
Tailieu.vncty.com   phan tich bien dong dan so, lao dong va viec lam ở huyen ...Tailieu.vncty.com   phan tich bien dong dan so, lao dong va viec lam ở huyen ...
Tailieu.vncty.com phan tich bien dong dan so, lao dong va viec lam ở huyen ...Trần Đức Anh
 
Con người và nhân tố con người trong sự phát triển nền KT tri thức
Con người và nhân tố con người trong sự phát triển nền KT tri thứcCon người và nhân tố con người trong sự phát triển nền KT tri thức
Con người và nhân tố con người trong sự phát triển nền KT tri thứcnataliej4
 
ẢNh hưởng của yếu tố sinh học và yếu tố xã hội đến sức khỏe khỏe con người vi...
ẢNh hưởng của yếu tố sinh học và yếu tố xã hội đến sức khỏe khỏe con người vi...ẢNh hưởng của yếu tố sinh học và yếu tố xã hội đến sức khỏe khỏe con người vi...
ẢNh hưởng của yếu tố sinh học và yếu tố xã hội đến sức khỏe khỏe con người vi...Man_Ebook
 
Điều dưỡng cơ bản 1.pdf
Điều dưỡng cơ bản 1.pdfĐiều dưỡng cơ bản 1.pdf
Điều dưỡng cơ bản 1.pdfMan_Ebook
 
09 cndd dieuduongcb1
09 cndd dieuduongcb109 cndd dieuduongcb1
09 cndd dieuduongcb1TS DUOC
 
Lý luận sở hữu của mác trong nền kinh tế thị trường theo định hướng xã hội ch...
Lý luận sở hữu của mác trong nền kinh tế thị trường theo định hướng xã hội ch...Lý luận sở hữu của mác trong nền kinh tế thị trường theo định hướng xã hội ch...
Lý luận sở hữu của mác trong nền kinh tế thị trường theo định hướng xã hội ch...nataliej4
 
Giáo trình dạy học hiện đại
Giáo trình dạy học hiện đạiGiáo trình dạy học hiện đại
Giáo trình dạy học hiện đạijackjohn45
 
Luật số 38/2005/QH11 của Quốc hội : Luật Giáo dục
Luật số 38/2005/QH11 của Quốc hội : Luật Giáo dụcLuật số 38/2005/QH11 của Quốc hội : Luật Giáo dục
Luật số 38/2005/QH11 của Quốc hội : Luật Giáo dụcThcsphanboichautp
 
đề 3 bookbooming
đề 3 bookboomingđề 3 bookbooming
đề 3 bookboomingbookbooming
 
Sức khỏe môi trường
Sức khỏe môi trườngSức khỏe môi trường
Sức khỏe môi trườngTS DUOC
 
Tài liệu sức khỏe môi trường - Bộ Y Tế
Tài liệu sức khỏe môi trường - Bộ Y TếTài liệu sức khỏe môi trường - Bộ Y Tế
Tài liệu sức khỏe môi trường - Bộ Y TếĐiều Dưỡng
 

Similar to Boi duong hsg (20)

Tailieu.vncty.com boi-duong-tu-duy-sang-tao-qua-giai-bt-hinh-hoc
Tailieu.vncty.com   boi-duong-tu-duy-sang-tao-qua-giai-bt-hinh-hocTailieu.vncty.com   boi-duong-tu-duy-sang-tao-qua-giai-bt-hinh-hoc
Tailieu.vncty.com boi-duong-tu-duy-sang-tao-qua-giai-bt-hinh-hoc
 
H oa hong
H oa hongH oa hong
H oa hong
 
Tailieu.vncty.com phat-trien-tu-duy-ham-qua-day-phuong-trinh
Tailieu.vncty.com   phat-trien-tu-duy-ham-qua-day-phuong-trinhTailieu.vncty.com   phat-trien-tu-duy-ham-qua-day-phuong-trinh
Tailieu.vncty.com phat-trien-tu-duy-ham-qua-day-phuong-trinh
 
Sáng kiến kinh nghiệm : Hướng dẫn học sinh lớp 8 giải các bài toán cực trị tr...
Sáng kiến kinh nghiệm : Hướng dẫn học sinh lớp 8 giải các bài toán cực trị tr...Sáng kiến kinh nghiệm : Hướng dẫn học sinh lớp 8 giải các bài toán cực trị tr...
Sáng kiến kinh nghiệm : Hướng dẫn học sinh lớp 8 giải các bài toán cực trị tr...
 
Tailieu.vncty.com phat-trien-tu-duy-thuat-giai-qua-day-pt
Tailieu.vncty.com   phat-trien-tu-duy-thuat-giai-qua-day-ptTailieu.vncty.com   phat-trien-tu-duy-thuat-giai-qua-day-pt
Tailieu.vncty.com phat-trien-tu-duy-thuat-giai-qua-day-pt
 
Tailieu.vncty.com phan tich bien dong dan so, lao dong va viec lam ở huyen ...
Tailieu.vncty.com   phan tich bien dong dan so, lao dong va viec lam ở huyen ...Tailieu.vncty.com   phan tich bien dong dan so, lao dong va viec lam ở huyen ...
Tailieu.vncty.com phan tich bien dong dan so, lao dong va viec lam ở huyen ...
 
Bao cao tong ket to cm
Bao cao tong ket to cmBao cao tong ket to cm
Bao cao tong ket to cm
 
Vai trò của triết học trong đời sống xã hội (TẢI FREE ZALO 0934 573 149)
Vai trò của triết học trong đời sống xã hội (TẢI FREE ZALO 0934 573 149)Vai trò của triết học trong đời sống xã hội (TẢI FREE ZALO 0934 573 149)
Vai trò của triết học trong đời sống xã hội (TẢI FREE ZALO 0934 573 149)
 
Con người và nhân tố con người trong sự phát triển nền KT tri thức
Con người và nhân tố con người trong sự phát triển nền KT tri thứcCon người và nhân tố con người trong sự phát triển nền KT tri thức
Con người và nhân tố con người trong sự phát triển nền KT tri thức
 
ẢNh hưởng của yếu tố sinh học và yếu tố xã hội đến sức khỏe khỏe con người vi...
ẢNh hưởng của yếu tố sinh học và yếu tố xã hội đến sức khỏe khỏe con người vi...ẢNh hưởng của yếu tố sinh học và yếu tố xã hội đến sức khỏe khỏe con người vi...
ẢNh hưởng của yếu tố sinh học và yếu tố xã hội đến sức khỏe khỏe con người vi...
 
Điều dưỡng cơ bản 1.pdf
Điều dưỡng cơ bản 1.pdfĐiều dưỡng cơ bản 1.pdf
Điều dưỡng cơ bản 1.pdf
 
09 cndd dieuduongcb1
09 cndd dieuduongcb109 cndd dieuduongcb1
09 cndd dieuduongcb1
 
Lý luận sở hữu của mác trong nền kinh tế thị trường theo định hướng xã hội ch...
Lý luận sở hữu của mác trong nền kinh tế thị trường theo định hướng xã hội ch...Lý luận sở hữu của mác trong nền kinh tế thị trường theo định hướng xã hội ch...
Lý luận sở hữu của mác trong nền kinh tế thị trường theo định hướng xã hội ch...
 
Giáo trình dạy học hiện đại
Giáo trình dạy học hiện đạiGiáo trình dạy học hiện đại
Giáo trình dạy học hiện đại
 
Giới thiệu
Giới thiệuGiới thiệu
Giới thiệu
 
Luật số 38/2005/QH11 của Quốc hội : Luật Giáo dục
Luật số 38/2005/QH11 của Quốc hội : Luật Giáo dụcLuật số 38/2005/QH11 của Quốc hội : Luật Giáo dục
Luật số 38/2005/QH11 của Quốc hội : Luật Giáo dục
 
Qt043
Qt043Qt043
Qt043
 
đề 3 bookbooming
đề 3 bookboomingđề 3 bookbooming
đề 3 bookbooming
 
Sức khỏe môi trường
Sức khỏe môi trườngSức khỏe môi trường
Sức khỏe môi trường
 
Tài liệu sức khỏe môi trường - Bộ Y Tế
Tài liệu sức khỏe môi trường - Bộ Y TếTài liệu sức khỏe môi trường - Bộ Y Tế
Tài liệu sức khỏe môi trường - Bộ Y Tế
 

Recently uploaded

Trích dẫn trắc nghiệm tư tưởng HCM5.docx
Trích dẫn trắc nghiệm tư tưởng HCM5.docxTrích dẫn trắc nghiệm tư tưởng HCM5.docx
Trích dẫn trắc nghiệm tư tưởng HCM5.docxnhungdt08102004
 
BỘ ĐỀ PHÁT TRIỂN THEO CẤU TRÚC ĐỀ MINH HỌA BGD NGÀY 22-3-2024 KỲ THI TỐT NGHI...
BỘ ĐỀ PHÁT TRIỂN THEO CẤU TRÚC ĐỀ MINH HỌA BGD NGÀY 22-3-2024 KỲ THI TỐT NGHI...BỘ ĐỀ PHÁT TRIỂN THEO CẤU TRÚC ĐỀ MINH HỌA BGD NGÀY 22-3-2024 KỲ THI TỐT NGHI...
BỘ ĐỀ PHÁT TRIỂN THEO CẤU TRÚC ĐỀ MINH HỌA BGD NGÀY 22-3-2024 KỲ THI TỐT NGHI...Nguyen Thanh Tu Collection
 
Kiểm tra chạy trạm lí thuyết giữa kì giải phẫu sinh lí
Kiểm tra chạy trạm lí thuyết giữa kì giải phẫu sinh líKiểm tra chạy trạm lí thuyết giữa kì giải phẫu sinh lí
Kiểm tra chạy trạm lí thuyết giữa kì giải phẫu sinh líDr K-OGN
 
GIÁO TRÌNH KHỐI NGUỒN CÁC LOẠI - ĐIỆN LẠNH BÁCH KHOA HÀ NỘI
GIÁO TRÌNH  KHỐI NGUỒN CÁC LOẠI - ĐIỆN LẠNH BÁCH KHOA HÀ NỘIGIÁO TRÌNH  KHỐI NGUỒN CÁC LOẠI - ĐIỆN LẠNH BÁCH KHOA HÀ NỘI
GIÁO TRÌNH KHỐI NGUỒN CÁC LOẠI - ĐIỆN LẠNH BÁCH KHOA HÀ NỘIĐiện Lạnh Bách Khoa Hà Nội
 
Chàm - Bệnh án (da liễu - bvdlct ctump) .pptx
Chàm - Bệnh án (da liễu - bvdlct ctump) .pptxChàm - Bệnh án (da liễu - bvdlct ctump) .pptx
Chàm - Bệnh án (da liễu - bvdlct ctump) .pptxendkay31
 
30 ĐỀ PHÁT TRIỂN THEO CẤU TRÚC ĐỀ MINH HỌA BGD NGÀY 22-3-2024 KỲ THI TỐT NGHI...
30 ĐỀ PHÁT TRIỂN THEO CẤU TRÚC ĐỀ MINH HỌA BGD NGÀY 22-3-2024 KỲ THI TỐT NGHI...30 ĐỀ PHÁT TRIỂN THEO CẤU TRÚC ĐỀ MINH HỌA BGD NGÀY 22-3-2024 KỲ THI TỐT NGHI...
30 ĐỀ PHÁT TRIỂN THEO CẤU TRÚC ĐỀ MINH HỌA BGD NGÀY 22-3-2024 KỲ THI TỐT NGHI...Nguyen Thanh Tu Collection
 
chuong-7-van-de-gia-dinh-trong-thoi-ky-qua-do-len-cnxh.pdf
chuong-7-van-de-gia-dinh-trong-thoi-ky-qua-do-len-cnxh.pdfchuong-7-van-de-gia-dinh-trong-thoi-ky-qua-do-len-cnxh.pdf
chuong-7-van-de-gia-dinh-trong-thoi-ky-qua-do-len-cnxh.pdfVyTng986513
 
sách sinh học đại cương - Textbook.pdf
sách sinh học đại cương   -   Textbook.pdfsách sinh học đại cương   -   Textbook.pdf
sách sinh học đại cương - Textbook.pdfTrnHoa46
 
30 ĐỀ PHÁT TRIỂN THEO CẤU TRÚC ĐỀ MINH HỌA BGD NGÀY 22-3-2024 KỲ THI TỐT NGHI...
30 ĐỀ PHÁT TRIỂN THEO CẤU TRÚC ĐỀ MINH HỌA BGD NGÀY 22-3-2024 KỲ THI TỐT NGHI...30 ĐỀ PHÁT TRIỂN THEO CẤU TRÚC ĐỀ MINH HỌA BGD NGÀY 22-3-2024 KỲ THI TỐT NGHI...
30 ĐỀ PHÁT TRIỂN THEO CẤU TRÚC ĐỀ MINH HỌA BGD NGÀY 22-3-2024 KỲ THI TỐT NGHI...Nguyen Thanh Tu Collection
 
QUẢN LÝ HOẠT ĐỘNG GIÁO DỤC KỸ NĂNG SỐNG CHO HỌC SINH CÁC TRƯỜNG TRUNG HỌC CƠ ...
QUẢN LÝ HOẠT ĐỘNG GIÁO DỤC KỸ NĂNG SỐNG CHO HỌC SINH CÁC TRƯỜNG TRUNG HỌC CƠ ...QUẢN LÝ HOẠT ĐỘNG GIÁO DỤC KỸ NĂNG SỐNG CHO HỌC SINH CÁC TRƯỜNG TRUNG HỌC CƠ ...
QUẢN LÝ HOẠT ĐỘNG GIÁO DỤC KỸ NĂNG SỐNG CHO HỌC SINH CÁC TRƯỜNG TRUNG HỌC CƠ ...ThunTrn734461
 
Thong bao 337-DHPY (24.4.2024) thi sat hach Ngoai ngu dap ung Chuan dau ra do...
Thong bao 337-DHPY (24.4.2024) thi sat hach Ngoai ngu dap ung Chuan dau ra do...Thong bao 337-DHPY (24.4.2024) thi sat hach Ngoai ngu dap ung Chuan dau ra do...
Thong bao 337-DHPY (24.4.2024) thi sat hach Ngoai ngu dap ung Chuan dau ra do...hoangtuansinh1
 
Chuong trinh dao tao Su pham Khoa hoc tu nhien, ma nganh - 7140247.pdf
Chuong trinh dao tao Su pham Khoa hoc tu nhien, ma nganh - 7140247.pdfChuong trinh dao tao Su pham Khoa hoc tu nhien, ma nganh - 7140247.pdf
Chuong trinh dao tao Su pham Khoa hoc tu nhien, ma nganh - 7140247.pdfhoangtuansinh1
 
Sáng kiến Dạy học theo định hướng STEM một số chủ đề phần “vật sống”, Khoa họ...
Sáng kiến Dạy học theo định hướng STEM một số chủ đề phần “vật sống”, Khoa họ...Sáng kiến Dạy học theo định hướng STEM một số chủ đề phần “vật sống”, Khoa họ...
Sáng kiến Dạy học theo định hướng STEM một số chủ đề phần “vật sống”, Khoa họ...Nguyen Thanh Tu Collection
 
ôn tập lịch sử hhhhhhhhhhhhhhhhhhhhhhhhhh
ôn tập lịch sử hhhhhhhhhhhhhhhhhhhhhhhhhhôn tập lịch sử hhhhhhhhhhhhhhhhhhhhhhhhhh
ôn tập lịch sử hhhhhhhhhhhhhhhhhhhhhhhhhhvanhathvc
 
Sáng kiến “Sử dụng ứng dụng Quizizz nhằm nâng cao chất lượng ôn thi tốt nghiệ...
Sáng kiến “Sử dụng ứng dụng Quizizz nhằm nâng cao chất lượng ôn thi tốt nghiệ...Sáng kiến “Sử dụng ứng dụng Quizizz nhằm nâng cao chất lượng ôn thi tốt nghiệ...
Sáng kiến “Sử dụng ứng dụng Quizizz nhằm nâng cao chất lượng ôn thi tốt nghiệ...Nguyen Thanh Tu Collection
 
30 ĐỀ PHÁT TRIỂN THEO CẤU TRÚC ĐỀ MINH HỌA BGD NGÀY 22-3-2024 KỲ THI TỐT NGHI...
30 ĐỀ PHÁT TRIỂN THEO CẤU TRÚC ĐỀ MINH HỌA BGD NGÀY 22-3-2024 KỲ THI TỐT NGHI...30 ĐỀ PHÁT TRIỂN THEO CẤU TRÚC ĐỀ MINH HỌA BGD NGÀY 22-3-2024 KỲ THI TỐT NGHI...
30 ĐỀ PHÁT TRIỂN THEO CẤU TRÚC ĐỀ MINH HỌA BGD NGÀY 22-3-2024 KỲ THI TỐT NGHI...Nguyen Thanh Tu Collection
 
30 ĐỀ PHÁT TRIỂN THEO CẤU TRÚC ĐỀ MINH HỌA BGD NGÀY 22-3-2024 KỲ THI TỐT NGHI...
30 ĐỀ PHÁT TRIỂN THEO CẤU TRÚC ĐỀ MINH HỌA BGD NGÀY 22-3-2024 KỲ THI TỐT NGHI...30 ĐỀ PHÁT TRIỂN THEO CẤU TRÚC ĐỀ MINH HỌA BGD NGÀY 22-3-2024 KỲ THI TỐT NGHI...
30 ĐỀ PHÁT TRIỂN THEO CẤU TRÚC ĐỀ MINH HỌA BGD NGÀY 22-3-2024 KỲ THI TỐT NGHI...Nguyen Thanh Tu Collection
 
Sơ đồ tư duy môn sinh học bậc THPT.pdf
Sơ đồ tư duy môn sinh học bậc THPT.pdfSơ đồ tư duy môn sinh học bậc THPT.pdf
Sơ đồ tư duy môn sinh học bậc THPT.pdftohoanggiabao81
 
Đề cương môn giải phẫu......................
Đề cương môn giải phẫu......................Đề cương môn giải phẫu......................
Đề cương môn giải phẫu......................TrnHoa46
 

Recently uploaded (20)

Trích dẫn trắc nghiệm tư tưởng HCM5.docx
Trích dẫn trắc nghiệm tư tưởng HCM5.docxTrích dẫn trắc nghiệm tư tưởng HCM5.docx
Trích dẫn trắc nghiệm tư tưởng HCM5.docx
 
BỘ ĐỀ PHÁT TRIỂN THEO CẤU TRÚC ĐỀ MINH HỌA BGD NGÀY 22-3-2024 KỲ THI TỐT NGHI...
BỘ ĐỀ PHÁT TRIỂN THEO CẤU TRÚC ĐỀ MINH HỌA BGD NGÀY 22-3-2024 KỲ THI TỐT NGHI...BỘ ĐỀ PHÁT TRIỂN THEO CẤU TRÚC ĐỀ MINH HỌA BGD NGÀY 22-3-2024 KỲ THI TỐT NGHI...
BỘ ĐỀ PHÁT TRIỂN THEO CẤU TRÚC ĐỀ MINH HỌA BGD NGÀY 22-3-2024 KỲ THI TỐT NGHI...
 
Kiểm tra chạy trạm lí thuyết giữa kì giải phẫu sinh lí
Kiểm tra chạy trạm lí thuyết giữa kì giải phẫu sinh líKiểm tra chạy trạm lí thuyết giữa kì giải phẫu sinh lí
Kiểm tra chạy trạm lí thuyết giữa kì giải phẫu sinh lí
 
GIÁO TRÌNH KHỐI NGUỒN CÁC LOẠI - ĐIỆN LẠNH BÁCH KHOA HÀ NỘI
GIÁO TRÌNH  KHỐI NGUỒN CÁC LOẠI - ĐIỆN LẠNH BÁCH KHOA HÀ NỘIGIÁO TRÌNH  KHỐI NGUỒN CÁC LOẠI - ĐIỆN LẠNH BÁCH KHOA HÀ NỘI
GIÁO TRÌNH KHỐI NGUỒN CÁC LOẠI - ĐIỆN LẠNH BÁCH KHOA HÀ NỘI
 
Chàm - Bệnh án (da liễu - bvdlct ctump) .pptx
Chàm - Bệnh án (da liễu - bvdlct ctump) .pptxChàm - Bệnh án (da liễu - bvdlct ctump) .pptx
Chàm - Bệnh án (da liễu - bvdlct ctump) .pptx
 
30 ĐỀ PHÁT TRIỂN THEO CẤU TRÚC ĐỀ MINH HỌA BGD NGÀY 22-3-2024 KỲ THI TỐT NGHI...
30 ĐỀ PHÁT TRIỂN THEO CẤU TRÚC ĐỀ MINH HỌA BGD NGÀY 22-3-2024 KỲ THI TỐT NGHI...30 ĐỀ PHÁT TRIỂN THEO CẤU TRÚC ĐỀ MINH HỌA BGD NGÀY 22-3-2024 KỲ THI TỐT NGHI...
30 ĐỀ PHÁT TRIỂN THEO CẤU TRÚC ĐỀ MINH HỌA BGD NGÀY 22-3-2024 KỲ THI TỐT NGHI...
 
chuong-7-van-de-gia-dinh-trong-thoi-ky-qua-do-len-cnxh.pdf
chuong-7-van-de-gia-dinh-trong-thoi-ky-qua-do-len-cnxh.pdfchuong-7-van-de-gia-dinh-trong-thoi-ky-qua-do-len-cnxh.pdf
chuong-7-van-de-gia-dinh-trong-thoi-ky-qua-do-len-cnxh.pdf
 
sách sinh học đại cương - Textbook.pdf
sách sinh học đại cương   -   Textbook.pdfsách sinh học đại cương   -   Textbook.pdf
sách sinh học đại cương - Textbook.pdf
 
30 ĐỀ PHÁT TRIỂN THEO CẤU TRÚC ĐỀ MINH HỌA BGD NGÀY 22-3-2024 KỲ THI TỐT NGHI...
30 ĐỀ PHÁT TRIỂN THEO CẤU TRÚC ĐỀ MINH HỌA BGD NGÀY 22-3-2024 KỲ THI TỐT NGHI...30 ĐỀ PHÁT TRIỂN THEO CẤU TRÚC ĐỀ MINH HỌA BGD NGÀY 22-3-2024 KỲ THI TỐT NGHI...
30 ĐỀ PHÁT TRIỂN THEO CẤU TRÚC ĐỀ MINH HỌA BGD NGÀY 22-3-2024 KỲ THI TỐT NGHI...
 
QUẢN LÝ HOẠT ĐỘNG GIÁO DỤC KỸ NĂNG SỐNG CHO HỌC SINH CÁC TRƯỜNG TRUNG HỌC CƠ ...
QUẢN LÝ HOẠT ĐỘNG GIÁO DỤC KỸ NĂNG SỐNG CHO HỌC SINH CÁC TRƯỜNG TRUNG HỌC CƠ ...QUẢN LÝ HOẠT ĐỘNG GIÁO DỤC KỸ NĂNG SỐNG CHO HỌC SINH CÁC TRƯỜNG TRUNG HỌC CƠ ...
QUẢN LÝ HOẠT ĐỘNG GIÁO DỤC KỸ NĂNG SỐNG CHO HỌC SINH CÁC TRƯỜNG TRUNG HỌC CƠ ...
 
Thong bao 337-DHPY (24.4.2024) thi sat hach Ngoai ngu dap ung Chuan dau ra do...
Thong bao 337-DHPY (24.4.2024) thi sat hach Ngoai ngu dap ung Chuan dau ra do...Thong bao 337-DHPY (24.4.2024) thi sat hach Ngoai ngu dap ung Chuan dau ra do...
Thong bao 337-DHPY (24.4.2024) thi sat hach Ngoai ngu dap ung Chuan dau ra do...
 
1 - MÃ LỖI SỬA CHỮA BOARD MẠCH BẾP TỪ.pdf
1 - MÃ LỖI SỬA CHỮA BOARD MẠCH BẾP TỪ.pdf1 - MÃ LỖI SỬA CHỮA BOARD MẠCH BẾP TỪ.pdf
1 - MÃ LỖI SỬA CHỮA BOARD MẠCH BẾP TỪ.pdf
 
Chuong trinh dao tao Su pham Khoa hoc tu nhien, ma nganh - 7140247.pdf
Chuong trinh dao tao Su pham Khoa hoc tu nhien, ma nganh - 7140247.pdfChuong trinh dao tao Su pham Khoa hoc tu nhien, ma nganh - 7140247.pdf
Chuong trinh dao tao Su pham Khoa hoc tu nhien, ma nganh - 7140247.pdf
 
Sáng kiến Dạy học theo định hướng STEM một số chủ đề phần “vật sống”, Khoa họ...
Sáng kiến Dạy học theo định hướng STEM một số chủ đề phần “vật sống”, Khoa họ...Sáng kiến Dạy học theo định hướng STEM một số chủ đề phần “vật sống”, Khoa họ...
Sáng kiến Dạy học theo định hướng STEM một số chủ đề phần “vật sống”, Khoa họ...
 
ôn tập lịch sử hhhhhhhhhhhhhhhhhhhhhhhhhh
ôn tập lịch sử hhhhhhhhhhhhhhhhhhhhhhhhhhôn tập lịch sử hhhhhhhhhhhhhhhhhhhhhhhhhh
ôn tập lịch sử hhhhhhhhhhhhhhhhhhhhhhhhhh
 
Sáng kiến “Sử dụng ứng dụng Quizizz nhằm nâng cao chất lượng ôn thi tốt nghiệ...
Sáng kiến “Sử dụng ứng dụng Quizizz nhằm nâng cao chất lượng ôn thi tốt nghiệ...Sáng kiến “Sử dụng ứng dụng Quizizz nhằm nâng cao chất lượng ôn thi tốt nghiệ...
Sáng kiến “Sử dụng ứng dụng Quizizz nhằm nâng cao chất lượng ôn thi tốt nghiệ...
 
30 ĐỀ PHÁT TRIỂN THEO CẤU TRÚC ĐỀ MINH HỌA BGD NGÀY 22-3-2024 KỲ THI TỐT NGHI...
30 ĐỀ PHÁT TRIỂN THEO CẤU TRÚC ĐỀ MINH HỌA BGD NGÀY 22-3-2024 KỲ THI TỐT NGHI...30 ĐỀ PHÁT TRIỂN THEO CẤU TRÚC ĐỀ MINH HỌA BGD NGÀY 22-3-2024 KỲ THI TỐT NGHI...
30 ĐỀ PHÁT TRIỂN THEO CẤU TRÚC ĐỀ MINH HỌA BGD NGÀY 22-3-2024 KỲ THI TỐT NGHI...
 
30 ĐỀ PHÁT TRIỂN THEO CẤU TRÚC ĐỀ MINH HỌA BGD NGÀY 22-3-2024 KỲ THI TỐT NGHI...
30 ĐỀ PHÁT TRIỂN THEO CẤU TRÚC ĐỀ MINH HỌA BGD NGÀY 22-3-2024 KỲ THI TỐT NGHI...30 ĐỀ PHÁT TRIỂN THEO CẤU TRÚC ĐỀ MINH HỌA BGD NGÀY 22-3-2024 KỲ THI TỐT NGHI...
30 ĐỀ PHÁT TRIỂN THEO CẤU TRÚC ĐỀ MINH HỌA BGD NGÀY 22-3-2024 KỲ THI TỐT NGHI...
 
Sơ đồ tư duy môn sinh học bậc THPT.pdf
Sơ đồ tư duy môn sinh học bậc THPT.pdfSơ đồ tư duy môn sinh học bậc THPT.pdf
Sơ đồ tư duy môn sinh học bậc THPT.pdf
 
Đề cương môn giải phẫu......................
Đề cương môn giải phẫu......................Đề cương môn giải phẫu......................
Đề cương môn giải phẫu......................
 

Boi duong hsg

  • 1. Chuyªn ®Ò BDHSG To¸n THCS §æi míi ph¬ng ph¸p d¹y häc to¸n theo híng tÝch cùc ho¸ ho¹t ®éng häc tËp cña häc sinh A. Mét vµi nÐt vÒ lý luËn d¹y häc 1.T tëng tÝch cùc ho¸ ho¹t ®éng häc tËp cña häc sinh X· héi ph¸t triÓn vµ sù ®æi míi ®Êt níc ®ßi hái ph¶i n©ng cao chÊt lîng gi¸o dôc nh»m ®µo t¹o nh÷ng con ngêi lao ®éng ®¶m b¶o môc tiªu hiÖn ®¹i ho¸ ®Êt n- íc. + Theo Kharlamop.I.F. “Häc tËp lµ mét qu¸ tr×nh nhËn thøc tÝch cùc” Theo tõ ®iÓn TiÕng ViÖt: TÝch cùc lµ mét tr¹ng th¸i tinh thÇn cã t¸c dông kh¼ng ®Þnh vµ thóc ®Èy sù ph¸t triÓn . Trong ho¹t ®éng häc tËp nã diÔn ra ë nhiÒu ph¬ng diÖn kh¸c nhau: Tri gi¸c tµi liÖu, th«ng hiÓu tµi liÖu, ghi nhí, luyÖn tËp, vËn dông, kh¸i qu¸t... vµ ®îc thÓ hiÖn ë nhiÒu h×nh thøc ®a d¹ng, phong phó. §éng c¬ häc tËp lµ nguån t¹o ra tÝnh tÝch cùc trong ho¹t ®éng häc vµ khi ®· h×nh thµnh l¹i cã gi¸ trÞ nh mét ®éng c¬ thóc giôc ho¹t ®éng, lµ thuéc tÝnh cña nh©n c¸ch, cßn tÝnh tÝch cùc l¹i lµ mét tr¹ng th¸i tinh thÇn lµm nÒn cho ho¹t ®éng diÔn ra cã hiÖu qu¶ vµ cã thuéc tÝnh thiªn vÒ c¶m xóc . G.I. Sukina ®· chia tÝnh tÝch cùc ra lµm ba cÊp ®é 1. TÝnh tÝch cùc b¾t chíc, t¸i hiÖn: xuÊt hiÖn do t¸c ®éng kÝch thÝch bªn ngoµi. Trong trêng hîp nµy ngêi häc thao t¸c trªn ®èi tîng, b¾t chíc theo mÉu hoÆc m« h×nh cña GV, nh»m chuyÓn ®èi tîng tõ ngoµi vµo trong theo c¬ chÕ “ho¹t ®éng bªn ngoµi bªn trong cã cïng cÊu tróc”. Nhê ®ã, kinh nghiÖm ho¹t ®éng ®îc tÝch luü th«ng qua kinh nghiÖm ngêi kh¸c. 2. TÝnh tÝch cùc t×m tßi: ®i liÒn víi qu¸ tr×nh h×nh thµnh kh¸i niÖm. Gi¶i quyÕt c¸c t×nh huèng nhËn thøc, t×m ra c¸c ph¬ng thøc hµnh ®éng trªn c¬ së cã tÝnh tù gi¸c, cã sù tham gia cña ®éng c¬, nhu cÇu, høng thó vµ ý chÝ cña HS. Lo¹i nµy xuÊt hiÖn kh«ng chØ do yªu cÇu cña GV mµ cßn hoµn toµn tù ph¸t trong qu¸ tr×nh nhËn thøc. Nã tån t¹i kh«ng chØ ë d¹ng tr¹ng th¸i, c¶m xóc mµ cßn ë d¹ng thuéc tÝnh bÒn v÷ng cña ho¹t ®éng. ë møc ®é nµy tÝnh ®éc lËp cao h¬n møc trªn, cho phÐp HS tiÕp nhËn nhiÖm vô vµ tù m×nh t×m ra ph¬ng tiÖn thùc hiÖn. 1
  • 2. Chuyªn ®Ò BDHSG To¸n THCS 3. TÝnh tÝch cùc s¸ng t¹o: thÓ hiÖn khi chñ thÓ nhËn thøc tù t×m tßi kiÕn thøc míi, tù t×m kiÕm ra ph¬ng thøc hµnh ®éng riªng vµ trë thµnh phÈm chÊt bÒn v÷ng cña c¸ nh©n. §©y lµ møc ®é biÓu hiÖn tÝnh tÝch cùc nhËn thøc cao nhÊt . Nh vËy nãi vÒ tÝnh tÝch cùc nhËn thøc, ngêi ta thêng ®¸nh gi¸ vÒ møc ®é nhËn thøc cña ngêi häc trong qu¸ tr×nh thùc hiÖn môc ®Ých d¹y häc. Kharlamop I.F. viÕt: “TÝnh tÝch cùc trong ho¹t ®éng nhËn thøc lµ tr¹ng th¸i ho¹t ®éng cña HS, ®îc ®Æc trng bëi kh¸t väng häc tËp, sù cè g¾ng trÝ tuÖ víi nghÞ lùc cao trong qu¸ tr×nh n¾m v÷ng kiÕn thøc cho chÝnh m×nh”. + TÝch cùc ho¸ ho¹t ®éng häc tËp cña häc sinh: Tèi ®a ho¸ sù tham gia ho¹t ®éng cña ngêi häc víi ®Þnh híng chØ ®¹o lµ tù nhËn thøc, tù ph¸t triÓn, tù thùc hiÖn, tù kiÓm tra vµ ®¸nh gi¸, qua ®ã h×nh thµnh vµ ph¸t triÓn t duy ®éc lËp vµ s¸ng t¹o cña HS. GS.TSKH NguyÔn B¸ Kim chØ râ 4 yªu cÇu ®Ó tÝch cùc ho¸ ho¹t ®éng häc tËp cña HS: - X¸c lËp vÞ trÝ chñ thÓ cña ngêi häc, ®¶m b¶o tÝnh tù gi¸c, tÝch cùc s¸ng t¹o cña ho¹t ®éng häc tËp. - D¹y häc ph¶i dùa trªn nghiªn cøu t¸c ®éng cña nh÷ng quan niÖm vµ kiÕn thøc s½n cã cña ngêi häc, nh»m khai th¸c mÆt thuËn lîi, h¹n chÕ mÆt khã kh¨n, nghiªn cøu nh÷ng chíng ng¹i hoÆc sai lÇm cã thÓ cã cña nh÷ng kiÕn thøc ®ã trong qu¸ tr×nh häc tËp cña HS. - D¹y häc kh«ng chØ nh»m môc ®Ých lµ tri thøc vµ kü n¨ng bé m«n, mµ quan träng h¬n c¶ lµ viÖc häc, d¹y c¸ch häc cho HS. - Qu¸ tr×nh d¹y häc ph¶i bao hµm c¶ viÖc d¹y c¸ch tù häc th«ng qua viÖc ®Ó HS tù ho¹t ®éng nh»m ®¸p øng nhu cÇu cña b¶n th©n vµ cña x· héi . Tãm l¹i: §Ó ph¸t huy ®îc tÝnh tÝch cùc trong ho¹t ®éng häc tËp cña HS cÇn mét qu¸ tr×nh lµm cho ngêi häc trë thµnh chñ thÓ tÝch cùc trong ho¹t ®éng häc tËp cña chÝnh hä. 2. §Þnh híng ®æi míi ph¬ng ph¸p d¹y häc + Ph¬ng ph¸p d¹y häc - Ph¬ng ph¸p d¹y häc lµ con ®êng, lµ c¸ch thøc cña ho¹t ®éng ®Ó ®¹t môc ®Ých d¹y häc. 2
  • 3. Chuyªn ®Ò BDHSG To¸n THCS - Ph¬ng ph¸p d¹y häc lµ c¸ch thøc ho¹t ®éng vµ giao lu cña thÇy g©y nªn nh÷ng ho¹t ®éng vµ giao lu cÇn thiÕt cña trß nh»m ®¹t môc ®Ých d¹y häc . - Ho¹t ®éng cña thÇy g©y nªn ho¹t ®éng cña trß: Ho¹t ®éng cña thÇy lµ t¸c ®éng ®iÒu khiÓn. Tuy nhiªn t¸c ®éng kh«ng chØ gåm ho¹t ®éng mµ cßn cã sù øng xö cña thÇy gi¸o. ThuËt ng÷ “d¹y häc” vèn ®îc dïng ®Ó ph¶n ¸nh ho¹t ®éng cña ngêi d¹y, thÕ nhng ®èi tîng cña ho¹t ®éng d¹y häc lµ HS, HS võa lµ ®èi tîng cña ho¹t ®éng d¹y l¹i võa lµ chñ thÓ cña ho¹t ®éng häc. V× vËy ph¬ng ph¸p d¹y häc võa bao hµm c¸ch d¹y cña thÇy vµ c¸ch häc cña trß. + §æi míi ph¬ng ph¸p d¹y häc NghÞ quyÕt Trung ¬ng II kho¸ VIII cña §¶ng Céng s¶n ViÖt Nam chØ râ “§æi míi m¹nh mÏ ph¬ng ph¸p gi¸o dôc ®µo t¹o, kh¾c phôc lèi truyÒn thô mét chiÒu, rÌn luyÖn thµnh nÕp t duy s¸ng t¹o cña ngêi häc...” §µo t¹o con ngêi n¨ng ®éng s¸ng t¹o cã n¨ng lùc ph¸t hiÖn vÊn ®Ò vµ tù gi¶i quyÕt vÊn ®Ò lµ mét trong nh÷ng nhiÖm vô träng t©m cña ngµnh gi¸o dôc. M©u thuÉn gi÷a yªu cÇu ®µo t¹o con ngêi míi x©y dùng x· héi c«ng nghiÖp ho¸ vµ thùc tr¹ng l¹c hËu cña ph¬ng ph¸p d¹y häc ®· lµm n¶y sinh thóc ®Èy c«ng cuéc ®æi míi ph¬ng ph¸p d¹y häc ë tÊt c¶ c¸c cÊp häc trong ngµnh gi¸o dôc . Ph¬ng ph¸p d¹y häc kh«ng ph¶i lµ b¶n th©n ho¹t ®éng vµ øng xö cña GV ë b×nh diÖn xem xÐt riªng lÎ, cô thÓ mµ theo NguyÔn B¸ Kim: “Ph¬ng ph¸p d¹y häc lµ h×nh ¶nh kh¸i qu¸t ho¸ nh÷ng ho¹t ®éng øng xö nµo ®ã cña GV. H×nh ¶nh nµy thêng ®îc h×nh thµnh do ph¶n ¸nh nh÷ng ho¹t ®éng øng xö thµnh c«ng cña GV trong qu¸ tr×nh d¹y häc vµ ph¶n ¸nh nh÷ng thµnh tùu cña khoa häc gi¸o dôc hoÆc nh÷ng khoa häc kh¸c th«ng qua khoa häc gi¸o dôc... Ph¬ng ph¸p d¹y häc lµ ph¬ng tiÖn ®Ó ®¹t môc ®Ých d¹y häc” . Trong nh÷ng n¨m gÇn ®©y t tëng d¹y häc chñ ®¹o ®îc ph¸t biÓu díi nhiÒu h×nh thøc kh¸c nhau nh “lÊy HS lµm trung t©m”,“ph¸t huy tÝnh tÝch cùc”, “ph¬ng ph¸p d¹y häc tÝch cùc”, “tÝch cùc ho¸ ho¹t ®éng häc tËp”, “Ho¹t ®éng ho¸ ngêi häc” .... §Þnh híng ®æi míi ph¬ng ph¸p d¹y häc hiÖn nay lµ tæ chøc cho häc sinh häc tËp trong ho¹t ®éng vµ b»ng ho¹t ®éng tù gi¸c, tÝch cùc vµ s¸ng t¹o. + Lµm thÕ nµo ®Ó tÝch cùc ho¸ ho¹t ®éng häc tËp cña häc sinh ? 3
  • 4. Chuyªn ®Ò BDHSG To¸n THCS Nhµ t©m lÝ häc I.X.Iakimanxkai cho r»ng: Nhµ trêng cÇn trang bÞ cho HS hai hÖ thèng tri thøc: 1. VÒ hiÖn thùc ®èi tîng; 2. VÒ néi dung c¸ch thøc thùc hiÖn c¸c hµnh ®éng trÝ tuÖ, ®¶m b¶o viÖc n¾m v÷ng c¸c tri thøc khoa häc vÒ hiÖn thùc ®èi tîng ®ã. C¸c tri thøc lo¹i mét ®îc ph¶n ¸nh trong SGK, cßn c¸c tri thøc lo¹i hai ®îc h×nh thµnh chñ yÕu ë HS b»ng con ®êng tù ph¸t. ë ®ã tri thøc lo¹i hai lµ c¸c thñ ph¸p cña häc tËp nh: tri thøc logic (ph©n tÝch, so s¸nh, kh¸i qu¸t ho¸, ph©n lo¹i…); tri thøc tæ chøc hîp lÝ c¸c qu¸ tr×nh nhËn thøc kh¸c nhau… Lerner I.Ia. cßn thªm vµo ®ã hai hÖ thèng n÷a: Kinh nghiÖm ho¹t ®éng s¸ng t¹o vµ kinh nghiÖm th¸i ®é t×nh c¶m. C¸c nhµ lÝ luËn d¹y häc P.I. Pitcaxixti,B.I. C«r«tiaiev kh¼ng ®Þnh: t¬ng øng víi hai lo¹i ho¹t ®éng nhËn thøc t¸i t¹o vµ t×m tßi, s¸ng t¹o cña HS th× cã hai lo¹i th«ng tin t¸i hiÖn vµ dù ®o¸n. Th«ng tin t¸i hiÖn lµ nh÷ng tri thøc HS lÜnh héi ë d¹ng cã s½n, th«ng qua viÖc ghi nhËn vµ t¸i hiÖn l¹i. Th«ng tin dù ®o¸n lµ c¸c tri thøc häc tËp ®îc HS kh«i phôc l¹i b»ng c¸ch thiÕt kÕ, t×m kiÕm vµ kiÓm tra tÝnh ®óng ®¾n cña ®iÒu dù ®o¸n. Trong khi ho¹t ®éng t¸i hiÖn chØ cã mét ph¬ng ¸n vµ viÖc thùc hiÖn nã chÝnh x¸c lu«n dÉn ®Õn kÕt qu¶, th× ho¹t ®éng t×m tßi s¸ng t¹o l¹i dùa vµo nh÷ng th«ng tin Èn tµng, cha têng minh. HS kiÓm tra dù ®o¸n trªn c¬ së t×m kiÕm vµ lùa chän ph¬ng ¸n cã kh¶ n¨ng nhÊt trong hÖ thèng kiÕn thøc ®· cã cña m×nh. Dùa vµo kÕt qu¶ nghiªn cøu cña P.I. Pitcaxixti, B.I. C«r«tiaiev cã hai c¸ch chiÕm lÜnh kiÕn thøc: 1. T¸i hiÖn kiÕn thøc: ®Þnh híng ®Õn ho¹t ®éng t¸i t¹o, ®îc x©y dùng trªn c¬ së HS lÜnh héi c¸c tiªu chuÈn, h×nh mÉu cã s½n. 2. T×m kiÕm kiÕn thøc: ®Þnh híng ®Õn ho¹t ®éng c¶i t¹o tÝch cùc, dÉn ®Õn viÖc “ph¸t minh” kiÕn thøc vµ kinh nghiÖm ho¹t ®éng . Nh vËy, PPDH nµo ®¶m b¶o phèi hîp gi÷a c¸ch d¹y t¸i hiÖn kiÕn thøc vµ t×m kiÕm kiÕn thøc, trong ®ã tËn dông c¬ héi vµ ®iÒu kiÖn ®Ó c¸ch d¹y t×m kiÕm kiÕn thøc chiÕm u thÕ, ®ång thêi kÕt hîp hµi hoµ víi tÝnh s½n sµng häc tËp cña HS, th× vÒ c¬ b¶n PPDH ®ã cã kh¶ n¨ng tÝch cùc ho¸ ho¹t ®éng häc tËp cña HS 3. Dạy luyện tập toán cho học sinh 4
  • 5. Chuyªn ®Ò BDHSG To¸n THCS Qu¸ tr×nh d¹y häc lµ mét qu¸ tr×nh t©m lý. Trong qu¸ tr×nh häc tËp HS ph¶i c¶m gi¸c, tri gi¸c, vËn dông trÝ nhí, t×nh c¶m, ý chÝ... Trong nh÷ng n¨m gÇn ®©y, t©m lý häc ®· chó ý vµo “d¹y häc ph¸t triÓn”: D¹y häc ®i tríc sù ph¸t triÓn, ®iÒu ®ã cã nghÜa lµ d¹y häc ph¶i tiÕn hµnh trong ®iÒu kiÖn dù kiÕn ®îc møc ®é ph¸t triÓn cña HS cao h¬n hiÖn t¹i. Quan ®iÓm d¹y häc ®i tríc sù ph¸t triÓn vµ kÐo theo sù ph¸t triÓn ®îc coi lµ quan ®iÓm khoa häc vµ c¸ch m¹ng. Theo L.X. Vygotski, d¹y häc ph¶i theo ®óng chøc n¨ng cña nã, ph¶i ®i tríc sù ph¸t triÓn, nã sÏ thóc ®Èy, kÐo theo sù ph¸t triÓn ®i lªn. MÊu chèt cña d¹y häc ph¸t triÓn lµ x¸c ®Þnh ®óng c¸c tr×nh ®é ph¸t triÓn cña häc sinh: Tr×nh ®é ph¸t triÓn hiÖn thêi vµ kh¶ n¨ng ph¸t triÓn gÇn nhÊt. Møc ®é hiÖn t¹i ®îc biÓu hiÖn qua qu¸ tr×nh HS ®éc lËp gi¶i quyÕt nhiÖm vô, kh«ng cÇn sù trî gióp tõ bªn ngoµi. Cßn kh¶ n¨ng ph¸t triÓn gÇn nhÊt ®îc thÓ hiÖn trong t×nh huèng HS hoµn thµnh nhiÖm vô khi cã sù hîp t¸c, gióp ®ì cña ngêi kh¸c. Tõ ®ã «ng ®a ra nguyªn lý d¹y häc ph¶i t¸c ®éng vµo vïng ph¸t triÓn gÇn nhÊt, cã nghÜa lµ ph¬ng ph¸p d¹y häc tu©n theo nguyªn t¾c t«n träng kinh nghiÖm ®· cã cña HS vµ t¨ng dÇn møc ®é khã kh¨n. Nh vËy d¹y häc kh«ng bÞ ®éng chê sù ph¸t triÓn, mµ ngîc l¹i ph¸t triÓn c¸c chøc n¨ng t©m lÝ. VÊn ®Ò ®éng c¬ häc tËp, høng thó nhËn thøc cã ý nghÜa rÊt quan träng ®Õn hiÖu qu¶ cña qu¸ tr×nh d¹y häc. §Ó ®¶m b¶o thµnh c«ng cña qu¸ tr×nh d¹y häc, thÇy gi¸o ph¶i ®Æc biÖt chó ý tíi mÆt t©m lý trong qu¸ tr×nh d¹y häc. - D¹y häc lµ mét qu¸ tr×nh x· héi, trong ®ã cã sù t¬ng t¸c gi÷a ngêi vµ ngêi, gi÷a ngêi vµ x· héi. HiÓu ®îc tÝnh x· héi cña d¹y häc vµ ¶nh hëng cña x· héi ®èi víi nhµ trêng sÏ gióp ngêi d¹y ®iÒu khiÓn ®îc qu¸ tr×nh d¹y häc. Sù gièng nhau vµ kh¸c nhau vÒ yªu cÇu x· héi, vÒ sù ph¸t triÓn nh©n c¸ch cña tõng ngêi ®ßi hái mét qu¸ tr×nh d¹y häc thèng nhÊt cïng víi biÖn ph¸p ph©n ho¸, do vËy trong ho¹t ®éng cña m×nh ngêi thÇy cÇn quan t©m ®Õn kinh nghiÖm sèng vµ ®iÒu kiÖn häc tËp thùc tÕ cña HS ®Ó x©y dùng kÕ ho¹ch vµ néi dung d¹y häc thÝch hîp. Tãm l¹i, c¨n cø vµo nhËn thøc vÒ qu¸ tr×nh d¹y häc trong giai ®o¹n hiÖn nay (qu¸ tr×nh nhËn thøc, qu¸ tr×nh t©m lý vµ qu¸ tr×nh x· héi), hÖ thèng BT cÇn ph¶i 5
  • 6. Chuyªn ®Ò BDHSG To¸n THCS ph¶n ¸nh tÝch cùc vµ cã chän läc c¸c tri thøc, ph¬ng ph¸p, kü n¨ng ... liªn quan chÆt chÏ ®Õn ho¹t ®éng to¸n häc, thóc ®Èy c¸c chøc n¨ng t©m lý, høng thó nhËn thøc vµ chó ý ®Õn kinh nghiÖm sèng vµ ®iÒu kiÖn thùc tÕ cña häc sinh. TiÕn tr×nh gi¶i bµi tËp to¸n Gi¶i BTT lµ thùc hiÖn mét lo¹t c¸c ho¹t ®éng liªn tôc vµ kh¸ phøc t¹p v× BTT lµ sù kÕt hîp ®a d¹ng nhiÒu kh¸i niÖm , quan hÖ to¸n häc... V× vËy ®Ó gi¶i ®ược BTT ®ßi hái häc sinh n¾m ch¾c c¸c kh¸i niÖm, ®Þnh lý, quy t¾c... c¸c kiÕn thøc trong mèi quan hÖ to¸n häc cña ch¬ng tr×nh ®· häc. Theo V.M. Bra®ix¬ “BTT cã thÓ xem lµ ®· ®îc gi¶i chØ sau khi ®· t×m ®ược lêi gi¶i ®¶m b¶o c¸c ®iÒu kiÖn: Kh«ng sai sãt, cã lËp luËn khoa häc, mang tÝnh toµn diÖn vµ tèi ưu”. Theo Polya G. : “Gi¶i mét BTT chóng ta ph¶i lËp ®ược mét lưîc ®å x¸c ®Þnh m¹nh l¹c nh÷ng thao t¸c (l«gÝc, to¸n häc hay thùc tiÔn) b¾t ®Çu tõ gi¶ thiÕt vµ kÕt thóc b»ng kÕt luËn, dÉn d¾t tõ c¸c ®èi tượnng mµ ta cã trong tay ®Õn ®èi t- ưîng ta muèn ®¹t tíi”. + Polya G. quan niÖm gi¶i mét BTT lµ mét qu¸ tr×nh t×m kiÕm nh÷ng ho¹t ®éng thÝch hîp ®Ó ®¹t kÕt qu¶. Theo «ng tiÕn tr×nh gi¶i mét BTT gåm 4 bước: - HiÓu râ BTT (understanding the problem) - X©y dùng chư¬ng tr×nh gi¶i (devising a plan) - Thùc hiÖn chư¬ng tr×nh gi¶i (carrying out the plan) - KiÓm tra lêi gi¶i t×m ®ược (looking back) Bước 1: HiÓu râ bµi tËp to¸n - X¸c ®Þnh ®èi tưîng vµ c¸c ®iÒu kiÖn vµ hÖ thèng hµnh ®éng, lµm râ c¸c mèi quan hÖ ë gi¶ thiÕt, mèi quan hÖ gi÷a gi¶ thiÕt vµ kÕt luËn. X¸c ®Þnh ®ùîc d¹ng BTT, xem xÐt cÊu tróc cña BTT tõ ®ã suy nghÜ hùíng gi¶i BTT ®ã. Bíc 2: X©y dùng chư¬ng tr×nh gi¶i - Tõ sù ph©n tÝch mèi quan hÖ gi÷a c¸c yÕu tè cña BTT, tõ suy nghÜ hùíng gi¶i ë bùíc 1, HS t×m con ®ùêng cô thÓ, kh¶ n¨ng ®¹t ®îc môc ®Ých, ®Þnh hùíng c¸c hµnh ®éng tiÕn tíi qu¸ tr×nh gi¶i BTT. - Qu¸ tr×nh nµy kÕt hîp gi÷a logic h×nh thøc (viÖc v¹ch ra “cÊu tróc” cña kÕ ho¹ch) vµ l«gic biÖn chøng (chØ ra tÝnh cô thÓ, tÝnh kh¶ thi vµ phù¬ng thøc thùc hiÖn kÕ ho¹ch). 6
  • 7. Chuyªn ®Ò BDHSG To¸n THCS Bíc 3: Thùc hiÖn chù¬ng tr×nh gi¶i - KÕ ho¹ch gi¶i vÉn cßn ë ý tùëng, HS ph¶i thùc hiÖn mét hÖ thèng hµnh ®éng phï hîp víi nh÷ng chi tiÕt cô thÓ cña BTT. - Sö dông c¸c thao t¸c tư duy nh÷ng lËp luËn logic ®Ó thùc hiÖn kÕ ho¹ch. - Cã thÓ gi¶i BTT theo nhiÒu c¸ch gi¶i kh¸c nhau, t×m ra c¸ch gi¶i tèi ưu. - ë b ư íc nµy thao t¸c tư duy logÝc, ho¹t ®éng ng«n ng÷ ®ãng vai trß quan träng. Bíc 4: Kh¶o s¸t lêi gi¶i t×m ®ưîc - C«ng viÖc ®ưîc tiÕn hµnh trong suèt qu¸ tr×nh gi¶i BTT, viÖc kiÓm tra nh»m chÝnh x¸c ho¸ lêi gi¶i (c¸c bưíc suy luËn, c¸c kh©u tÝnh to¸n...). - Qua kh¶o s¸t lêi gi¶i cßn rót ra ®îc kinh nghiÖm cho HS, gi¶i bµi BTT lµ ph¬ng tiÖn häc tËp. Tõ kh¶o s¸t lêi gi¶i HS cã thÓ hîp thøc ho¸ BTT thµnh tri thøc vµ kinh nghiÖm cña b¶n th©n. * Gi¶i bµi tËp to¸n theo ®Þnh híng ang«rit vµ ¬ristic “C¸c ang«rit tån t¹i dưíi nhiÒu h×nh thøc biÓu diÔn kh¸c nhau, ng«n ng÷ tù nhiªn, ng«n ng÷ to¸n häc, s¬ ®å khèi, ng«n ng÷ phư¬ng tr×nh, lËp tr×nh..., ang«rit (thuËt to¸n, thuËt gi¶i) lµ mét b¶n quy ®Þnh nh÷ng thao t¸c cÇn thùc hiÖn ®Ó gi¶i mét BTT” . “ThuËt to¸n ®ưîc hiÓu nh mét quy t¾c mµ tõ nh÷ng chØ dÉn râ rµng vµ chÝnh x¸c ®Ó ngưêi (hay m¸y) thùc hiÖn mét lo¹t thao t¸c nh»m ®¹t ®îc môc ®Ých ®Æt ra hay gi¶i mét líp BTT nhÊt ®Þnh”. R. §Ò C¸c ®· nghÜ ®Õn mét phư¬ng ph¸p toµn n¨ng ®Ó gi¶i mäi bµi tËp, Leibnis th× ®a ra ý niÖm râ rµng vÒ mét ph¬ng ph¸p toµn mü ®Ó gi¶i to¸n. Polya G. ®· rÊt ®Ò cao viÖc h×nh thµnh vµ ph¸t triÓn n¨ng lùc s¸ng t¹o qua gi¶i bµi tËp to¸n nhng «ng ®· kh¼ng ®Þnh “T×m kiÕm mét phư¬ng ph¸p toµn n¨ng vµ toµn mü ch¼ng mang l¹i kÕt qu¶ g× h¬n ®i t×m mét viªn ®¸ thÇn kú, ®Ó cã thÓ biÕn mäi kim lo¹i thµnh vµng” . Như vËy kh«ng thÓ cã phư¬ng ph¸p ®Ó gi¶i tÊt c¶ c¸c BTT, ngoµi nh÷ng d¹ng to¸n cã thÓ dïng phư¬ng ph¸p theo ®Þnh hưíng ang«rit cßn ph¶i sö dông phư¬ng ph¸p ¬ristic. ThuËt ng÷ ¬ristic cã nguån gèc Hy L¹p lµ “¬rªca” ®ưîc hiÓu lµ sù t×m tßi, t×m ®o¸n, s¸ng t¹o… 7
  • 8. Chuyªn ®Ò BDHSG To¸n THCS “ ¥ristic ®îc hiÓu lµ tæng thÓ nãi chung c¸c quy t¾c phư¬ng ph¸p kh¸i qu¸t tõ kinh nghiÖm qu¸ khø ®ưîc dïng trong qu¸ tr×nh nghiªn cøu ph¸t hiÖn, s¸ng t¹o ra c¸i míi”. Gi¶i to¸n theo ®Þnh hưíng ¥ristÝc mang tÝnh chÊt “t×m ®o¸n” thưêng dïng ®Ó gi¶i nh÷ng BTT mang tÝnh chÊt lµ mét vÊn ®Ò, t×m hiÓu vµ ph¸t hiÖn ra vÊn ®Ò, t×m c¸ch gi¶i quyÕt vÊn ®Ò ®ã lµ ho¹t ®éng to¸n häc cÇn thiÕt. Cách thức học phương pháp tìm lời giải bài toán Học phương pháp tìm lời giải không phải là học một thuật giải mà học những kinh nghiệm giải toán mang tính chất tìm tòi, phát hiện. Do đó cách thức học phương pháp để tìm lời giải bài toán yêu cầu: - Thông qua việc giải những bài toán cụ thể, học sinh cần nắm được các bước tìm lời giải bài toán và có ý thức vận dụng các bước đó trong quá trình giải toán. - Cũng thông qua việc giải những bài toán cụ thể, giáo viên cần đặt ra cho học sinh những câu hỏi gợi ý, tạo tình huống để các em tìm tòi, dự đoán, phát hiện và cuối cùng tìm ra lời giải bài toán. Như vậy, quá trình học sinh học phương pháp tìm lời giải bài toán là một quá trình biến những tri thức, phương pháp tổng quát thành kinh nghiệm giải toán của bản thân mình thông qua việc giải hàng loạt bài toán cụ thể. do đó đòi hỏi người giải toán phải có một chặng đường lao động tích cực, có nhiều yếu tố sáng tạo. Cã ý kiÕn cho r»ng “d¹y luyÖn tËp to¸n gièng nh luyÖn tËp qu©n sù”; ®èi víi ®èi tîng häc sinh kh¸, giái thÇy chØ híng dÉn c¸c thao t¸c vµ HS tù m×nh lµm ®îc; ®èi víi häc sinh trung b×nh thÇy lµm mÉu c¸c ®éng t¸c vµ häc sinh lµm theo ®îc; ®èi víi HS yÕu thÇy gi¸o ph¶i cho HS lµm tõng ®éng t¸c theo m×nh cho ®Õn lóc HS tù lµm ®îc mµ kh«ng cã thÇy lµm mÉu ë phÝa tríc. Qua thùc tÕ d¹y häc vµ c«ng t¸c qu¶n lý d¹y vµ häc, chóng t«i ®a ra mét sè ®Þnh híng cho tiÕt d¹y luyÖn tËp h×nh häc nh sau: + Ph©n lo¹i c¸c bµi tËp ë SGK, SBT (BTT còng cè kiÕn thøc cña bµi häc, BTT «n kiÕn thøc cña bµi häc tríc, BT bæ sung lý thuyÕt, BT kh¾c s©u kiÕn thøc). + C¨n cø vµo ®èi tîng HS cña líp gi¸o viªn gi¶ng d¹y ®Ó lùa chän mét trong c¸c ý tëng : - Víi ®èi tîng häc sinh cña líp lµ häc sinh häc trung b×nh vµ yÕu m«n to¸n: . Thêng dïng c¸c BTT «n tËp kiÕn thøc ®Ó kiÓm tra nhanh ®Çu tiÕt luyÖn tËp (nh÷ng bµi cha ®îc sö dông sau phÇn häc lý thuyÕt). 8
  • 9. Chuyªn ®Ò BDHSG To¸n THCS . Híng dÉn häc sinh gi¶i c¸c BTT bæ sung lý thuyÕt vµ BTT kh¾c s©u kiÕn thøc (c¸c BT ë SGK vµ SBT ®îc lùa chän), qua viÖc sö dông c¸c BTT ®ã, gióp häc sinh t×m ®îc quy tr×nh hoÆc ®Þnh híng gi¶i c¸c BTT cïng d¹ng. - Víi ®èi tîng lµ häc sinh trung b×nh vµ trung b×nh kh¸: .C¬ b¶n häc sinh ®· gi¶i ®îc c¸c BTT thÇy gi¸o ra vÒ nhµ chuÈn bÞ nªn nÕu ®Õn líp trong tiÕt d¹y luyÖn tËp thÇy gi¸o híng dÉn gi¶i c¸c BTT ®ã sÏ kh«ng t¹o ra ®îc sù míi mÏ, dÉn ®Õn HS kh«ng høng thó trong häc tËp. Trong trêng hîp nµy gi¸o viªn chän mét BTT t¬ng tù vµ thªm c¸c c©u hái nh»m x©u chuçi c¸c BTT, HS ®· ®îc chuÈn bÞ. Thùc tÕ cho thÊy HS høng thó trong häc tËp vµ tiÕt d¹y thµnh c«ng h¬n nhiÒu. - §èi tîng lµ häc sinh kh¸, giái: .ThÇy gi¸o chØ kiÓm tra nhanh c¸c BTT cã tÝnh chÊt còng cè kiÕn thøc. Dïng BT ®iÓn h×nh luyÖn tËp víi c¸c ®Þnh híng kh¸c nhau nh»m t¹o ra nhiÒu c¸ch gi¶i (nÕu cã thÓ), tõ BTT ®· cã t¹o ra BTT míi b»ng c¸c ho¹t ®éng t¬ng tù, t- ¬ng tù ho¸, kh¸i qu¸t ho¸, lËt ngîc vÊn ®Ò… t¹o thµnh mét sè BTT nh»m ph¸t triÓn t duy s¸ng t¹o cho HS. Lấy một số ví dụ về dạy luyện tập hình học cho học sinh giỏi. RÌn luyÖn ho¹t ®éng to¸n häc cho häc sinh kh¸, giái Theo NguyÔn B¸ Kim “d¹y häc ph©n ho¸ xuÊt ph¸t tõ sù biÖn chøng cña thèng nhÊt vµ ph©n ho¸, tõ yªu cÇu thùc hiÖn tèt c¸c môc tiªu d¹y häc ®èi víi tÊt c¶ mäi häc sinh, ®ång thêi ph¸t triÓn tèi ®a vµ tèi ưu nh÷ng kh¶ n¨ng c¸ nh©n. ViÖc kÕt hîp gi÷a gi¸o dôc ®¹i trµ vµ gi¸o dôc mòi nhän gi÷a phæ cËp víi n©ng cao trong d¹y häc to¸n phæ th«ng cÇn tiÕn hµnh theo c¸c tư tưởng chñ ®¹o sau: - LÊy tr×nh ®é ph¸t triÓn chung cña HS trong líp lµm nÒn t¶ng… - Sö dông biÖn ph¸p ph©n ho¸ ®a diÖn häc sinh yÕu kÐm lªn tr×nh ®é chung… - Cã nh÷ng néi dung bæ sung vµ biÖn ph¸p ph©n ho¸ gióp häc sinh kh¸ giái ®¹t ®îc nh÷ng yªu cÇu n©ng cao trªn c¬ së ®¹t ®ưîc nh÷ng yªu cÇu c¬ b¶n”. Chóng ta quan t©m ®Õn ý tưëng thø ba: “Cã nh÷ng néi dung bæ sung vµ biÖn ph¸p ph©n ho¸ gióp häc sinh kh¸, giái ®¹t ®ược nh÷ng yªu cÇu n©ng cao trªn c¬ së ®¹t ®ược nh÷ng yªu cÇu c¬ b¶n” 9
  • 10. Chuyªn ®Ò BDHSG To¸n THCS Víi c¸ch d¹y häc ph©n ho¸ như ®· nªu lµ ph©n ho¸ néi t¹i (ph©n ho¸ trong) trong mét líp häc. Thùc tÕ d¹y häc ë c¸c trêng THCS ®ang cßn phæ biÕn ph©n ho¸ ngoµi, nghÜa lµ cã nh÷ng líp tr×nh ®é HS kh¸ h¬n, trong nh÷ng n¨m gÇn ®©y c¸c trêng chuyªn, líp chän ë THCS kh«ng tån t¹i n÷a, nhưng víi tÝnh chÊt gi¸o dôc cã tÝnh ®¹i chóng vµ phæ cËp nh hiÖn nay th× viÖc ph©n ho¸ ngoµi vÉn lµ phæ biÕn (v× thuËn lîi h¬n trong qu¸ tr×nh d¹y häc, ë bËc THPT ph©n ban còng lµ h×nh thøc ph©n ho¸ ngoµi). C¨n cø vµo c¸c ho¹t ®éng to¸n häc liªn quan mËt thiÕt ®Õn néi dung m«n to¸n ë trưêng phæ th«ng lµ: NhËn d¹ng vµ thÓ hiÖn; nh÷ng ho¹t ®éng to¸n häc phøc hîp; nh÷ng ho¹t ®éng trÝ tuÖ phæ biÕn trong to¸n häc; nh÷ng ho¹t ®éng trÝ tuÖ chung vµ nh÷ng ho¹t ®éng ng«n ng÷, tõ ®ã trong qu¸ tr×nh d¹y häc gi¸o viªn trong mäi t×nh huèng dï tưêng minh hay Èn tµng còng ®Òu cã ý tưëng gãp phÇn rÌn luyÖn ho¹t ®éng to¸n häc cho häc sinh. 10
  • 11. Chuyªn ®Ò BDHSG To¸n THCS b.C¸c chuyªn ®Ò båi dìng hsg to¸n thcs Chuyªn ®Ò 1: PhÇn I: Sè chÝnh ph¬ng I- §Þnh nghÜa: Sè chÝnh ph¬ng lµ sè b»ng b×nh ph¬ng ®óng cña mét sè nguyªn. II- tÝnh chÊt: 1- Sè chÝnh ph¬ng chØ cã thÓ cã ch÷ sè tËn cïng b»ng 0, 1, 4, 5, 6, 9; kh«ng thÓ cã ch÷ tËn cïng b»ng 2, 3, 7, 8. 2- Khi ph©n tÝch ra thõa sè nguyªn tè, sè chÝnh ph¬ng chØ chøa c¸c thõa sè nguyªn tè víi sè mò ch½n. 3- Sè chÝnh ph¬ng chØ cã thÓ cã mét trong hai d¹ng 4n hoÆc 4n+1. Kh«ng cã sè chÝnh ph¬ng nµo cã d¹ng 4n + 2 hoÆc 4n + 3 (n ∈ N). 4- Sè chÝnh ph¬ng chØ cã thÓ cã mét trong hai d¹ng 3n hoÆc 3n +1. Kh«ng cã sè chÝnh ph¬ng nµo cã d¹ng 3n + 2 ( n ∈ N ). 5- Sè chÝnh ph¬ng tËn cïng b»ng 1, 4 hoÆc 9 th× ch÷ sè hµng chôc lµ ch÷ sè ch½n. Sè chÝnh ph¬ng tËn cïng b»ng 5 th× ch÷ sè hµng chôc lµ 2. Sè chÝnh ph¬ng tËn cïng b»ng 6 th× ch÷ sè hµng chôc lµ ch÷ sè lÎ. 6- Sè chÝnh ph¬ng chia hÕt cho 2 th× chia hÕt cho 4. Sè chÝnh ph¬ng chia hÕt cho 3 th× chia hÕt cho 9 Sè chÝnh ph¬ng chia hÕt cho 5 th× chia hÕt cho 25 Sè chÝnh ph¬ng chia hÕt cho 8 th× chia hÕt cho 16. III- Mét sè d¹ng bµi tËp vÒ sè chÝnh ph¬ng. A- D¹ng 1: chøng minh mét sè lµ sè chÝnh ph¬ng. Bµi 1: Chøng minh r»ng mäi sè nguyªn x, y th×: A= (x + y)(x + 2y)(x + 3y)(x + 4y) + 4 y lµ sè chÝnh ph¬ng. Gi¶i : Ta cã A = (x + y)(x + 2y)(x + 3y)(x + 4y) + 4 y = ( 2 2 2 2 4 5 4 )( 5 6 )x xy y x xy y y+ + + + + §Æt 2 2 5 5 ( )x xy y t t Z+ + = ∈ th× A = ( 2 2 4 2 4 4 2 2 2 2 )( ) ( 5 5 )t y t y y t y y t x xy y− + + = − + = = + + V× x, y, z ∈ Z nªn 2 2 2 2 , 5 , 5 5 5x Z xy Z y Z x xy y Z∈ ∈ ∈ ⇒ + + ∈ VËy A lµ sè chÝnh ph¬ng. Bµi 2: Chøng minh tÝch cña 4 sè tù nhiªn liªn tiÕp céng 1 lu«n lµ sè chÝnh ph- ¬ng. Gi¶i : Gäi 4 sè tù nhiªn, liªn tiÕp ®ã lµ n, n+1, n+2, n+3 (n ∈ Z). Ta cã: n(n + 1)(n + 2)(n + 3) + 1 = n . ( n + 3)(n + 1)(n + 2) + 1 = ( 2 2 3 )( 3 2) 1 (*)n n n n+ + + + §Æt 2 3 ( )n n t t N+ = ∈ th× (*) = t(t + 2) + 1 = t2 + 2t + 1 = (t + 1)2 11
  • 12. Chuyªn ®Ò BDHSG To¸n THCS = (n2 + 3n + 1)2 V× n ∈ N nªn n2 + 3n + 1 ∈ N. VËy n(n + 1)(n + 2)(+ 3) + 1 lµ sè chÝnh ph¬ng. Bµi 3: Cho S = 1.2.3 + 2.3.4 + 3.4.5 + ...+ k(k + 1)(k + 2) Chøng minh r»ng 4S + 1 lµ sè chÝnh ph¬ng. Gi¶i : Ta cã: k(k + 1)(k + 2) = 1 4 k (k + 1)(k + 2). 4= 1 4 k(k + 1)(k + 2). [ ]( 3) ( 1)k k+ − − = 1 4 k(k + 1)(k + 2)(k + 3) - 1 4 k(k + 1)(k + 2)(k - 1) => 4S =1.2.3.4 - 0.1.2.3 + 2.3.4.5 - 1.2.3.4 + . . . + k(k + 1)(k + 2)(k + 3) - k(k + 1)(k + 2)(k - 1) = k(k + 1)(k + 2)(k + 3) => 4S + 1 = k(k + 1)(k + 2)(k + 3) + 1 Theo kÕt qu¶ bµi 2 => k(k + 1)(k + 2)(k + 3) + 1 lµ sè chÝnh ph¬ng. Bµi 4: Cho d·y sè 49; 4489; 444889; 44448889; . . . - D·y sè trªn ®îc x©y dùng b»ng c¸ch thªm sè 48 vµo gi÷a c¸c ch÷ sè ®øng tríc vµ ®øng sau nã. Chøng minh r»ng tÊt c¶ c¸c sè cña d·y trªn ®Òu lµ sè chÝnh ph¬ng. Ta cã 44 ...488...89 = 44...488...8 + 1 = 44...4 . 10n + 8 . 11 ... 1 + 1 n ch÷ sè 4 n - 1 ch÷ sè 8 n ch÷ sè 4 n ch÷ sè 8 n ch÷ sè 4 n ch÷ sè 1 = 4. 10 1 10 1 .10 8. 1 9 9 n n n− − + + = 2 2 4.10 4.10 8.10 8 9 4.10 4.10 1 9 9 n n n n n − + − + + + = = 2 2.10 1 3 n  +  ÷   Ta thÊy 2.10n + 1 = 200...01 cã tæng c¸c ch÷ sè chia hÕt cho 3 nªn nã chia hÕt cho 3 n - 1 ch÷ sè 0 => 2 2.10 1 3 n  +  ÷   ∈ Z hay c¸c sè cã d¹ng 44 ... 488 ... 89 lµ sè chÝnh ph¬ng. C¸c bµi t¬ng tù: Chøng minh r»ng sè sau ®©y lµ sè chÝnh ph¬ng. A = 11 ... 1 + 44 ... 4 + 1 2n ch÷ sè 1 n ch÷ sè 4 B = 11 ... 1 + 11 . . .1 + 66 . . . 6 + 8 2n ch÷ sè 1 n+1 ch÷ sè 1 n ch÷ sè 6 C= 44 . . . 4 + 22 . . . 2 + 88 . . . 8 + 7 2n ch÷ sè 4 n+1 ch÷ sè 2 n ch÷ sè 8 D = 22499 . . .9100 . . . 09 12
  • 13. Chuyªn ®Ò BDHSG To¸n THCS n-2 ch÷ sè 9 n ch÷ sè 0 E = 11 . . .155 . . . 56 n ch÷ sè 1 n-1 ch÷ sè 5 KÕt qu¶: A= 2 2 2 10 2 10 8 2.10 7 ; ; 3 3 3 n n n B C      + + + = = ÷  ÷  ÷       D = (15.10n - 3)2 E = 2 3 210       +n Bµi 5: Chøng minh r»ng tæng c¸c b×nh ph¬ng cña 5 sè tù nhiªn liªn tiÕp kh«ng thÓ lµ mét sè chÝnh ph¬ng. Gäi 5 sè tù nhiªn liªn tiÕp ®ã lµ n - 2, n - 1, n +1, n + 2 ( n ∈ N, n >2). Ta cã (n - 2)2 + ( n - 1)2 + n2 + (n + 1)2 + (n + 2)2 = 5 . (n2 + 2) V× n2 kh«ng thÓ tËn cïng bëi 3 hoÆc 8 do ®ã n2 + 2 kh«ng thÓ chia hÕt cho 5 => 5. (n2 + 2) kh«ng lµ sè chÝnh ph¬ng hay A kh«ng lµ sè chÝnh ph¬ng. Bµi 6: Chøng minh r»ng sè cã d¹ng n6 - n4 + 2n3 + 2n2 trong ®ã n ∈ N vµ n >1 kh«ng ph¶i lµ sè chÝnh ph¬ng. n6 - n 4 + 2n3 + 2n2 = n2 . (n4 - n2 + 2n +2) = n2 . [n2 (n-1)(n+1) +2(n+1)] = n2 [(n+1)(n3 - n2 + 2)] = n2 (n + 1) . [(n3 + 1) - (n2 - 1)] = n2 (n + 1)2 . (n2 - 2n + 2) Víi n∈N, n > 1 th× n2 - 2n + 2 = ( n -1)2 + 1 > ( n - 1)2 Vµ n2 - 2n + 2 = n2 - 2(n - 1) < n2 VËy (n - 1)2 < n2 - 2n + 2 < n2 => n2 - 2n + 2 kh«ng ph¶i lµ mét sè chÝnh ph¬ng. Bµi 7: Cho 5 sè chÝnh ph¬ng bÊt kú cã ch÷ sè hµng chôc kh¸c nhau cßn ch÷ sè hµng ®¬n vÞ ®Òu lµ 6. Chøng minh r»ng tæng c¸c ch÷ sè hµng chôc cña 5 sè chÝnh ph¬ng ®ã lµ mét sè chÝnh ph¬ng. Ta biÕt mét sè chÝnh ph¬ng cã ch÷ sè hµng ®¬n vÞ lµ 6 th× ch÷ sè hµng chôc cña nã lµ sè lÎ. V× vËy ch÷ sè hµng chôc cña 5 sè chÝnh ph¬ng ®ã lµ 1,3,5,7,9 khi ®ã tæng cña chóng b»ng 1 + 3 + 5 + 7 + 9 = 25 = 52 lµ sè chÝnh ph¬ng. Bµi 8: Chøng minh r»ng tæng b×nh ph¬ng cña 2 sè lÎ bÊt kú kh«ng ph¶i lµ sè chÝnh ph¬ng. a vµ b lÎ nªn a = 2k + 1, b= 2m + 1 (Víi k, m ∈ N). => a2 + b2 = (2k + 1)2 + ( 2m + 1)2 = 4k2 + 4k + 1 + 4m2 + 4m + 1 = 4 (k2 + k + m2 + m) + 2 => a2 + b2 kh«ng thÓ lµ sè chÝnh ph¬ng. Bµi 9: Chøng minh r»ng nÕu p lµ tÝch cña n (víi n > 1) sè nguyªn tè ®Çu tiªn th× p - 1 vµ p + 1 kh«ng thÓ lµ c¸c sè chÝnh ph¬ng. V× p lµ tÝch cña n sè nguyªn tè ®Çu tiªn nªn pM2 vµ p kh«ng thÓ chia hÕt cho 4 (1) a- Gi¶ sö p + 1 lµ sè chÝnh ph¬ng. §Æt p + 1 = m2 ( m ∈ N). V× p ch½n nªn p + 1 lÎ => m2 lÎ => m lÎ. §Æt m = 2k + 1 (k ∈ N). Ta cã m2 = 4k2 + 4k + 1 => p + 1 = 4k2 + 4k + 1 13
  • 14. Chuyªn ®Ò BDHSG To¸n THCS => p = 4k2 + 4k = 4k (k + 1) M 4 m©u thuÉn víi (1). => p + 1 kh«ng ph¶i lµ sè chÝnh ph¬ng. b- p = 2.3.5... lµ sè chia hÕt cho 3 => p - 1 cã d¹ng 3k + 2. => p - 1 kh«ng lµ sè chÝnh ph¬ng. VËy nÕu p lµ tÝch n (n >1) sè nguyªn tè ®Çu tiªn th× p - 1 vµ p + 1 kh«ng lµ sè chÝnh ph¬ng. Bµi 10: Gi¶ sö N = 1.3.5.7 . . . 2007. 2011 Chøng minh r»ng trong 3 sè nguyªn liªn tiÕp 2N - 1, 2N vµ 2N + 1 kh«ng cã sè nµo lµ sè chÝnh ph¬ng. a- 2N - 1 = 2.1.3.5.7 . . . 2011 - 1 Cã 2N M 3 => 2N - 1 = 3k + 2 (k ∈ N) => 2N - 1 kh«ng lµ sè chÝnh ph¬ng. b- 2N = 2.1.3.5.7 . . . 2011 => 2N ch½n. => N lÎ => N kh«ng chia hÕt cho 2 vµ 2N M 2 nhng 2N kh«ng chia hÕt cho 4. 2N ch½n nªn 2N kh«ng chia cho 4 d 1 hoÆc d 3 => 2N kh«ng lµ sè chÝnh ph¬ng. c- 2N + 1 = 2.1.3.5.7 . . . 2011 + 1 2N + 1 lÎ nªn 2N + 1 kh«ng chia hÕt cho 4 2N kh«ng chia hÕt cho 4 nªn 2N + 1 kh«ng chia cho 4 d 1. => 2N + 1 kh«ng lµ sè chÝnh ph¬ng. Bµi 11: Cho a = 11 . . . 1 ; b = 100 . . . 05 2010 ch÷ sè 1 2009 ch÷ sè 0 Chøng minh 1ab + lµ sè tù nhiªn. Gi¶i: b = 100 . . . 05 = 100 . . . 0 - 1 + 6 = 99 . . . 9 + 6 = 9a + 6 2009 ch÷ sè 0 2010 ch÷ sè 0 2010 ch÷ sè 9 ⇒ ab + 1 = a(9a + 6) + 1 = 9a2 + 6a + 1 = (3a + 1)2 ⇒ Naaab ∈+=+=+ 13)13(1 2 B. d¹ng 2: t×m gi¸ trÞ cña biÕn ®Ó biÓu thøc lµ sè chÝnh ph¬ng Bµi 1: T×m sè tù nhiªn n sao cho c¸c sè sau lµ sè chÝnh ph¬ng a) n2 + 2n + 12 b) n(n + 3) c) 13n + 3 d) n2 + n + 1589 Gi¶i: a) V× n2 + 2n + 12 lµ sè chÝnh ph¬ng nªn ®Æt n2 + 2n + 12 = k2 (k ∈ N) ⇒ (n2 + 2n + 1) + 11 = k2 ⇔ k2 – (n + 1)2 = 11 ⇔ (k + n + 1)(k – n - 1) = 11 NhËn xÐt thÊy k + n + 1 > k - n - 1 vµ chóng lµ nh÷ng sè nguyªn d¬ng, nªn ta cã thÓ viÕt (k + n + 1) (k - n - 1) = 11.1 ⇔ k + n + 1 = 11 ⇔ k = 6 k - n – 1 = 1 n = 4 b) ®Æt n(n + 3) = a2 (n ∈ N) ⇒ n2 + 3n = a2 ⇔ 4n2 + 12n = 4a2 ⇔ (4n2 + 12n + 9) – 9 = 4a2 ⇔ (2n + 3)2 – 4a2 = 9 ⇔ (2n + 3 + 2a)(2n + 3 – 2a) = 9 14
  • 15. Chuyªn ®Ò BDHSG To¸n THCS NhËn xÐt thÊy 2n + 3 + 2a > 2n + 3 – 2a vµ chóng lµ nh÷ng sè nguyªn d¬ng, nªn ta cã thÓ viÕt (2n + 3 + 2a)(2n + 3 – 2a) = 9.1 ⇔ 2n + 3 + 2a = 9 ⇔ n = 1 2n + 3 – 2a = 1 a = 2 c) §Æt 13n + 3 = y2 (y ∈ N) ⇒ 13(n - 1) = y2 – 16 ⇔ 13(n - 1) = (y + 4)(y – 4) ⇒(y + 4)(y – 4) M 13 mµ 13 lµ sè nguyªn tè nªn y + 4 M 13 hoÆc y – 4 M 13 ⇒ y = 13k ± 4 (víi k ∈ N) ⇒ 13(n - 1) = (13k ± 4)2 – 16 = 13k.(13k ± 8) ⇒13k2 ± 8k + 1 VËy n = 13k2 ± 8k + 1 (víi k ∈ N) th× 13n + 3 lµ sè chÝnh ph¬ng d) §Æt n2 + n + 1589 = m2 (m ∈ N) ⇒ (4n2 + 1)2 + 6355 = 4m2 ⇔ (2m + 2n + 1) (2m – 2n – 1) = 6355 NhËn xÐt thÊy 2m + 2n + 1 > 2m – 2n – 1 > 0 vµ chóng lµ nh÷ng sè lÎ, nªn ta cã thÓ viÕt (2m + 2n + 1) (2m – 2n – 1) = 6355.1 = 1271.5 = 205.31 = 155.41 Suy ra n cã thÓ cã c¸c gi¸ trÞ sau : 1588 ; 316 ; 43 ; 28 Bµi t¬ng tù : T×m a ®Ó c¸c sè sau lµ nh÷ng sè chÝnh ph¬ng a) a2 + a + 43 b) a2 + 81 c) a2 + 31a + 1984 KÕt qu¶: a) 2; 42; 13 b) 0; 12; 40 c) 12 ; 33 ; 48 ; 97 ; 176 ; 332 ; 565 ; 1728 Bµi 2 : T×m sè tù nhiªn n ≥ 1 sao cho tæng 1! + 2! + 3! + … + n! lµ mét sè chÝnh ph¬ng. Víi n = 1 th× 1! = 1 = 12 lµ sè chÝnh ph¬ng Víi n = 2 th× 1! + 2! = 3 kh«ng lµ sè chÝnh ph¬ng Víi n = 3 th× 1! + 2! + 3! = 1 + 1.2 + 1.2.3 = 9 = 33 lµ sè chÝnh ph¬ng Víi n ≥ 4 ta cã 1! + 2! + 3! + 4! = 1 + 1.2 + 1.2.3 + 1.2.3.4 = 33 cßn 5!; 6!; …; n! ®Òu tËn cïng bëi 0 do ®ã 1! + 2! + 3! + … n! cã tËn cïng bëi ch÷ sè 3 nªn nã kh«ng ph¶i lµ sè chÝnh ph¬ng. VËy cã 2 sè tù nhiªn n tho¶ m·n ®Ò bµi lµ n = 1; n = 3 Bµi 3: Cã hay kh«ng sè tù nhiªn n ®Ó 2010 + n2 lµ sè chÝnh ph¬ng. Gi¶ sö 2010 + n2 lµ sè chÝnh ph¬ng th× 2010 + n2 = m2 (m N∈ ) Tõ ®ã suy ra m2 - n2 = 2010 ⇔ (m + n) (m – n) = 2010 Nh vËy trong 2 sè m vµ n ph¶i cã Ýt nhÊt 1 sè ch½n (1) MÆt kh¸c m + n + m – n = 2m ⇒ 2 sè m + n vµ m – n cïng tÝnh ch½n lÎ (2) Tõ (1) vµ (2) ⇒ m + n vµ m – n lµ 2 sè ch½n. ⇒ (m + n) (m – n) M 4 nhng 2006 kh«ng chia hÕt cho 4 ⇒ §iÒu gi¶ sö sai. VËy kh«ng tån t¹i sè tù nhiªn n ®Ó 2006 + n2 lµ sè chÝnh ph¬ng. Bµi 4: BiÕt x N∈ vµ x > 2. T×m x sao cho )1()2()1(.)1( −−=−− xxxxxxxx §¼ng thøc ®· cho ®îc viÕt l¹i nh sau: )1()2()1( 2 −−=− xxxxxx 15
  • 16. Chuyªn ®Ò BDHSG To¸n THCS Do vÕ tr¸i lµ mét sè chÝnh ph¬ng nªn vÕ ph¶i còng lµ mét sè chÝnh ph¬ng. Mét sè chÝnh ph¬ng chØ cã thÓ tËn cïng bëi mét trong c¸c ch÷ sè 0; 1; 4; 5; 6; 9 nªn x chØ cã thÓ tËn cïng bëi mét trong c¸c ch÷ sè 1; 2; 5; 6; 7; 0 (1) Do x lµ ch÷ sè nªn x ≤ 9, kÕt hîp víi ®iÒu kiÖn ®Ò bµi ta cã x N∈ vµ 2 < x ≤ 9 (2) Tõ (1) vµ (2) ⇒ x chØ cã thÓ nhËn mét trong c¸c gi¸ trÞ 5; 6; 7 B»ng phÐp thö ta thÊy chØ cã x = 7 tho¶ m·n ®Ò bµi, khi ®ã 762 = 5776 Bµi 5: T×m sè tù nhiªn n cã 2 ch÷ sè biÕt r»ng 2n + 1 vµ 3n + 1 ®Òu lµ c¸c sè chÝnh ph¬ng. Ta cã 10 ≤ n ≤ 99 nªn 21 ≤ 2n + 1 ≤ 199. T×m sè chÝnh ph¬ng lÎ trong kho¶ng trªn ta ®îc 2n + 1 b»ng 25; 49; 81; 121; 169 t¬ng øng víi sè n b»ng 12; 24; 40; 60; 84 Sè 3n + 1 b»ng 37; 73; 121; 181; 253. ChØ cã 121 lµ sè chÝnh ph¬ng. VËy n = 40 Bµi 6: Chøng minh r»ng nÕu n lµ sè tù nhiªn sao cho n + 1 vµ 2n + 1 ®Òu lµ c¸c sè chÝnh ph¬ng th× n lµ béi sè cña 24 V× n + 1 vµ 2n + 1 lµ c¸c sè chÝnh ph¬ng nªn ®Æt n + 1 = k2 , 2n + 1 = m2 (k, m N∈ ) Ta cã m lµ sè lÎ ⇒ m = 2a + 1 ⇒ m2 = 4a(a + 1) + 1 Mµ )1(2 2 )1(4 2 12 += + = − = aa aam n ⇒ n ch½n ⇒ n + 1 lÎ ⇒ k lÎ ⇒ ®Æt k = 2b + 1 (víi b N∈ ) ⇒ k2 = 4b(b+1) + 1 ⇒ n = 4b(b+1) ⇒ n M 8 (1) Ta cã: k2 + m2 = 3n + 2 ≡ 2 (mod3) MÆt kh¸c k2 chia cho 3 d 0 hoÆc 1, m2 chia cho 3 d 0 hoÆc 1 Nªn ®Ó k2 + m2 ≡ 2 (mod3) th× k2 ≡ 1 (mod3) m2 ≡ 1 (mod3) ⇒ m2 – k2 M 3 hay (2n + 1) – (n + 1) M 3 ⇒ n M 3 (2) Mµ (8; 3) = 1 (3) Tõ (1), (2), (3) ⇒ n M 24 Bµi 7: T×m tÊt c¶ c¸c sè tù nhiªn n sao cho sè 28 + 211 + 2n lµ sè chÝnh ph¬ng Gi¶ sö 28 + 211 + 2n = a2 (a ∈ N) th× 2n = a2 – 482 = (a + 48) (a – 48) 2p . 2q = (a + 48) (a – 48) víi p, q ∈ N ; p + q = n vµ p > q ⇒ a + 48 = 2p ⇒ 2p 2q = 96 ⇔ 2q (2p-q – 1) = 25 .3 a – 48 = 2q ⇒ q = 5 vµ p – q = 2 ⇒ p = 7 ⇒ n = 5 + 7 = 12 Thö l¹i ta cã: 28 + 211 + 2n = 802 C.d¹ng 3 : T×m sè chÝnh ph¬ng Bµi 1 : Cho A lµ sè chÝnh ph¬ng gåm 4 ch÷ sè. NÕu ta thªm vµo mçi ch÷ sè cña A mét ®¬n vÞ th× ta ®îc sè chÝnh ph¬ng B. H·y t×m c¸c sè A vµ B. Gäi A = 2 kabcd = . NÕu thªm vµo mçi ch÷ sè cña A mét ®¬n vÞ th× ta cã sè 16
  • 17. Chuyªn ®Ò BDHSG To¸n THCS B = 2 )1)(1)(1)(1( mdcba =++++ víi k, m ∈ N vµ 32 < k < m < 100 a, b, c, d = 9;1 ⇒ Ta cã: A = 2 kabcd = B = 2 1111 mabcd =+ . §óng khi céng kh«ng cã nhí ⇒ m2 – k2 = 1111 ⇔ (m - k)(m + k) = 1111 (*) NhËn xÐt thÊy tÝch (m – k)(m + k) > 0 nªn m – k vµ m + k lµ 2 sè nguyªn d¬ng. Vµ m – k < m + k < 200 nªn (*) cã thÓ viÕt (m – k) (m + k) = 11.101 Do ®ã: m – k = 11 ⇔ m = 56 ⇔ A = 2025 m + k = 101 n = 45 B = 3136 Bµi 2: T×m mét sè chÝnh ph¬ng gåm 4 ch÷ sè biÕt r»ng sè gåm 2 ch÷ sè ®Çu lín h¬n sè gåm 2 ch÷ sè sau mét ®¬n vÞ. §Æt 2 kabcd = ta cã 1=−cdab vµ k ∈ N, 32 ≤ k < 100 Suy ra : 101cd = k2 – 100 = (k – 10)(k + 10) ⇒ k + 10 M 101 hoÆc k – 10 M 101 Mµ (k – 10; 101) = 1 ⇒ k + 10 M 101 V× 32 ≤ k < 100 nªn 42 ≤ k + 10 < 110 ⇒ k + 10 = 101 ⇒ k = 91 ⇒ abcd = 912 = 8281 Bµi 3: T×m sè chÝnh ph¬ng cã 4 ch÷ sè biÕt r»ng 2 ch÷ sè ®Çu gièng nhau, 2 ch÷ sè cuèi gièng nhau. Gäi sè chÝnh ph¬ng ph¶i t×m lµ: aabb = n2 víi a, b ∈ N, 1 ≤ a ≤ 9; 0 ≤ b ≤ 9 Ta cã: n2 = aabb = 11. ba0 = 11.(100a + b) = 11.(99a + a + b) (1) NhËn xÐt thÊy aabb M 11 ⇒ a + b M 11 Mµ 1 ≤ a ≤ 9; 0 ≤ b ≤ 9 nªn 1 ≤ a + b ≤ 18 ⇒ a + b = 11 Thay a + b = 11 vµo (1) ®îc n2 = 112 (9a + 1) do ®ã 9a + 1 lµ sè chÝnh ph¬ng B»ng phÐp thö víi a = 1; 2;…; 9 ta thÊy chØ cã a = 7 tho¶ m·n ⇒ b = 4 Sè cÇn t×m lµ: 7744 Bµi 4: T×m mét sè cã 4 ch÷ sè võa lµ sè chÝnh ph¬ng võa lµ mét lËp ph¬ng. Gäi sè chÝnh ph¬ng ®ã lµ abcd . V× abcd võa lµ sè chÝnh ph¬ng võa lµ mét lËp ph¬ng nªn ®Æt abcd = x2 = y3 víi x, y ∈ N V× y3 = x2 nªn y còng lµ mét sè chÝnh ph¬ng. Ta cã : 1000 ≤ abcd ≤ 9999 ⇒ 10 ≤ y ≤ 21 vµ y chÝnh ph¬ng ⇒ y = 16 ⇒ abcd = 4096 Bµi 5 : T×m mét sè chÝnh ph¬ng gåm 4 ch÷ sè sao cho ch÷ sè cuèi lµ sè nguyªn tè, c¨n bËc hai cña sè ®ã cã tæng c¸c ch÷ sè lµ mét sè chÝnh ph¬ng. Gäi sè ph¶i t×m lµ abcd víi a, b, c, d nguyªn vµ 1 ≤ a ≤ 9; 0 ≤ b, c, d ≤ 9 abcd chÝnh ph¬ng ⇒ d { }9,6,5,4,1,0∈ d nguyªn tè ⇒ d = 5 §Æt abcd = k2 < 10000 ⇒ 32 ≤ k < 100 k lµ mét sè cã hai ch÷ sè mµ k2 cã tËn cïng b»ng 5 ⇒ k tËn cïng b»ng 5 Tæng c¸c ch÷ sè cña k lµ mét sè chÝnh ph¬ng ⇒ k = 45 ⇒ abcd = 2025 17
  • 18. Chuyªn ®Ò BDHSG To¸n THCS VËy sè ph¶i t×m lµ: 2025 Bµi 6: T×m sè tù nhiªn cã hai ch÷ sè biÕt r»ng hiÖu c¸c b×nh ph¬ng cña sè ®ã vµ viÕt sè bë hai ch÷ sè cña sè ®ã nhng theo thø tù ngîc l¹i lµ mét sè chÝnh ph¬ng Gäi sè tù nhiªn cã hai ch÷ sèph¶i t×m lµ ab (a, b ∈ N, 1 ≤ a, b ≤ 9) Sè viÕt theo thø tù ngîc l¹i ba Ta cã ab 2 - ba 2 = (10a + b)2 – (10b + a)2 = 99 (a2 – b2 ) M 11 ⇒ a2 – b2 M 11 Hay (a - b) (a + b) M 11 V× 0 < a – b ≤ 8, 2 ≤ a + b ≤ 18 nªn a + b M 11 ⇒ a + b = 11 Khi ®ã: ab 2 - ba 2 = 32 . 112 . (a – b) §Ó ab 2 - ba 2 lµ sè chÝnh ph¬ng th× a – b ph¶i lµ sè chÝnh ph¬ng do ®ã a – b = 1 hoÆc a – b = 4 NÕu a – b = 1 kÕt hîp víi a + b = 11 ⇒ a = 6, b = 5 , ab = 65 Khi ®ã 652 – 562 = 1089 = 332 NÕu a – b = 4 kÕt hîp víi a + b = 11 ⇒ a = 7,5 lo¹i VËy sè ph¶i t×m lµ 65 Bµi 7: Cho mét sè chÝnh ph¬ng cã 4 ch÷ sè. NÕu thªm 3 vµo mçi ch÷ sè ®ã ta còng ®îc mét sè chÝnh ph¬ng. T×m sè chÝnh ph¬ng ban ®Çu. (KÕt qu¶: 1156) Bµi 8: T×m sè cã 2 ch÷ sè mµ b×nh ph¬ng cña sè Êy b»ng lËp ph¬ng cña tæng c¸c ch÷ sè cña nã. Gäi sè ph¶i t×m lµ ab víi a, b ∈ N, 1 ≤ a ≤ 9; 0 ≤ b ≤ 9 Theo gi¶ thiÕt ta cã: ab = (a + b)3 ⇔ (10a +b)2 = (a + b)3 ⇒ ab lµ mét lËp ph¬ng vµ a + b lµ mét sè chÝnh ph¬ng §Æt ab = t3 (t ∈ N), a + b = 12 (1 ∈ N) V× 10 ≤ ab ≤ 99 ⇒ ab = 27 hoÆc ab = 64 NÕu ab = 27 ⇒ a + b = 9 lµ sè chÝnh ph¬ng NÕu ab = 64 ⇒ a + b = 10 kh«ng lµ sè chÝnh ph¬ng ⇒ lo¹i VËy sè cÇn t×m lµ ab = 27 Bµi 9 : T×m 3 sè lÎ liªn tiÕp mµ tæng b×nh ph¬ng lµ mét sè cã 4 ch÷ sè gièng nhau. Gäi 3 sè lÎ liªn tiÕp ®ã lµ 2n - 1 ; 2n + 1 ; 2n + 3 (n ∈ N) Ta cã : A = (2n – 1)2 + (2n + 1)2 + (2n +3)2 = 12n2 + 12n + 11 Theo ®Ò bµi ta ®Æt 12n2 + 12n + 11 = aaaa = 1111 . a víi a lÎ vµ 1 ≤ a ≤ 9 ⇒ 12n(n + 1) = 11(101a – 1) ⇒ 101a – 1 M 3⇒ 2a – 1 M 3 V× 1 ≤ a ≤ 9 nªn 1 ≤ 2a – 1 ≤17 vµ 2a – 1 lÎ nªn 2a – 1 { }15;9;3∈ ⇒ a { }8;5;2∈ V× a lÎ ⇒ a = 5 ⇒ n = 21 18
  • 19. Chuyªn ®Ò BDHSG To¸n THCS 3 sè cÇn t×m lµ: 41; 43; 45 Bµi 10 : T×m sè cã 2 ch÷ sè sao cho tÝch cña sè ®ã víi tæng c¸c ch÷ sè cña nã b»ng tæng lËp ph¬ng c¸c ch÷ sè cña sè ®ã. ab (a + b) = a3 + b3 ⇔ 10a + b = a2 – ab + b2 = (a + b)2 – 3ab ⇔ 3a (3 + b) = (a + b) (a + b – 1) a + b vµ a + b – 1 nguyªn tè cïng nhau do ®ã a + b = 3a hoÆc a + b – 1 = 3a a + b – 1 = 3 + b a + b = 3 + b ⇒ a = 4, b = 8 hoÆc a = 3, b = 7 VËy ab = 48 hoÆc ab = 37 Chuyªn ®Ò 2: ph¬ng tr×nh nghiÖm nguyªn 1. T×m nghiÖm nguyªn cña Ph¬ng tr×nh vµ hÖ ph¬ng tr×nh bËc nhÊt hai Èn Tuú tõng bµi cô thÓ mµ lµm c¸c c¸ch kh¸c nhau. VD1: T×m nghiÖm nguyªn cña ph¬ng tr×nh: 2x + 3y = 11 (1) C¸ch 1: Ph¬ng ph¸p tæng qu¸t: Ta cã: 2x + 3y = 11 2 1 5 2 311 − −−= − =⇔ y y y x §Ó ph¬ng tr×nh cã nghiÖm nguyªn 2 1− ⇔ y nguyªn §Æt Zt y ∈= − 2 1 ⇒ y = 2t + 1 x = -3t + 4 C¸ch 2 : Dïng tÝnh chÊt chia hÕt V× 11 lÎ ⇒ 2x + 3y lu«n lµ sè lÎ mµ 2x lu«n lµ sè ch½n ⇒ 3y lÎ ⇒ y lÎ Do ®ã : y = 2t + 1 víi Zt ∈ x = -3t + 4 C¸ch 3 : Ta nh©n thÊy ph¬ng tr×nh cã mét cÆp nghiÖm nguyªn ®Æc biÖt lµ x0 = 4 ; y0 = 1 ThËt vËy : 2 . 4 + 3.1 = 11 (2) Trõ (1) cho (2) vÕ theo vÕ ta cã : 2(x - 4) + 3(y - 1) = 0 ⇔ 2(x -4) = -3(y -1) (3) Tõ (3) ⇒ 3(y - 1) M 2 mµ (2 ; 3) = 1 ⇒ y - 1 M 2 ⇔ y = 2t + 1 víi Zt ∈ Thay y = 2t + 1 vµo (3) ta cã : x = -3t + 4 19
  • 20. Chuyªn ®Ò BDHSG To¸n THCS NhËn xÐt : Víi c¸ch gi¶i nµy ta ph¶i mß ra mét cÆp nghiÖm nguyªn (x0, y0) cña ph¬ng tr×nh ax + by = c ; c¸ch nµy sÏ gÆp khã kh¨n nÕu hÖ sè a, b, c qu¸ lín. C¸c bµi tËp t¬ng tù : T×m nghiÖm nguyªn cña ph¬ng tr×nh. a) 3x + 5y = 10 b) 4x + 5y = 65 c) 5x + 7y = 112 VD2 : HÖ ph¬ng tr×nh. T×m nghiÖm nguyªn d¬ng cña hÖ ph¬ng tr×nh sau : 3x + y + z = 14 (1) 5x + 3y + z = 28 (2) Gi¶i : Tõ hÖ ®· cho ta cã : 2(x + y) = 14 vËy x = 7 - y (*) Thay (*) vµo (1) ta ®îc z = 14 - y - 3x = 2y -7 V× x > 0 nªn 7 - y > 0 ⇒ y < 7 mµ z > 0 nªn 2y - 7 > 0 ⇒ y > 2 7 VËy 2 7 < y < 7 vµ Zy ∈ { }6;5;4∈⇒ y Gi¶i tiÕp hÖ ®· cho cã 3 nghiÖm (3; 4; 1); (2; 5; 3); (1; 6; 5) Bµi tËp t¬ng tù: a) T×m nghiÖm nguyªn cña hÖ 2x -5y = 5 2y - 3z = 1 b) Tr¨m tr©u ¨n tr¨m bã cá – tr©u ®øng ¨n n¨m, tr©u n»m ¨n ba, tr©u giµ 3 con 1 bã. T×m sè tr©u mçi lo¹i. c) T×m sè nguyªn d¬ng nhá nhÊt chia cho 1000 d 1 vµ chia cho 761 d 8. 2. T×m nghiÖm nguyªn cña ph¬ng tr×nh, hÖ ph¬ng tr×nh bËc cao. Ph¬ng ph¸p 1 : Dïng dÊu hiÖu chia hÕt ®Ó gi¶i ph¬ng tr×nh. VD1: a) T×m cÆp sè nguyªn (x ; y) tho¶ m·n ph¬ng tr×nh 6x2 + 5y2 = 74 (1) C¸ch 1 : Ta cã : 6 (x2 - 4) = 5 (10 - y2 ) (2) Tõ (2) ⇒ 6(x2 - 4) M 5 vµ (6 ; 5) = 1 ⇒ x2 - 4 M 5 ⇒ x2 = 5t + 4 víi Nt ∈ Thay x2 - 4 = 5t vµo (2) ta cã : y2 = 10 – 6t V× x2 > 0 vµ y2 > 0 ⇒ 5t + 4 > 0 10 - 6t > 0 ⇒ 3 5 5 4 <<− t víi Nt ∈ ⇒ t = 0 hoÆc t = 1 Víi t = 0 ⇒ y2 = 10 (lo¹i) Víi t = 1 ⇒ x2 = 9 ⇔ x = 3± y2 = 4 y = 2± VËy c¸c cÆp nghiÖm nguyªn lµ :........................ 20
  • 21. Chuyªn ®Ò BDHSG To¸n THCS C¸ch 2 : Tõ (1) ta cã x2 + 1 M 5 0 < x2 ≤ 12 ⇒ x2 = 4 hoÆc x2 = 9 Víi x2 = 4 ⇒ y2 = 10 (lo¹i) Víi x2 = 9 ⇒ y2 = 4 (tho¶ m·n) VËy..................... C¸ch 3 : Ta cã : (1) ⇒ y2 ch½n 0 < y2 ≤ 14 ⇒ y2 = 4 ⇒ x2 = 9 VËy............... VD2 : Chøng minh r»ng ph¬ng tr×nh sau kh«ng cã nghiÖm nguyªn a) x5 + 29x = 10(3y + 1) b) 7x = 2y - 3z - 1 Gi¶i : x5 - x + 30x = 10(3y+1) VP M 30 cßn VT M 30 ⇒ ph¬ng tr×nh v« nghiÖm Ph¬ng ph¸p 2: Ph©n tÝch mét vÕ thµnh tÝch, mét vÕ thµnh h»ng sè nguyªn VD1: T×m nghiÖm nguyªn cña ph¬ng tr×nh: a) xy + 3x - 5y = -3 b) 2x2 - 2xy + x - y + 15 = 0 c) x2 + x = y2 - 19 Gi¶i : a) C¸ch 1: x(y + 3) – 5(y + 3) = -18 ⇔ (x – 5) (y + 3) = -18... C¸ch 2 : 3 18 5 3 35 + −= + − = yy y x b) T¬ng tù. c) 4x2 + 4x = 4y2 - 76 ⇔ (2x + 1)2 - (2y)2 = -75... Ph¬ng ph¸p 3 : Sö dông tÝnh ch½n lÎ (®Æc biÖt cña chia hÕt) VD2 : T×m nghiÖm nguyªn. x3 - 2y3 - 4z3 = 0 Gi¶i : ⇔ x3 = 2(y3 + 2z3 ) VP M 2 ⇒ x3 M 2 ⇒ x M 2 ®Æt x = 2k 8k3 = 2(y3 + 2z3 ) ⇔ 4k3 = y3 + 2z3 ⇒ y3 = 4k3 - 2z3 = 2(2k3 - z3 ) ⇒ y ch½n. §Æt y = 2t ta cã : 8t3 = 2(2k3 - z3 ) ⇒ 4t3 = 2k3 - z3 ⇒ z3 = 2k3 - 4t3 ⇒ z ch½n ⇒ z = 2m ⇒ 8m3 = 2(k3 - 2t3 ) ⇒ ......k ch½n....... Ph¬ng ph¸p 4 : Ph¬ng ph¸p sö dông tÝnh chÊt cña sè chÝnh ph¬ng VD1 : T×m nghiÖm nguyªn cña. a) x2 - 4xy + 5y2 = 169 21
  • 22. Chuyªn ®Ò BDHSG To¸n THCS b) x2 - 6xy + 13y2 = 100 Gi¶i : a) (x - 2y)2 + y2 = 169 = 0 + 169 = 25 + 144... b) (x – 3y)2 + (2y)2 = 100 = 0 + 100 = 36 + 64 = ... Ph¬ng ph¸p 5 : Ph¬ng ph¸p c«ng thøc nghiÖm ph¬ng tr×nh bËc 2 VD1 : T×m nghiÖm nguyªn cña ph¬ng tr×nh. a) 2x2 -2xy + x + y + 15 = 0 b) 5(x2 + xy + y2 ) = 7(x+2y) (®Ò thi häc sinh giái tØnh 2009 – 2010) c) x(x + 1) = y (y + 1) (y2 + 2) Ph¬ng ph¸p 6 : Ph¬ng ph¸p ®Æt Èn phô VD: T×m nghiÖm nguyªn cña ph¬ng tr×nh: 6 7 32 22 22 12 2 2 2 2 = ++ ++ + ++ ++ xx xx xx xx (1) §Æt y = x2 + 2x + 2 (y ∈ Z) (1) 6 7 1 1 = + + − ⇔ y y y y ⇔ 5y2 – 7y – 6 = 0 5 3 1 −=y (lo¹i) ; y2 = 2 (tho¶ m·n) ⇒ x1 = 0; x2 = -2 C¸c bµi tËp t¬ng tù: a) x3 + (x + 1)3 + (x + 2)3 = (x + 3)3 b) 12 1 )1( 1 )2( 1 2 = + − + xxx * Mét sè ph¬ng ph¸p kh¸c. VD1 : T×m nghiÖm nguyªn cña ph¬ng tr×nh : 2x2 + 4x = 19 -3y2 Gi¶i : ⇔ 4x2 + 8x + 4 = 42 - 6y2 (2x + 2)2 = 6 (7 - y2 ) V× (2x + 2)2 ≥ 0 ⇒ 7 - y2 ≥ 0 ⇒ 72 ≤y Mµ y Z∈ ⇒ y = 0 ; 1± ; 2± Tõ ®©y ta t×m ®îc gi¸ trÞ t¬ng øng cña x 3. Mét sè bµi to¸n liªn quan tíi h×nh häc. a) Cho tam gi¸c cã ®é dµi cña 3 ®êng cao lµ nh÷ng sè nguyªn d¬ng vµ ®êng trßn néi tiÕp tam gi¸c ®ã cã b¸n kÝnh b»ng 1(®.v.®.d). Chøng minh tam gi¸c ®ã lµ tam gi¸c ®Òu Gi¶i: Gäi ®é dµi c¸c c¹nh vµ c¸c ®êng cao t¬ng øng theo thø tù lµ a; b; c vµ x; y; z. R lµ b¸n kÝnh ®êng trßn néi tiÕp. Ta cã R = 1⇒ x; y; z > 2 vµ gi¶ sö x ≥ y ≥ z > 2 Ta cã : ax = by = cz = (a + b+ c).1 (=2S) Suy ra: a cba x ++ = ; c cba z b cba y ++ = ++ = ; cba a x ++ =⇒ 1 ; cba b y ++ = 1 ; cba c z ++ = 1 ⇒ 1 111 =++ zyx mµ x ≥ y ≥ z > 2 22
  • 23. Chuyªn ®Ò BDHSG To¸n THCS ⇒ xz 11 ≥ vµ yz 11 ≥ nªn zzyx 3111 ≤++ ⇒ z 3 1 ≤ ⇒ 3≤z ⇒ z = 3 T¬ng tù ta cã: x = 3; y = 3 ⇒ tam gi¸c ®ã lµ tam gi¸c ®Òu b) T×m tÊt c¶ c¸c h×nh ch÷ nhËt víi ®é dµi c¸c c¹nh lµ c¸c sè nguyªn d¬ng cã thÓ c¾t thµnh 13 h×nh vu«ng b»ng nhau sao cho mçi c¹nh cña h×nh vu«ng lµ sè nguyªn d¬ng kh«ng lín h¬n 4 (®.v.®.d) Gi¶i : Gäi c¸c c¹nh h×nh ch÷ nhËt cÇn t×m lµ a vµ b, c¹nh h×nh vu«ng lµ c. Tõ gi¶ thiÕt h×nh ch÷ nhËt c¾t thµnh 13 h×nh vu«ng nªn ph¶i cã: ab = 13c2 (1) víi 0 < c ≤ 4 (2) Tõ (1) suy ra a hoÆc b chia hÕt cho 13. V× vai trß a, b nh nhau ta cã thÓ gi¶ gi¶ sö a chia hÕt cho 13, tøc lµ a = 13d Thay vµo (1) ta ®îc : 13db = 13c2 Hay db = c2 Ta h·y xÐt c¸c trêng hîp cã thÓ cã cña c. Víi c = 1, chØ cã thÓ: d = 1, b = 1, suy ra a = 13 Víi c = 2, chØ cã thÓ: d = 1, b = 4, suy ra a = 13 d = 2, b = 2, suy ra a = 26 d = 4, b = 1, suy ra a = 52 Víi c = 3, chØ cã thÓ: d = 1, b = 9, suy ra a = 13 d = 3, b = 3, suy ra a = 39 d = 9, b = 1, suy ra a = 117 Víi c = 4, chØ cã thÓ: d = 1, b = 16, suy ra a = 13 d = 2, b = 8, suy ra a = 26 d = 4, b = 4, suy ra a = 52 d = 8, b = 2, suy ra a = 104 d = 16, b = 1, suy ra a = 208 Víi 12 nghiÖm cña ph¬ng tr×nh (1) chØ cã 4 trêng hîp tho¶ m·n bµi to¸n. Bµi to¸n cã 4 nghiÖm. Ta t×m ®îc 4 h×nh ch÷ nhËt tho¶ m·n ®Ò bµi: (a = 13, b = 1); (a = 26, b = 2); (a = 39, b = 3); (a = 52, b = 4) Chuyªn ®Ò 3: Gi¶i ph¬ng tr×nh v« tû vµ hÖ ph¬ng tr×nh (Dµnh cho båi dìng häc sinh giái tØnh) I. Gi¶i ph¬ng tr×nh v« tû * C¸c ph¬ng ph¸p 1. Luü thõa khö c¨n 23
  • 24. Chuyªn ®Ò BDHSG To¸n THCS 2. §Æt Èn phô 3. Dïng bÊt ®¼ng thøc 4. XÐt kho¶ng II. ¸p dông c¸c ph¬ng ph¸p A. Phương pháp luỹ thừa khử căn 1. Gi¶i c¸c ph¬ng tr×nh a) )1(2321 =−+− xx §iÒu kiÖn: x≤ 2 3 Víi 2 3 ≥x PT (1) 43522321 2 =+−+−+−⇔ xxxx xxx 383522 2 −=+−⇔      ≤ −+=+− ⇔ 3 8 )2(48964)352(4 22 x xxxx PT (2) 052282 =+−⇔ xx    = = ⇔ )(26 )(2 Kotmx tmx VËy PT ®· cho cã nghiÖm x=2 b) )1()1()1(3 22 −+=+− xxxx §K: 1≥x Víi 1≥x PT (1) 112)1(3 22 −+−+=+−⇔ xxxxxx 12442 2 −=+−⇔ xxxx 1222 −=+−⇔ xxxx Do 1≥x nªn 2 vÕ cña PT nµy kh«ng ©m v× vËy PT nµy 232324 48444 xxxxxxx −=+−−+−⇔ 04895 234 =+−+−⇔ xxxx 0)1()2( 22 =+−−⇔ xxx    =+− =− ⇔ 01 02 2 xx x 2=⇔ x ™ c) 1222 33 −=−−− xx (1) Gi¶i: Pt (1) ⇔ ( ) 1222 3 33 −=−−− xx 24
  • 25. Chuyªn ®Ò BDHSG To¸n THCS 1)22()2(.()22)(2(3222 333 −=−−−−−−+−−⇔ xxxxxx 3 2 46231 +−=−⇒ xxx )462(27331 232 +−=−+−⇔ xxxxx 010715951 23 =+−+⇔ xxx 0)10752)(1( 2 =−+−⇔ xxx    =+− = ⇔ 010752 1 2 xx x      −−= +−= = ⇔ 78326 78326 1 x x x B. Ph¬ng ph¸p ®Æt Èn phô (2) Gi¶i c¸c ph¬ng tr×nh: a) 3123 =++− xx Gi¶i: §K: 1−≥x §Æt ax =−3 2 ; bx =+1 ( 0≥b ) Ta cã hÖ PT    =+ −=− 3 323 ba ba Suy ra 06623 =−+− aaa 0)6)(1( 2 =+−⇔ aa )/(31 mTxa =⇒=⇔ VËy ph¬ng tr×nh nghiÖm 3=x b. )1(552 =+− xx §K: 5−≥x §Æt : yx =+5 ( )0≥y ta cã hÖ ph¬ng tr×nh     =− =− 5 5 2 2 xy yx 0)()( 22 =−+−⇒ yxyx    =++ = ⇔ 01yx yx +)    =−− ≥ ⇔=+⇒= 05 0 5 2 xx x xxyx 25
  • 26. Chuyªn ®Ò BDHSG To¸n THCS      ± = ≥ ⇔ 2 211 0 x x 2 211+ =⇔ x (Ko T/m) +) 01 =++ yx 015 =+++⇒ xx 51 +−=+⇔ xx )1(5 +−=+⇔ xx ⇔    +=++ ≤+ (*)512 01 2 xxx x PT (*) 042 =−+ xx       −− = +− = ⇔ 2 171 2 171 x x (ko t/m) VËy PT v« nghiÖm c) 6 2 4 ).2(5)4)(2( = + + ++++ x x xxx §K: 0 2 4 ≥ + + x x §Æt )2)(4()2.( 2 4 2 ++=⇒=+ + + xxaax x x Ta cã PT: 0652 =−+ aa    −= = 6 1 a a +) 01861 2 =−++⇒= xxa 0762 =++⇒ xx      −−= +− = 23 )( 1 23 x tmx +) 036866 2 =−++⇒−= xxa 02862 =−+⇒ xx     −−= +−= )(373 373 tmx x VËy pt cã 2 nghiÖm 373;23 −−+−=x C. ¸p dông bÊt ®¼ng thøc 26
  • 27. Chuyªn ®Ò BDHSG To¸n THCS (3) Gi¶i c¸c ph¬ng tr×nh a) 45224252642 =−−−+−++ xxxx (1) §K: 2 5 ≥x Víi §k: 2 5 ≥x PT (1) ⇔ 4152352 =−−++− xx Ta cã: 4152352 ≥−−++− xx §¼ng thøc xÈy ra      ≥ ≤−−+− ⇔ 2 5 0)152)(352( x xx ⇔ 3 2 5 ≤≤ x VËy nghiÖm cña PT ®· cho lµ 3 2 5 ≤≤ x b) )1(271064 2 +−=−+− xxxx Gi¶i §K 64 ≤≤ x Trªn TX§ )64)(11(64 22 xxxx −+−+≤−+− ⇔ 264 ≤−+− xx L¹i cã 22)5(2710 22 ≥+−=+− xxx xxxx −+−≥+−⇒ 6427102 §¼ng thøc xÈy ra 5 64 5 64 =⇔      ≤≤ = −=− ⇔ x x x xx VËy PT (1) cã nghiÖm lµ x=5 c) Gi¶i ph¬ng tr×nh 211 22 +−=+−+−+ xxxxxx Gi¶i 27
  • 28. Chuyªn ®Ò BDHSG To¸n THCS §K:     ≥++− ≥−+ 01 01 2 2 xx xx ¸p dông B§T c« si cho c¸c sè kh«ng ©m ta cã ⇒       +++− ≤++− +−+ ≤−+ 2 11 1).1( 2 11 1).1( 2 2 2 2 xx xx xx xx 111 22 +≤+−+−+ xxxxx Ta cã 122 +≥+− xxx (V× 0)1( 2 ≥−x ) 211 222 +−≤+−+−+⇒ xxxxxx §¼ng thøc xÈy ra 1=⇔ x VËy pt cã nghiÖm lµ x=1 D. XÐt kho¶ng (4) Gi¶i c¸c PT a) )1(353448 22 ++−=+ xxx Gi¶i TX§: x∀ PT(1) 343548 22 −=+−+⇔ xxx 34 3548 13 22 −= +++ ⇔ x xx ThÊy 1=x lµ nghiÖm cña PT (1) +) 1335481 22 >+++⇒> xxx ⇒      >− < +++ ⇒ 134 1 3548 13 22 x xx PT v« nghiÖm +) 1 4 3 <≤ x 133548 22 <+++⇒ xx 28
  • 29. Chuyªn ®Ò BDHSG To¸n THCS ⇒      <− > +++⇒ 134 1 3548 13 22 x xx PT v« nghiÖm VËy ph¬ng tr×nh cã nghiÖm duy nhÊt lµ x=1 b) 1235 3 46 =−−− xx (1) Gi¶i Ta cã: 1>x th× 1; 64 >xx 1<x th× 1; 64 <xx +) XÐt 1>x 123;45 46 >−<−⇒ xx 3 46 235 −−−⇒ xx  PT (1) v« nghiÖm Xet 1<x tương tự ta suy ra phương trình vô nghiệm ThÊy x= 1 hoÆc x= -1 lµ nghiÖm cña PT (1) Bµi tËp: Gi¶i c¸c PT (1) a) )(15)2(2 32 Bxx +=+ (b) )(917.17 22 Bxxxx =−+−+ (2) xxx +=− 3.3 (A) (3) 83124 22 ++=++ xxx (D) (4) )(13626 2 Cxxxx +−=++− (5) )(231034 Axx −=−− (6) 08645.27 56105 =+− xx (C) III. Gi¶i hÖ ph¬ng tr×nh * C¸c ph¬ng ph¸p: 1. Ph¬ng ph¸p thÕ 2. C«ng thøc trõ, nh©n, chia c¸c vÕ 3. §Æt Èn phô 4. Dïng bÊt ®¼ng thøc. IV. ¸p dông c¸c ph¬ng ph¸p. A. Ph¬ng ph¸p thÕ. 1. Gi¶i c¸c hÖ pg¬ng tr×nh 29
  • 30. Chuyªn ®Ò BDHSG To¸n THCS a)    =+ =+ 2947 113 yx yx Gi¶i HÖ ®· cho t¬ng ®¬ng víi    =−+ −= 29)311(47 311 xx xy    = −= ⇔ 155 311 x xy    = = 2 3 y x VËy hÖ ®· cho cã nghiÖm lµ: (x;y) = (3;2) b)     =−++ =+− 027624 065 2 22 yxyx xyx Gi¶i HÖ ®· cho t¬ng ®¬ng víi    =−++ =−− 027624 0)3)(2( 2 yxyx yxyx           =−+ =    =−+ = ⇔ 027642 3 027620 2 2 2 yy yx yy yx 30
  • 31. Chuyªn ®Ò BDHSG To¸n THCS                               −− = + = =               −− = − = = ⇔ 14 1271 14 1271 3 20 5493 20 3549 2 y y yx y y yx        +− = +− = 20 5493 10 5493 y x HoÆc        −− = −− = 20 5493 10 5493 y x        +− = +− = 14 1271 14 12733 y x HoÆc        −− = −− = 14 1271 14 12733 y x c)     =++ ++=++ 2004200320032003 222 3zyx zxyzxyzyx Gi¶i:     =++ ++=++ )2(3 )1( 2004200320032003 222 zyx zxyzxyzyx Ta cã: PT (1) 0222222 222 =−−−++⇔ zxyzxyzyx 0)()()( 222 =−+−+−⇔ xzzyyx zyx ==⇔ ThÕ vµo (2) ta cã: 20042003 33 =x 20032003 3=x 31
  • 32. Chuyªn ®Ò BDHSG To¸n THCS 3=x Do ®ã x= y=z = 3 VËy nghiÖm cña hÖ ®· cho lµ: (x;y;z) = (3;3;3) B. Ph¬ng ph¸p céng, trõ, nh©n, chia c¸c vÕ (2) Gi¶i c¸c hÖ ph¬ng tr×nh a)     =− =+ 226 2235 yx yx Gi¶i: HÖ ®· cho t¬ng ®¬ng víi:     =− =+ 226 4265 yx yx     =+ = ⇔ 2235 666 yx x       − = = 2 1 6 1 y x b)     += += 12 12 3 3 xy yx Gi¶i: HÖ ®· cho t¬ng ®¬ng víi     =−+− += 0)(2 12 33 3 yxyx yx     =+++− += ⇔ 0)2)(( 12 22 3 yxyxyx yx    = += ⇔ yx yx 123 (do 0222 >+++ yxyx ) 32
  • 33. Chuyªn ®Ò BDHSG To¸n THCS    = =−− ⇔ yx xx 0123    = =−−+ ⇔ yx xxx 0)1)(1( 2            =          + = − = −= ⇔ yx x x x 2 51 2 51 1    −= −= ⇔ 1 1 y x hoÆc        − = − = 2 51 2 51 y x hoÆc        + = + = 2 51 2 51 y x c)      =++ =++ =++ 9 4 1( xzxz zyzy yxyx trong ®ã 0,, zyx Gi¶i HÖ ®¸ cho t¬ng ®¬ng víi      =++ =++ =++ 10)1)(1( 5)1)(1( 2)1)(1( xz zy yx [ ]        =++ =++ =++ =+++ ⇔ 10)1)(1( 5)1)(1( 2)1)(1( 100)1)(1)(1( 2 xz zy yx zyx        =++ =++ =++ =+++ ⇔ 10)1)(1( 5)1)(1( 2)1)(1( 10)1)(1)(1( xz zy yx zyx 33 (Do x,y,z>0)
  • 34. Chuyªn ®Ò BDHSG To¸n THCS      =+ =+ =+ ⇔ 11 21 51 y x z      = = = ⇔ 4 0 1 z y x VËy hÖ ®· cho cã nghiÖm lµ (x;y;z)=(1;0;4) C. Ph¬ng ph¸p ®Æt Èn phô (3). Gi¶i c¸c hÖ ph¬ng tr×nh a)     ++=+ ++=+ 6 5 2233 22 xyyxyx yyxx §Æt: x-y=a; x+y =b HÖ ®· cho trë thµnh    = =+ )2(6 )1(5 2 ba aab Tõ PT (2) ta suy ra 0≠a Do ®ã: 2 6 a b = ThÕ vµo (1) ta ®îc: 5 6 =+ a a 0652 =+−⇔ aa (V× 0≠a ) 0)3)(2( =−−⇔ aa    = = ⇔ 3 2 a a +) 2 3 2 =⇒= ba 34
  • 35. Chuyªn ®Ò BDHSG To¸n THCS Hay       − = = ⇔     =− =+ 4 1 4 7 2 2 3 y x yx yx +) 3 2 3 =⇒= ba Hay       − = = ⇔     =− =+ 6 7 6 11 3 3 2 y x yx yx Tãm l¹i hÖ ph¬ng tr×nh ®· cho cã nghiÖm lµ: (x;y) =       −       − 6 7 ; 6 11 ; 4 1 ; 4 7 b)    =++ =++ 5 173333 yxyx yyxx Gi¶i: §Æt x+y = a; xy=b HÖ ®· cho trë thµnh    =+ =−+ 5 17333 ba abba    =+− −= ⇔ 065 5 2 bb ba    =−− −= ⇔ 0)3)(2( 5 bb ba    = = ⇔ 2 3 b a HoÆc    = = 3 2 b a 35
  • 36. Chuyªn ®Ò BDHSG To¸n THCS +)    = = 2 3 b a Ta cã hÖ ph¬ng tr×nh    = =+ 2 3 xy yx    =+− −= ⇔ 023 3 2 yy yx    =−− −= ⇔ 0)2)(1( 3 yy yx ⇔    = = 1 2 y x HoÆc    = = 2 1 y x +)    = = 3 2 b a Ta cã hÖ ph¬ng tr×nh    = =+ 3 2 xy yx    =+− −= ⇔ 032 2 2 yy yx (V« nghiÖm) HÖ nµy v« nghiÖm VËy nghiÖm cña hÖ ®· cho lµ: (x;y) = (1;2); (2;1) c) ( )     −=+ −=+ 78)( 2156 22 224 yxxy yxyx Gi¶i HÖ ®· cho t¬ng ®¬ng víi     −=+ −=+++ 78 21544 33 4334 xyyx yxyyxx ⇔     −=+ −=+++ 16770215215 167707831231278 33 4334 xyyx yxyyxx ⇔     −=+ =+++ 78 )1(078979778 33 4334 xyyx yxyyxx 36
  • 37. Chuyªn ®Ò BDHSG To¸n THCS §Æt y x t = PT (1) trë thµnh 078979778 34 =+++ ttt 0)131213)(32)(23( 2 =+−++⇔ tttt       − = − = ⇔ 2 3 3 2 t t +) yxt 3 2 3 2 − =⇒ − = ThÕ vµo (2) ta ®îc 78 27 26 4 =y 814 =⇔ y 3=⇔ y HoÆc 3−=y Suy ra:    = −= 3 2 y x HoÆc    −= = 3 2 y x +) yxt 2 3 2 3 − =⇒ − = ThÕ vµo (2) ta ®îc 78 8 39 4 =y 164 =⇔ y 2=⇔ y HoÆc 2−=y Suy ra:    = −= 2 3 y x HoÆc    −= = 2 3 y x Tãm l¹i hÖ ®· cho cã nghiÖm lµ: (x;y) = (-2;3); (2;-3); (-3;2) ; (3;-2) D. ¸p dông bÊt ®¼ng thøc (4) Gi¶i c¸c hÖ ph¬ng tr×nh a)    =++ =++ xyzzyx zyx 444 1 Gi¶i: NhËn xÐt: Tõ B§T 0)()()( 222 ≥−+−+− accbba 37
  • 38. Chuyªn ®Ò BDHSG To¸n THCS Ta suy ra: (*)222 cabcabcba ++≥++ ¸p dông liªn tiÕp B§T (*) ta ®îc 222222444 xzzyyxzyx ++≥++ )( zyxxyz ++≥ ⇔ xyzzyx ≥++ 444 §¼ng thøc xÈy ra khi: 3 1 === zyx VËy hÖ ®· cho cã nghiÖm lµ:       = 3 1 ; 3 1 ; 3 1 );;( zyx b)     −=−+ −=−+ yxx yxx 62432 332 4 24 Gi¶i: §K: 320 ≤≤ x HÖ ®· cho t¬ng ®¬ng víi     −=−+ +−=−++−+ 332 216)32()32( 24 244 yxx yyxxxx Theo bÊt ®¼ng thøc BunhiaCèp xki ta cã 64)32)(11()32( 222 =−++≤−+ xxxx 832 ≤−+⇒ xx ( ) [ ] 256)32(232 24 44 ≤−+≤−+ xxxx ⇒ 43244 ≤−+ xx Suy ra 12)32()32( 44 ≤−++−+ xxxx MÆt kh¸c ( ) 12123216 22 ≥+−=+− yyy §¼ng thøc xÈy ra khi x= 16 vµ y=3 (t/m) VËy hÖ ®· cã nghiÖm lµ (x;y) = (16;3) Chuyªn ®Ò 4: BẤT ĐẲNG THỨC VÀ GIÁ TRỊ LỚN NHẤT, GIÁ TRỊ NHỎ NHẤT A - CÁC PHƯƠNG PHÁP CHỨNG MINH BẤT ĐẲNG THỨC 1) Phương pháp đổi tương đương • Để chứng minh: A B≥ Ta biến đổi 1 1 n nA B A B A B≥ ⇔ ≥ ≥ ≥K (đây là bất đẳng thức đúng) 38
  • 39. Chuyªn ®Ò BDHSG To¸n THCS Hoặc từ bất đẳng thức đứng n nA B≥ , ta biến đổi 1 1 1 1n n n nA B A B A B A B− −≥ ⇔ ≥ ≥ ≥ ⇔ ≥K Ví dụ 1.1 ( ) ( ) 22 2 2 2 2 : ) 2 (1) b) a (1) CMR a a b a b b c ab bc ca + ≥ + + + ≥ + + Giải ( ) ( ) ( ) ( ) 22 2 2 2 2 ) 1 2 0 2 0 0 (2) a a b a b a b ab a b ⇔ + − + ≥ ⇔ + − ≥ ⇔ − ≥ Do bất đẳng thức (2) đúng nên bất đẳng thức (1) được chứng minh. b) ( ) ( ) ( ) ( ) ( ) ( ) ( ) ( ) ( ) 2 2 2 2 2 2 2 2 2 2 2 2 ) 1 2 2 0 2 2 2 0 0 (2) b a b c ab bc ca a ab b b bc c c ca a a b b c c a ⇔ + + − + + ≥ ⇔ − + + − + + − + ≥ ⇔ − + − + − ≥ Bất đẳng thức (2) đúng suy ra điều phải chứng minh. Ví dụ 1.2 CMR ( ) ( )( ) ( ) ( ) ( ) 4 4 3 3 4 4 4 3 3 3 ) 2 (1) ) 3 (1) a a b a b a b b a b c a b c a b c + ≥ + + + + ≥ + + + + Giải ( ) ( ) ( ) ( ) ( ) ( ) ( )( ) ( ) ( ) ( ) 4 4 4 3 3 4 4 3 3 3 3 3 23 3 2 2 ) 1 2 2 0 0 0 0 0 2 a a b a a b ab b a a b ab b a a b b a b a b a b a b a ab b ⇔ + − + + + ≥ ⇔ − − − ≥ ⇔ − − − ≥ ⇔ − − ≥ ⇔ − + + ≥ Do bất đẳng thức (2) đúng suy ra điều phải chứng minh. ( ) ( ) ( ) ( ) ( ) ( ) ( ) ( ) ( ) ( ) ( ) 4 4 4 4 3 3 4 3 3 3 3 4 4 4 3 3 4 4 3 3 4 4 3 3 2 2 22 2 2 2 2 2 ) 1 3 3 3 0 0 0 b a b c a a b a c b ab b c ac bc c a b a b ab b c b c bc a c a c ac a b a ab b b c b bc c a c a ac c ⇔ + + − + + + + + + + + ≥ ⇔ + − − + + − − + + − − ≥ ⇔ − + + + − + + + − + + ≥ Ví dụ 1.3 39
  • 40. Chuyªn ®Ò BDHSG To¸n THCS ( )( ) ( ) ( ) ( ) ( ) ( ) 22 2 2 2 2 22 2 2 2 : a) ax 1 b) 1 CMR a b x b by a b c d a c b d + + ≥ + + + + ≥ + + + Giải ( ) ( ) 2 2 2 2 2 2 2 2 2 2 2 2 2 2 2 2 2 ) 1 2 2 0 0 a a x a y b x b y a x abxy b y a y abxy b y ay bx ⇔ + + + ≥ + + ⇔ − + ≥ ⇔ − ≥ ( ) ( )( ) ( ) ( ) ( )( ) 2 22 2 2 2 2 2 2 2 2 2 2 2 ) 1 2 (2) b a b c d a b c d a c b d a b c d ac bd ⇔ + + + + + + ≥ + + + ⇔ + + ≥ + Nếu ac + bd < 0 thì (2) đúng Nếu 0ac bd+ ≥ thì ( ) ( )( ) ( ) ( ) 22 2 2 2 2 2 2 2 2 2 2 2 2 2 2 2 2 2 2 0 a b c d ac bd a c a d b c b d a c abcd b d ad bc ⇔ + + ≥ + ⇔ + + + ≥ − + ⇔ − ≥ Suy ra đpcm. Ví dụ 1.4 Cho a, b, c > 0, chứng minh rằng: ( ) 2 2 2 1 a b c a b c b c a + ≥ + + Giải ( ) ( ) ( ) ( ) ( ) ( ) ( ) 3 3 3 2 2 2 2 2 2 2 2 2 2 2 2 1 0 2 2 2 0 0 a c b a c b a bc b ac c ab ac a ab b ab b bc c bc c ca a ac a b ab b c bc c a ⇔ + + − − − ≥ ⇔ − + + − + + − + ≥ ⇔ − + − + − ≥ ⇒ đpcm. Ví dụ 1.5 Cho a, b, c > 0. CMR: ( ) ( ) ( )2 2 2 3a b c a b a c b c b a c abc+ − + + − + + − ≤ (1) Giải ( ) ( ) ( ) ( ) ( ) ( ) ( ) ( )( ) ( ) ( ) ( ) ( ) ( )( ) ( )( ) ( ) ( ) ( )( ) 2 2 2 2 2 2 2 2 2 / , 0 1 3 0 0 0 0 0 G s a b c abc a b c a b a c b c b a c a a ab ac bc b b bc ba ac c c ac bc ab a a b a c b b c b a c c a c b a b a ac b bc c a c b c a b a b c c a c b c ≥ > ⇔ − + − + + − + + − ≥ ⇔ − − + + − − + + − − + ≥ ⇔ − − + − − + − − ≥ ⇔ − − − + + − − ≥ ⇔ − + − + − − ≥ Suy ra ĐPCM. 40
  • 41. Chuyªn ®Ò BDHSG To¸n THCS 2) Phương pháp biến đổi đồng nhất Để chứng minh BĐT: A ≥ B. Ta biến đổi biểu thức A – B thành tổng các biểu thức có giá trị không âm. Ví dụ 2.1 Chứng minh rằng: ( ) ( ) 2 2 2 2 2 2 2 ) 1 ) 4 4 4 4 8 1 a a b c d ab ac ad b a b c ab ac bc + + + ≥ + + + + ≥ − + Giải 2 2 2 2 2 2 2 2 2 2 2 2 2 2 2 ) Ta có = 4 4 4 4 0 2 2 2 4 a a b c d ab ac ad a a a a ab b ac c ad d a a a a b c d + + + − − −       − + + − + + − + + ÷  ÷  ÷             = − + − + − + ≥ ÷  ÷  ÷       ( ) ( ) ( ) ( ) ( ) 2 2 2 2 2 2 2 2 2 ) Ta có: 4 4 4 4 8 4 4 4 4 8 2 4 4 2 2 2 0 b a b c ab ac bc a ab b c ac bc a b c c a b a b c + + − + − = − + + + − = − + + − = − + ≥ Ví dụ 2.2 Chứng minh rằng: a) ( ) ( ) 33 3 4 a b a b+ ≥ + với a, b > 0 b) ( ) ( ) ( ) ( ) 3 3 33 3 3 8 a b c a b b c c a+ + ≥ + + + + + với a, b, c > 0 c) ( ) 3 3 3 3 24a b c a b c abc+ + ≥ + + + với a, b, c ≥0 Giải ( ) ( ) ( ) ( ) ( ) ( ) ( ) 3 23 3 2 2 2 ) Ta có:4 4 3 0 a a b a b a b a ab b a b a b a b  + − + = + − + − +   = + − ≥ ( ) ( ) ( ) ( ) ( ) ( ) ( ) ( ) ( ) ( ) ( ) ( ) ( ) ( ) ( ) ( ) 3 3 33 3 3 3 3 33 3 3 3 3 3 2 2 2 ) Ta có:8 4 4 4 3 4 3 0 b a b c a b b c c a a b a b a c c a b c b c a b a b a c a c b c b c + + − + − + − +      = + − + + + − + + + − +       = + − + + − + + − ≥ ( ) ( ) ( ) ( ) ( ) ( ) ( ) ( ) ( ) ( ) 3 3 3 3 3 2 2 3 3 3 3 3 2 2 3 2 2 2 3 3 2 2 2 2 2 2 2 2 2 ) Ta có: 24 3 3 24 3 3 3 6 3 3 24 3 2 3 2 3 2 3 3 3 0 c a b c a b c abc a b a b c a b c c a b c abc a a b ab b a c abc ac bc a b abc a b c b abc a c b c abc b a c a abc b a c c a b a b c + + − − − − = + + + + + + − − − − = + + + + + + + − − − = + − + + − + + − = − + − + − ≥ 41
  • 42. Chuyªn ®Ò BDHSG To¸n THCS Ví dụ 2.3 Với a, b, c > 0. Chứng minh rằng: 1 1 4 ) 1 1 1 9 ) 3 ) 2 a a b a b b a b c a b c a b c c b c c a a b + ≥ + + + ≥ + + + + ≥ + + + Giải ( ) ( ) ( ) ( ) ( ) ( ) 2 2 2 2 2 41 1 4 ) Ta có: 0 1 1 1 )Ta có: 9 2 2 2 0 a b ab a b a a b a b a b a b a b b c a c b a b c a b c b a c b c a a b b c a c ab bc ac + − − + − = = ≥ + + +         + + + + − = + − + + − + + − ÷  ÷  ÷  ÷         − − − = + + ≥ ( ) ( ) ( ) ( ) ( ) ( ) ( ) ( ) ( ) ( ) ( ) ( ) ( ) ( ) ( ) ( ) ( ) ( ) ( ) ( ) 2 2 2 3 1 1 1 )Ta có: 2 2 2 2 2 2 2 1 1 1 1 1 1 1 1 1 2 2 2 1 2 a b c a b c c b c c a a b b c c a a b a b a c b a b c c a c b b c c a a b a b b c c a b c c a a c a b a b b c a b b c a c a c b c a c a b a b b       + + − = − + − + − ÷  ÷  ÷ + + + + + +      − + − − + − − + − = + + + + +       = − − + − − + − − ÷  ÷  ÷ + + + + + +      − − − = + + + + + + + +( ) ( ) ( ) ( ) ( ) ( ) ( ) 0 Cách 2 3 1 1 Ta có: 1 3 2 2 2 1 6 2 1 2 2 2 2 c a b c a b c b c c a a b b c c a a b a b a c a b b c a c b c b c a c a b a b b c b c a c a c a b b c a b a c b c a b a c   ≥           + + − = + + + + + − ÷  ÷  ÷ + + + + + +      + + + + + + + + +  = + + −  + + +   + + + + + +       = + − + + − + + − ÷  ÷  ÷ + + + + + +       0≥ 42
  • 43. Chuyªn ®Ò BDHSG To¸n THCS Ví dụ 2.4 ) Cho a, b 0. CMR: a + b 2a ab≥ ≥ (Bất đẳng thức Cô – si) 3 ) Cho a, b, c 0. CMR: a + b + c 3b abc≥ ≥ (Bất đẳng thức Cô – si) ) Cho a b c và .c x y z CMR≥ ≥ ≤ ≤ ( ) ( ) ( )3 axa b c x y z by cz+ + + + ≥ + + (Bất đẳng thức Trê bư sếp) Giải a) Ta có: ( ) 2 2 0a b ab a b+ − = − ≥ b) ( ) ( ) ( ) ( ) 2 2 2 3 3 3 3 3 3 3 3 3 31 3 0 2 a b c abc a b c a b c b a c + + − = + + − + − + − ≥    c) ( ) ( ) ( ) ( )( ) ( ) ( ) ( ) ( ) 3 ax 0 a b c x y z by cz y x a b z x b c x z c a + + + + − + + = − − + − − + − − ≥ Ví dụ 2.5 Cho a, b, c > 0. Chứng minh: ( )2 2 2 ) ) 3 bc ac ab a a b c a b c bc ac ab b a b c a b c + + ≥ + + + + ≥ + + Giải ( ) ( ) ( ) ( ) ( ) ( ) ( ) ( ) 2 2 2 2 2 2 2 2 2 2 2 2 22 2 2 2 2 2 2 2 2 2 2 2 1 1 1 ) 0 2 2 2 ) 3 1 0 2 a b a c b cbc ac ab a a b c c b a a b c ab ac bc bc ac ab b a b c a b c c a b a b c b a c a b c b a c − − − + + − + + = + + ≥   + + − + + ÷     = − + − + − ≥    Ví dụ 2.6 Chứng minh rằng 2 2 1 1 2 ) 1 1 1 a a b ab + ≥ + + + nếu ab ≥ 0 2 2 1 1 2 ) 1 1 1 b a b ab + ≤ + + + nếu a2 + b2 < 2 2 2 1 1 2 ) 1 1 1 c a b ab + ≥ − − − nếu -1 < a, b < 1 ( ) ( ) 2 2 1 1 1 ) 11 1 d aba b + ≥ ++ + nếu a, b > 0 Giải 43
  • 44. Chuyªn ®Ò BDHSG To¸n THCS ( ) ( ) ( )( )( ) 2 2 2 2 2 2 2 1 1 2 1 1 1 1 ) = + 1 1 1 1 1 1 1 1 0 1 1 1 a a b ab a ab b ab a b ab a b ab     + − − − ÷  ÷ + + + + + + +    − − ≥ + + + ( ) ( ) ( )( )( ) 22 2 2 2 1 0 1 ) 1 0 1 02 1 1 1 ab a b aba b b ab ab a b ab + ≥ − −+ ≤ = ⇒ ⇒ ≤ − ≤ + + + ( ) ( ) ( )( )( ) 2 2 2 2 2 2 2 22 2 1 1 2 1 1 1 1 ) = + 1 1 1 1 1 1 1 1 0 1 1 1 c a b ab a ab b ab a b a b a b ab     + − − − ÷  ÷ − − − − − − −    − − ≥ − − − ( ) ( ) ( ) ( ) ( ) ( ) ( ) 2 2 2 2 2 2 11 1 1 ) = 0 11 1 1 1 1 ab a b ab d aba b a b ab − + − + − ≥ ++ + + + + 3) Phương pháp sử dung tính chất của bất đẳng thức Cơ sở của phương pháp này là các tính chất của bất đẳng thức và một số bất đẳng thức cơ bản như: ( ) 2 ) , 1 1 ) và a.b > 0 0 ) 0 ) 0 1 1 4 ) , 0 a a b b c a c b a b a b a b c ac bd c d d a b e a b a b a b ≥ ≥ ⇒ ≥ ≥ ⇒ ≤ ≥ ≥ ⇒ ≥ ≥ ≥ − ≥ > ⇒ + ≥ + Ví dụ 3.1 Cho a + b > 1 . Chứng minh: 4 4 1 8 a b+ > Giải ( ) ( ) ( ) 2 2 2 2 22 2 4 4 1 0 2 2 1 2 8 a b a b a b a b a b + − ≥ ⇒ + ≥ > + ⇒ + ≥ > Ví dụ 3.2 Với a, b, c > 0. CMR 44
  • 45. Chuyªn ®Ò BDHSG To¸n THCS 3 3 3 3 3 3 2 2 2 ) ) a b c a ab ac bc b c a a b c b a b c b c a + + ≥ + + + + ≥ + + Giải ( ) ( ) ( ) ( ) ( ) ( ) ( ) ( ) 2 2 2 3 3 3 2 2 2 2 2 2 3 2 3 3 3 2 2 2 ) Ta có: , 0 ) Ta có: 3 3 , 0 3 2 3 2 3 2 x a x xy y x y y a b c a ab b b bc c c ac a ab ac bc b c a x b x y x y y a b c a b b c c a a b c b c a ≥ + − > ⇒ + + ≥ + − + + − + + − = + + ≥ − > ⇒ + + ≥ − + − + − = + + Ví dụ 3.3 Cho a, b, c > 0. CMR: 2 2 2 ) ) 2 bc ac ab a a b c a b c a b c a b c b b c c a a b + + ≥ + + + + + + ≥ + + + Giải a) dễ dàng chứng minh 2 bc ac c a b + ≥ ⇒ đpcm b) dễ dàng chứng minh 2 4 a b c a b c + + ≥ ⇒ + đpcm Ví dụ 3.4 1 1 1 1 1 1 ) cho x, y, z >0. t/m: 4. : 1 2 2 2 a CMR x y z x y z x y z x y z + + = + + ≤ + + + + + + b) Cho a, b, c là độ dài ba cạnh của một tam giác. Chứng minh 1 1 1 1 1 1 a b c a c b b c a a b c + + ≥ + + + − + − + − c) Cho a, b, c > 0 thỏa mãn: abc = ab + bc + ca. Chứng minh: 1 1 1 3 2 3 2 3 2 3 16a b c b c a c a b + + ≤ + + + + + + Giải 45
  • 46. Chuyªn ®Ò BDHSG To¸n THCS ( ) ( ) 1 1 4 ) Ta có: , 0 1 1 1 1 1 1 2 1 1 2 4 16 a a b a b a b x y z x y x zx x y x z x y z > ⇒ + ≥ +     ⇒ = ≤ + ≤ + + ÷  ÷ + + + + + + +    Tương tự: 1 1 1 2 1 1 1 1 1 2 ; 2 16 2 16x y z x y z x y z x y z     ≤ + + ≤ + + ÷  ÷ + + + +    1 1 1 4 1 1 1 1 2 2 2 16x y z x y z x y z x y z   + + ≤ + + = ÷ + + + + + +   1 1 4 2 ) 2 1 1 2 ; 1 1 2 1 1 1 1 1 1 b a b c a b c a a a b c b c a b a b c b c a c a b c a b c b c a a b c + ≥ = + − − + + ≥ + − + − + ≥ − + + − ⇒ + + ≥ + + + − − + + − ( ) ( ) ( ) 1 1 1 1 1 1 1 3 ) 2 3 2 4 2 16 32 32 1 3 1 1 1 1 3 1 tt: ; 3 2 32 16 32 2 3 32 32 16 1 1 1 6 1 1 1 3 2 3 3 2 2 3 32 16 c a b c a c b c a c b c a b c a b c a b c a b c a b c a b c a b c a b c a b c   = ≤ + ≤ + +  + + + + + + +  ≤ + + ≤ + + + + + +   ⇒ + + ≤ + + = ÷ + + + + + +   Ví dụ 3.5 Cho a, b, c > 0. Chứng minh rằng: 2 2 2 ) 2 ) 1 1 1 a b c a a b b c c a a b c a b c b b c c a a b a b c + + < + + + + + ≥ + + + + + + + + Giải a) áp dụng BĐT: 0, 0. ta có: y y t x y t x x t + > > > < + Ta có: ; ; 2 a a c b b a c c b a b a b c b c b c a c a a b c a b c a b b c c a + + + < < < + + + + + + + + + ⇒ + + < + + + 46
  • 47. Chuyªn ®Ò BDHSG To¸n THCS b) 2 2 2 2 2 1 Ta có: 1 2 1 2 3 1 1 1 2 3 mà 2 a a a a a b c a b c a b c b c c a a b + ≥ ⇒ ≤ + ⇒ + + ≤ + + + + + ≥ + + + suy ra điều phải chứng minh. 4)Phương pháp sử dung bất đẳng thức Co-si 1 2 1 2 1 2*) Cho , ,..., 0, ta có: ... . ...n n n na a a a a a n a a a≥ + + + ≥ Dấu “=” xảy ra khi 1 2 ... 0na a a= = = ≥ Ví dụ 4.1 Cho a, b > 0 thỏa mãn ab = 1. CMR: ( ) ( )2 2 4 1 8a b a b a b + + + + ≥ + Giải Áp dụng BĐT Cosi ta có ( ) ( )( ) ( ) ( ) 2 2 2 2 2 2 2 2 4 4 2 4 4 4 1 2 1 4 2 4 2 2 8 a b ab a b ab a b a b a b a b a b a b a b a b a b a b a b a b + ≥ = + ≥ = + + ≥ + = + + ⇒ + + + + ≥ + + + + +   = + + + + + ≥ + + = ÷ +  Ví dụ 4.2 Chứng minh rằng: ( ) 2 ) 2 4 a b a b a a b b a + + + ≥ + với a, b 0≥ ) 2 a b c b b c c a a b + + > + + + với a,b,c > 0 Giải ( ) ( ) 2 2 1 1 a) Ta có: 2 4 2 2 2 1 1 1 mà , 4 4 2 2 4 a b a b a b a b ab a b a a b b a b a b a b a b a b b a + + +     + = + + ≥ + + ÷  ÷       + ≥ + ≥ ⇒ + + ≥ + ÷   + + ⇒ + ≥ + 47
  • 48. Chuyªn ®Ò BDHSG To¸n THCS 1 ) 1 1 2 tt: ; b c b c a b c b a a a a a b c a b c b b c c a c a b c a b a b c + + + +  × ≤ + = ÷   ⇒ ≥ + + + ≥ ≥ + + + + + + Cộng vế với vế ta được: 2 a b c b c c a a b + + ≥ + + + Dấu “=” xảy ra khi 0 a b c b a c a b c c a b = +  = + ⇒ + + =  = + vô lí. Vậy dấu “=” không xảy ra. Ví dụ 4.3 Cho a, b, c > 0. Chứng minh rằng: 2 2 2 3 3 3 2 2 2 2 2 2 3 3 3 2 2 2 2 2 2 2 2 2 ) 2 1 1 1 ) 3; ( 1) 2 a b c a b c a b c c a a b abc a b c b a b c a b b c c a abc + + + + ≤ + + + + + + + ≤ + + + = + + + Giải 2 2 2 2 2 2 2 2 2 2 2 2 2 2 2 2 2 2 3 3 3 2 2 2 2 2 2 ) Ta có: ; ; 2 2 2 2 2 2 2 a a b b c c a b c bc a c ac a b ab a b c a b c a b c b c a c a b bc ac ab abc ≤ ≤ ≤ + + + + + ⇒ + + ≤ + + = + + + ( )2 2 2 2 2 2 2 2 2 2 2 2 2 2 2 2 2 2 3 3 3 2 2 2 2 2 2 1 1 1 1 1 1 ) Ta có: 3 3 2 b a b c a b b c c a a b b c c a c a b a b c a b b c c a abc   + + = + + + + ÷ + + + + + +  + + = + + + ≤ + + + + Ví dụ 4.4 Cho a, b, c > 0. Chứng minh rằng 3 3 3 3 3 3 1 1 1 1 a b abc b c abc c a abc abc + + ≤ + + + + + + Giải ( ) ( ) 2 2 3 3 3 3 Ta có: 2 0a b ab a b ab a b abc abc c a b abc ab a b abc a b c + ≥ ⇒ + ≥ + > ⇒ ≤ = + + + + + + 3 3 3 3 tt: ; abc a abc b b c abc a b c c a abc a b c ≤ ≤ + + + + + + + + Cộng vế với vế suy ra điều phải chứng minh Ví dụ 4.5 Cho a, b, c > 0 thỏa mãn a2 +b2 + c2 = 3. Chứng minh rằng 48
  • 49. Chuyªn ®Ò BDHSG To¸n THCS 3 (1) ab bc ca c a b + + ≥ Gải ( ) ( ) ( ) 2 2 2 2 2 2 2 2 2 2 2 2 2 2 2 2 2 2 2 2 2 2 2 2 2 2 2 1 2 3 mà : a b b c c a a b c a b c c a b a b b c c a ab bc ab ac bc ca a b c c a b c a c b a b ⇔ + + + + + ≥ + + + + ≥ × + × + × = + + Suy ra điều phải chứng minh. Ví dụ 4.6 Cho x, y, z > 0 thỏa mãn xyz = 1. Chứng minh ( ) ( ) ( )( ) ( )( ) 3 3 3 5 5 5 5 5 5 3 ) 1 1 1 1 1 1 4 ) 1 x y z a y z z x x y xy yz zx b x xy y y yz z z zx x + + ≥ + + + + + + + + ≤ + + + + + + Giải a) Áp dụng bất đẳng thức cô si ta có: ( ) ( ) 3 1 1 3 1 1 8 8 4 x y z x y z + + + + ≥ + + Tương tự suy ra VT 333 3 3 2 4 2 4 4 xyzx y z+ + ≥ − ≥ − = ( ) ( ) ( ) 2 2 5 5 2 2 5 5 2 2 5 5 5 5 5 5 5 5 5 5 ) Ta có: 2 0 1 1 tt: ; 1 b x y xy x y x y x y xy xy z x xy y xy x y x y xy x y x y z yz x zx y y yz z x y z z xz x x y z xy yz zx x xy y y yz z z xz x + ≥ ⇒ + ≥ + > ≤ = = + + + + + + + + ≤ ≤ + + + + + + + + ⇒ + + ≤ + + + + + + Ví dụ 4.7 Cho x, y, z > 0. Chứng minh 3 3 3 x y z x y z yz zx xy + + ≥ + + Giải Áp dụng bất đẳng thức Cosi ta có: 3 3 3 3 3 3 3 ; 3 ; 3 x y z z y x z x y x y z yz zx xy x y z x y z yz zx xy + + ≥ + + ≥ + + ≥ ⇒ + + ≥ + + 5)Phương pháp sử dung bất đẳng thức Bunhiacopski 49
  • 50. Chuyªn ®Ò BDHSG To¸n THCS *) ( )( ) ( ) 22 2 2 2 a b x y xa by+ + ≥ + dấu “=” xảy ra khi a kx b ky =  = *) ( )( ) ( ) 22 2 2 2 2 2 a b c x y z xa by cz+ + + + ≥ + + dấu “=” xảy ra khi a kx b ky c kz =  =  = Tổng quát: ( )( ) ( ) 22 2 2 2 2 2 1 2 1 2 1 1 2 2... ... ...n n n xa a a x x x a x a x a x+ + + + + + ≥ + + + dấu “=” xảy ra khi ai = kxi Ví dụ 5.1 Cho a, b > 0. Chứng minh ( ) 22 2 1 1 4 ) ) a a b a b m nn m b a b a b + ≥ + + + ≥ + Giải a) Áp dụng bất đẳng thức Bunhiacopski ta có: ( ) 2 1 1 1 1 . . 4 1 1 4 a b a b a b a b a b a b    + + ≥ + = ÷  ÷     ⇒ + ≥ + ( ) ( ) ( ) 22 2 2 22 2 ) . . n m n m b a b a b n m a b a b n mn m a b a b     + + ≥ + = + ÷  ÷    + ⇒ + ≥ + Tổng quát: • Cho 0, 1.ib i n> = thì ( ) 22 2 2 1 21 2 1 2 1 2 ... ... ... nn n n a a aa a a b b b b b b + + + + + + ≥ + + + (1) • Với 0i ia c > với 1.i n= thì ( ) 2 1 21 2 1 2 1 1 2 2 ... ... ... nn n n n a a aa a a c c c a c a c a c + + + + + + ≥ + + + (2) Thật vậy: ( ) ( ) ( ) 2 2 2 2 21 2 1 2 1 2 1 2 1 2 1 2 1 2 22 2 2 1 21 2 1 2 1 2 ... ... . . ... . ... ... ... ... n n n n n n n nn n n a a a a a a b b b b b b a a a b b b b b b a a aa a a b b b b b b    + + + + + + ≥ + + + = + + + ÷ ÷  ÷    + + + ⇒ + + + ≥ + + + đặt aici = bi > 0 thay vào (1) được (2) Ví dụ 5.2 Cho a,b,c là các số thực dương. Chứng minh 50
  • 51. Chuyªn ®Ò BDHSG To¸n THCS 2 2 2 2 2 2 3 3 3 3 3 3 2 2 2 2 2 2 ) . ) 2 ) ) 2 a b c a b c a b c a a b c b b c a b c c a a b a b c a b c a b c c a b c d b c a b c c a a b + + + + ≥ + + + + ≥ + + + + + + + ≥ + + + + ≥ + + + Giải ( ) ( ) ( ) ( ) 22 2 2 22 2 2 22 2 23 3 3 4 4 4 2 2 2 3 3 3 4 4 4 2 2 2 ) Ta có: ) 2 2 ) ) 2 a b ca b c a a b c b c a a b c a b ca b c a b c b b c c a a b a b c a b ca b c a b c c a b c b c a ab bc ca ab bc ca a b c a b c a b c d b c a c a b ab ac ab bc ac bc + + + + ≥ = + + + + + + + + + + ≥ = + + + + + + + + + = + + ≥ ≥ + + + + + + + + = + + ≥ + + + + + + Ví dụ 5.3 Cho a, b, c > 0. Chứng minh: 25 16 8 a b c b c c a a b + + > + + + Giải ( ) ( ) ( ) ( ) 2 Ta có: 25 1 16 1 1 42 5 4 125 16 1 42 8 2 a b c VT b c c a a b a b c a b c b c c a a b a b c     = + + + + + − = ÷  ÷ + + +    + +  + + + + ≥ + + − = ÷ + + + + +  Dấu “=” xảy ra khi 0 5 4 1 b c a c a b a + + + = + ⇒ = vô lí suy ra điều phải chứng minh. Ví dụ 5.4 Cho x, y, z > 0. Chứng minh: 2 2 2 2 2 2 x y z x y z y z x y z x + + ≥ + + Giải Áp dụng bất đẳng thức Bunhiacopki ta có 22 2 2 2 2 2 1 1 3 3 x y z x y z x y z x y z x y z y z x y z x y z x y z x y z x      + + ≥ + + ≥ + + + + ≥ + + ÷  ÷ ÷      B – CÁC PHƯƠNG PHÁP TÌM GIÁ TRỊ LỚN NHẤT, GIÁ TRỊ NHỎ NHẤT Cho biểu thức f(x,y…) • Ta nói M là giá trị lớn nhất của f(x,y…) kí hiệu maxf(x,y…) = M, nếu hai điều kiện sau được thỏa mãn: - Với mọi x,y… để f(x,y…) xá định thì f(x,y…) ≤ M - Tồn tại x0, y0… sao cho f(x0,y0…) = M 51
  • 52. Chuyªn ®Ò BDHSG To¸n THCS • Ta nói m là giá trị nhỏ nhất của f(x,y…) kí hiệu minf(x,y…) = m, nếu hai điều kiện sau được thỏa mãn: - Với mọi x,y… để f(x,y…) xá định thì f(x,y…) ≥ m - Tồn tại x0, y0… sao cho f(x0,y0…) = m I) TÌM GTLN, GTNN CỦA ĐA THỨC BẬC HAI 1) Đa thức bậc hai một biến Ví dụ 1.1 a) Tìm GTNN của A = 3x2 – 4x + 1 b) Tìm GTLN của B = - 5x2 + 6x – 2 c) Tìm GTNN của C = (x – 2)2 + (x – 3)2 d) Cho tam thức bậc hai P = ax2 + bx + c Tìm GTNN của P nếu a > 0 Tìm GTNN của P nếu a > 0 Giải a) A = 2 2 4 4 1 2 1 1 3 3 3 9 3 3 3 3 x x x     − + − = − − ≥ − ÷  ÷     . Vậy minA= 1 2 khi 3 3 x− = b) B = 2 2 6 9 1 3 1 1 5 5 5 25 5 5 5 5 x x x     − − + − = − − − ≤ − ÷  ÷     . Vậy maxB = 1 3 khi 5 5 x− = c) C = 2 2 2 2 25 1 5 1 1 4 4 6 9 2 5 2 4 2 2 2 2 x x x x x x x     − + + − + = − + + = − + ≥ ÷  ÷     . Vậy maxC = 1 5 khi 2 2 x = d) Ta có: P = 22 2 2 2 4 4 2 4 b b b b b a x x c a a c a a a a a     + + + − = + + − ÷  ÷    Nếu a > 0 thì P ≥ 2 4 b c a − . Vậy minP = 2 4 b c a − khi 2 b x a = − Nếu a < 0 thì P ≤ 2 4 b c a − . Vậy maxP = 2 4 b c a − khi 2 b x a = − Ví dụ 1.2 a) Tìm GTNN của M = x2 – 3x + 1 với 2x ≥ b) Tìm GTLN của N = x2 – 5x + 1 với 3 8x− ≤ ≤ Giải a) M = ( ) ( )1 2 1 1x x− − − ≥ − . Vậy minM = -1 khi x = 2 b) N = ( ) ( )3 8 25 25x x+ − + ≤ . Vậy maxN = 25 khi x = -3, x = 8 2. Đa thức bậc hai hai biến a) Đa thức bậc hai hai biến có điều kiện Ví dụ 2a.1 a) Cho x + y = 1. Tìm GTLN của P = 3xy – 4 b) Cho x – 2y = 2. Tìm GTNN của Q = x2 + 2y2 – x + 3y Giải a) ( ) 2 1 13 13 1 1 3 1 4 3 2 4 4 x y x y P y y y −  + = ⇒ = − ⇒ = − − = − − − ≤ ÷   52
  • 53. Chuyªn ®Ò BDHSG To¸n THCS Vậy maxP = 13 1 khi 4 2 x − = 2 2 2 2 b) 2 2 2 2 4 8 4 2 2 2 3 3 9 11 11 6 9 2 6 2 16 8 8 x y x y Q y y y y y y y y y − = ⇒ = + ⇒ = + + + − − +   = + + = + + − ≥ − ÷   Vậy minQ = 11 3 khi 8 4 x− = − Ví dụ 2a.2 Tìm GTLN của của P = xy vói x, y thỏa mãn a) 6, 4x y y+ = ≥ b) , 2 S x y S y a+ = ≥ > a) ( ) ( )( )6 8 2 4 8P y y y y= − = − − − ≤ . Vậy maxP = 8 khi x = 2, y = 4 b) ( ) ( ) ( ) ( ) ( )Q S y y S a a y a y a S S a a= − = − − − + − ≤ − . Vậy maxQ = (S – a)a khi x = S – a, y = a b) Đa thức bậc hai hai biến Cho đa thức: P(x,y) = ax2 + bxy + cy2 + dx + ey + h (1), với a,b,c ≠ 0 Ta thường đưa P(x, y) về dạng P(x, y) = mF2 (x, y) + nG2 (y) + k (2) P(x, y) = mH2 (x, y) + nG2 (x) + k (3) Trong đó G(y), H(x) là hai biểu thức bậc nhất một ẩn, H(x, y) là biểu thức bậc nhất hai ẩn. Chẳng hạn nếu ta biến đổi (1) về (2) với a, (4ac – b2 ) ≠ 0 ( ) ( ) ( ) ( ) ( ) 2 2 2 2 2 2 2 2 2 2 22 2 2 2 2 2 4 ( , ) 4 4 4 4 4 4 4 4 4 2 4 2 2 4 22 2 4 4 4 4 aP x y a x abxy acy adx aey ah a x b y d abxy adx bdy ac b ae bd y ah d ae bdae bd ax by d ac b y ahd ac b ac b = + + + + + = + + + + + + − + − + − −−  = + + + − + + − ÷ − −  (Tương tự nhân hai vế của (1) với 4c để chuyển về (3)) Ví dụ 3.1 a) Tìm GTNN của P = x2 + y2 + xy + x + y b) Tìm GTLN của Q = -5x2 – 2xy – 2y2 + 14x + 10y – 1 Giải ( ) 2 2 2 2 2 2 2 a) 4 4 4 4 4 4 4 1 4 4 2 3 2 1 1 4 4 = 2 1 3 3 3 3 P x y xy x y x y xy x y y y x y y = + + + + = + + + + + + + −   + + + + − ≥ − ÷   Vậy minP = 4 1 khi 3 3 x y− = = − ( ) ( ) ( ) ( ) 2 22 2 2 2 b) 5 25 10 10 70 50 5 = 5 7 3 6 80 1 9 5 7 2 16 16 5 5 Q x xy y x y x y y Q x y y − = + + − − + − − + − − ⇒ = − − − − − + ≤ Vậy maxQ = 16 khi x = 1, y = 2 Ví dụ 3.2 53
  • 54. Chuyªn ®Ò BDHSG To¸n THCS Tìm cặp số (x, y) với y nhỏ nhất thỏa mãn: x2 + 5y2 + 2y – 4xy – 3 = 0 (*) Giải ( ) ( ) ( ) ( )( ) 2 2 2 (*) 2 1 4 1 4 3 1 0 3 1 x y y y y y y ⇔ − + + = ⇒ + ≤ ⇒ + − ≤ ⇒ − ≤ ≤ Vậy miny = -3 khi x = -6. Vậy ccawpj số (x, y) = (-6; -3) Ví dụ 3.3 Cho x, y liên hệ với nhau bởi hệ thức x2 + 2xy + 7(x + y) + 7y2 + 10 = 0 (**). Hãy tìm GTLN, GTNN của S = x + y + 1. Giải ( ) ( ) ( ) ( ) ( ) ( ) 2 2 2 22 ** 4 8 28 28 4 40 0 2 2 7 4 9 2 2 7 9 5 0 5 2 0 vì 2 5 2 0 4 1 x xy x y y x y y x y x y x y x y x y x y x y S ⇔ + + + + + = ⇔ + + + = ⇒ + + ≤ + + ≥ ⇔ + + + + ≤ ⇔ + + ≤ + + + + ≤ ⇔ − ≤ ≤ − Vậy minS = -4 khi x = -5, y = 0. maxS = -1 khi x = -2, y = 0. II. PHƯƠNG PHÁP MIỀN GIÁ TRỊ Ví dụ 1 Tìm GTLN, GTNN của A = 2 2 4 2 3 1 x x x + + + Giải Biểu thức A nhận giá trị a khi phương trình sau đây cóa nghiệm a = 2 2 4 2 3 1 x x x + + + ( ) ( )2 1 4 2 3 0 1a x x a⇔ − − + − = Nếu a = 1 thì phương trình (1) có nghiệm x = 2 4 − Nếu a ≠ 1 thì phương trình (1) có nghiệm khi –a2 + 4a +5 0 1 5.a≥ ⇔ − ≤ ≤ Vậy minA = -1 khi 2x = − maxA = 5 khi 2 2 x = Ví dụ 2 Tìm GTLN, GTNN của biểu thức B = 2 2 2 1 7 x y x y + + + + Giải Biểu thức B nhận giá trị b khi phương trình sau có nghiệm b = 2 2 2 1 7 x y x y + + + + ( )2 2 2 7 1 0 2bx x by y b⇔ − + − + − = Trong đó x là ẩn, y là tham số và b là tham số có điều kiện Nếu b = 0 2 1 0x y⇒ + + = 54
  • 55. Chuyªn ®Ò BDHSG To¸n THCS Nếu b ≠ để (2) có nghiệm x khi 1 – 4b(by2 – 2y + 7b -1) 0≥ (3) Coi (3) là bất phương trình ẩn y. BPT này xảy ra với mọi giá trị của y khi 16b2 + 4b2 (-28b2 + 4b + 1) 0≥ 2 5 1 28 4 5 0 14 2 b b b⇔ − + + ≥ ⇔ − ≤ ≤ Vậy minB = 5 7 14 khi x = - , 14 5 5 y− = − maxB = 1 khi x = 1, y = 2 2 III. PHƯƠNG PHÁP SỬ DỤNG BẤT ĐẲNG THỨC 1) Sử dụng bất đẳng thức Cô-si Ví dụ 1.1 Tìm GTLN, GTNN của A = 3 5 7 3x x− + − với 5 7 3 3 x≤ ≤ Giải ( ) ( ) ( )( )2 3 5 7 3 2 3 5 7 3 2 2 3 5 7 3A x x x x x x= − + − + − − = + − − Vậy A2 2 2.A≥ ⇒ ≥ Vậy minA = 5 7 2 khi , 3 3 x y= = 2 4 2 max 2 khi 2A A A x≤ ⇒ ≤ ⇒ = = (Biểu thức được cho dưới dạng tổng hai căn thức. Hai biểu thức lấy căn có tổng là hằng số) Ví dụ 1.2 Cho x, y > 0 thỏa mãn x + y 6≥ . Hãy tìm GTNN của P = 6 8 3 2x y x y + + + Giải Ta có: ( ) 3 3 6 8 3 3 6 8 .6 2 . 2 . 9 6 4 19 2 2 2 2 2 2 x y x y P x y x y x y = + + + + + ≥ + + = + + = Vậy minP = 19 khi x = 2, y = 4. Ví dụ 1.3 Tìm GTLN của biểu thức M = 2 3x y y x xy − + − với 3; 2x y≥ ≥ Giải To có: 23 yx M x y −− = + Theo bất đẳng thức Cô – si ta có: ( ) ( ) 23 3 2 3 3 ; 2 2 2 6 2 4 yx x y x y x y −− − ≤ ⇒ ≤ − ≤ ⇒ ≤ 55
  • 56. Chuyªn ®Ò BDHSG To¸n THCS 3 2 3 2 max khi 6; 4 6 4 6 4 M M x y⇒ ≤ + ⇒ = + = = Ví dụ 1.4 Cho x, y, z > 0 thỏa mãn: ( ) 1 1 1 2 1 1 1 1x y z + + ≥ + + + . Tìm TGLN của P = xyz Giải ( ) ( ) ( ) 1 1 1 1 1 1 2 1 1 1 1 1 1 1 y z z x y z y z y z     ⇒ ≥ − + + = + ≥ ÷ ÷ + + + + + + +   Tương tự: ( )( ) ( ) ( ) 1 1 2 ; 2 1 1 1 1 1 1 xz xy y x z z x y ≥ ≥ + + + + + + Nhân vế với vế của ba BĐT trên 1 1 1 max khi 8 8 2 P xyz P x y z⇒ = ≤ ⇒ = = = = Ví dụ 1.5 Cho 0 < x < 1, Tìm GTNN của Q = 3 4 1 x x + − Giải ( ) ( ) ( ) ( ) ( ) ( ) 2 2 2 4 1 4 13 3 Ta có: 7 2 . 7 2 3 1 1 4 13 minP = 2 3 khi 3 1 1 x xx x P x x x x xx x x x − − = + + ≥ + = + − − − ⇒ + = ⇒ = − − (Đặt P = ( )4 1-3 1- b xax c x x + + đồng nhất hệ số suy ra a = b = 1; c = 7) Ví dụ 1.6 Cho x, y, z, t > 0. Tìm GTNN của biểu thức. M x t t y y z z x t y y z z x x t − − − − = + + + + + + + Giải Áp dụng bất đẳng thức Cô-si ta có: 1 1 4 a b a b + ≥ + với a, b > 0. ( ) ( ) ( ) ( ) Ta có: 4 4 1 1 1 1 4 1 1 1 1 = 4 4 4 x t t y y z z x M M t y y z z x x t x y t z y x z t x y z t t y y z z x x t t y z x y z x t x y z t x y z t x y z t    − − − −    = + − = + + + + + + + − ÷  ÷ ÷  ÷ + + + +          + + + + + + + − = + + + + + ÷  ÷ + + + + + + + +    + + ≥ + − + + + + + + 4 0 minM = 0 khi x = y và z t = ⇒ = 2. Sử dụng BĐT Bunhiacopski (BCS) Ví dụ 2.1 Cho x, y, z thỏa mãn: xy + yz + zx =1. Tìm GTNN của biểu thức A = x4 + y4 + z4 Giải 56
  • 57. Chuyªn ®Ò BDHSG To¸n THCS Áp dụng BĐT BCS ta có ( ) ( )( ) ( ) ( ) ( )( ) 22 2 2 2 2 2 2 2 2 2 22 2 2 4 4 4 1 1 1 1 1 1 1 3 minP = khi x = y = z = 3 3 3 xy yz zx x y z x y z x y z x y z x y z P = + + ≤ + + + + = + + ⇒ ≤ + + ≤ + + + + ⇒ ≥ ⇒ Ví dụ 2.2 Tìm GTNN của P = 4 9 16a b c b c a c a b a b c + + + − + − + − trong đó a, b, c là độ dài ba cạnh của một tam giác. Giải ( ) ( ) ( ) ( ) 2 1 1 1 29 4 9 16 2 2 2 2 4 9 16 29 = 2 2 2 3 4 29 . 2 2 81 29 81 29 . 26 2 2 2 2 minP = 26 khi 7 6 5 a b c P b c a c a b a b c a b c b c a c a b a b c a b c b c a c a b a b c a b c a b c a b c       = + + + + + − ÷  ÷  ÷ + − + − + −      + +   + + − ÷ + − + − + −  + ++ + ≥ − + − + + − + + − + + = − = − = + + ⇒ = = Ví dụ 2.3 Tìm giá trị nhỏ nhất của Q = + + trong đó a, b, c > 0 thỏa mãn a + b + c = 1 Giải ( ) ( ) ( ) ( ) ( ) ( ) 2 2 Ta có: 1 2 2 2 2 2 2 3 1 minQ = 1 khi 3 a b c a b ca b c Q b c c a a b a b c b c a c a b ab bc ca a b c + + + + = + + ≥ = ≥ + + + + + + + + + + ⇒ = = = Ví dụ 2.4 Cho a, b, c > 0 thỏa mãn a + b + c = 1. Tìm GTNN của 2 2 2 1 1 1 1 P a b c ab bc ca = + + + + + Giải 57
  • 58. A M I OC E F N B M' Chuyªn ®Ò BDHSG To¸n THCS ( ) ( ) ( ) ( ) 2 2 2 2 2 2 2 2 2 1 1 1 9 1 9 1 4 7 2 1 2 9 21 9 30 1 minP = 30 khi a = b = c = 3 ab bc ca ab bc ca P a b c ab bc ca a b c ab bc ca ab bc ca ab bc caa b c a b c + + ≥ + + ⇒ ≥ + + + + +   = + + ÷ + + + + + +  + ≥ + ≥ + = + ++ + + + ⇒ Chuyªn ®Ò 5: : : Tø gi¸c néi tiÕp I -c¸c dÊu hiÖu nhËn biÕt tø gi¸c néi tiÕp 1- Tæng hai gãc ®èi b»ng 1800 2- Hai gãc liªn tiÕp cïng nh×n mét c¹nh díi hai gãc b»ng nhau. 3- NÕu hai c¹nh ®èi diÖn cu¶ gi¸c ABCD c¾t nhau t¹i M tháa m·n: MA.MB =MC.MD ; hoÆc hai ®êng chÐo c¾t nhau t¹i O tháa m·n OA.OC = OB.OD th× ABCD lµ tø gi¸c néi tiÕp 4- Sö dông ®Þnh lý Pt«lªmª II- C¸c vÝ dô VÝ dô1: Cho ®êng trßn t©m O vµ mét ®iÓm C ë ngoµi ®êng trßn ®ã. Tõ C kÎ hai tiÕp tuyÕn CE ; CF ( E vµ F lµ c¸c tiÕp ®iÓm) vµ c¸t tuyÕn CMN ( N n»m gi÷a C vµ M ) tíi ®êng trßn.§êng th¼ng CO c¾t ®êng trßn t¹i hai ®iÓm A vµ B. Gäi I lµ giao ®iÓm cña AB víi EF. Chøng minh r»ng: a, Bèn ®iÓm O, I, M, N cïng thuéc mét ®êng trßn b, = Gi¶i a, Do CE lµ tiÕp tuyÕn cña (O) nªn: = (Cïng ch¾n )  ∆CEM ~ ∆CNE .  =  CM.CN =CE2 MÆt Kh¸c , do CE; CF lµ c¸c tiÕp tuyÕn cña (O) nªn 58
  • 59. O O' A B C C' B' Chuyªn ®Ò BDHSG To¸n THCS AB⊥ EF t¹i I v× vËy trong tam gi¸c vu«ng CEO ®êng cao EI ta cã: CE2 = CI.CO Tõ (1) vµ (2) suy ra CM.CN = CI.CO => =  ∆CMI ~ ∆CON  =  Tø gi¸c OIMN néi tiÕp  b KÐo dµi NI c¾t ®êng trßn t¹i M’. Do tø gi¸c IONM néi tiÕp nªn : = = s® => = . Do ®ã: = =  VÝ Dô 2 Cho tam gi¸c ABC cã = 450 ; BC =a néi tiÕp trong ®êng trßn t©m O; c¸c ®êng cao BB’ vµ CC’. Gäi O’ lµ ®iÓm ®èi xøng cña O qua ®êng th¼ng B’C’. a. Chøng minh r»ng A; B’; C’; O’ cïng thuéc mét ®êng trßn b. TÝnh B’C’ theo a. Lêi gi¶i a. Do O lµ t©m ®êng trßn ngo¹i tiÕp tam gi¸c ABC nªn = 2 =900 Tõ ®ã suy ra c¸c ®iÓm O; B’; C’ Cïng thuéc ®êng trßn ®êng kÝnh BC.XÐt tø gi¸c néi tiÕp CC’OB’ cã : = 1800 - = 1800 - ( 900 - ) =1350 . Mµ O’ ®èi xøng víi O qua B’C’ nªn: = = 1350 =1800 - Hay tø gi¸c AC’O’B’ néi tiÕp. b. Do = 450 nªn ∆BB’A vu«ng c©n t¹i B’ V× vËy B’ n»m trªn ®êng trung trùc cña ®o¹n AB hay B’O ⊥ AB  C’OB’C lµ h×nh thang c©n nªn B’C’ =OC MÆt kh¸c ∆BOC vu«ng c©n nªn: B’C’ =OC = 2 2 2 2 aBC = III bµi tËp ¸p dông Bài tập 1: Cho tứ giác ABCD nội tiế đường tròn đường kính AD. Hai đường chéo AC và BD cắt nhau tại E. Vẽ EF vuông góc với AD. Chứng minh: 59
  • 60. Chuyªn ®Ò BDHSG To¸n THCS a/ Tứ giác EBEF, tứ giác DCEF nội tiếp. b/ CA là phân giác của ·BCF c/ Gọi M là trung điểm của DE. Chứng minh tứ giác BCMF nội tiếp. Bài tập 2: Tứ giác ABCD nội tiếp đường tròn đường kính AD. Hai đường chéo AC, BD cắt nhau tại E. Hình chiếu vuông góc của E trên AD là F. Đường thẳng CF cắt đường tròn tại điểm thứ hai là M. Giao điểm của BD và CF là N. Chứng minh: a/ CEFD là tứ giác nội tiếp b/ Tia FA là phân giác của góc BFM c/ BE.DN = EN.BD. Bài tập 3: Cho tam giác ABC vuông ở A và một điểm D nằm giữa A và B. Đường tròn đường kính BD cắt BC tại E. Các đường thẳng CD, AE lần lượt cắt đường tròn tại các điểm thứ hai F, G. Chứng minh: a/ Tam giác ABC đồng dạng với tam giác EBD b/ Tứ giác ADEC và AFBC nội tiếp được một đường tròn c/ AC song song với FG d/ Các đường thẳng AC, DE, BF đồng quy. Bài tập 4: Cho tam giác ABC có 0ˆ 90A = ; AB > AC, và một điểm M nằm trên đoạn AC ( M không trùng với A và C ). Gọi N và D lần lượt là giao điểm thứ hai của BC và MB với đường tròn đường kính MC; gọi S là giao điểm thứ hai giữa AD với đường tròn đường kính MC; T là giao điểm của MN và AB. Chứng minh: a/ Bốn điểm A, M, N, B cùng thuộc một đường tròn b/ CM là phân giác của góc BCS. c/ TA TC TD TB = Bài tập 5: Cho đường tròn (O) và điểm A nằm ngoài đường tròn. Qua A dựng hai tiếp tuyến AM và AN với đường tròn ( M, N là các tiếp điểm ) và một cact tuyến bất kỳ cắt đường tròn tại P, Q. Gọi L là trung điểm của PQ. a/ Chứng minh 5 điểm: O, L, M, A, N cùng thuộc một đường tròn b/ Chứng minh LA là phân giác của góc MLN c/ Gọi I là giao điểm của MN và LA. Chứng minh: MA2 = AI. AL d/ Gọi K là giao điểm của ML với (O). Chứng minh rằng: KN // AQ e/ Chứng minh tam giác KLN cân. 60
  • 61. Chuyªn ®Ò BDHSG To¸n THCS Bài tập 6: Cho đường tròn (O;R) tiếp xúc với đường thẳng d tại A. Trên d lấy điểm H không trùng với điểm A và AH < R. Qua H kẻ đường thẳng vuông góc với d, đường thẳng này cắt đường tròn tại hai điểm E và B ( E nằm giữa B và H ) a/ Chứng minh: góc ABE bằng góc EAH và tam giác AHB đồng dạng với tam giác EAH. b/ Lấy điểm C trên d sao cho H lá trung điểm của đoạn AC, đường thẳng CE cắt AB tại K. Chứng minh: AHEK là tứ giác nội tiếp c/ Xác định vị trí của điểm H để AB = R 3 Bài tập 7: Từ điểm P nằm ngoài đường tròn (O), kẻ hai tiếp tuyến PM và PN với đường tròn (O) ( M, N là tiếp điểm ). Đường thẳng đi qua điểm P cắt đường tròn (O) tại hai điểm E và F. Đường thẳng qua O song song với MP cắt PN tại Q. Gọi H là trung điểm của đoạn EF. Chứng minh: a/ Tứ giác PMON nội tiếp đường tròn b/ Các điểm P, N, O, H cùng nằm trên một đường tròn c/ Tam giác PQO cân d/ MP2 = PE. PF e/ = Bài tập 8: Cho tam giác ABC có ba góc nhọn nội tiếp đường tròn (O). Các đường cao AD, BE, CF cắt nhau tại H và cắt đường tròn (O) lần lượt tại M, N, P. Chứng minh rằng: a/ Các tứ giác AEHF, BFHD nội tiếp. b/ Bốn điểm B, C, E, F cùng nằm trên một đường tròn. c/ AE. AC = AH. AD và AD. BC = BE. AC d/ H và M đối xứng nhau qua BC e/ Xác định tâm đường tròn nội tiếp tam giác DEF. Bài tập 9: Cho tam giác ABC không cân, đường cao AH, nội tiếp trong đường tròn tâm O. Gọi E, F thứ tự là hình chiếu của B, C lên đường kính AD của đường tròn (O) và M, N thứ tự là trung điểm của BC, AB. Chứng minh: a/ Bốn điểm A, B, H, E cùng nằm trên một đường tròn tâm N và HE // CD. b/ M là tâm đường tròn ngoại tiếp tam giác HEF. 61
  • 62. Chuyªn ®Ò BDHSG To¸n THCS Bài tập 10: Cho đường tròn (O) và điểm A ở bên ngoài đường tròn. Vẽ các tiếp tuyến AB, AC và cát tuyến ADE với đường tròn ( B và C là các tiếp điểm ). Gọi H là trung điểm của DE. a/ CMR: A, B,H, O, C cùng thuộc một đường tròn. Xác định tâm của đường tròn này. b/ Chứng minh: HA là tia phân giác . c/ Gọi I là giao điểm của BC và DE. Chứng minh: AB2 = AI.AH d/ BH cắt (O) tại K. Chứng minh: AE // CK. Bài tập 11: Từ một điểm S ở ngoài đường tròn (O) vẽ hai tiếp tuyến SA, SB và cát tuyến SCD của đường tròn đó. a/ Gọi E là trung điểm của dây CD. Chứng minh 5 điểm S, A, E, O, B cùng thuộc một đường tròn. b/ Nếu SA = AO thì SAOB là hình gì? Tại sao?. c/ CMR: AC.BD = BC.DA = . 2 AB CD Bài tập 12: Trên đường thẳng d lấy 3 điểm A, B, C theo thứ tự đó. Trên nửa mặt phẳng bờ d kẻ hai tia Ax, By cùng vuông góc với d. Trên tia Ax lấy I. Tia vuông góc với CI tại C cắt By tại K. Đường tròn đường kính IC cắt IK tại P. a/ Chứng minh tứ giác CBPK nội tiếp được đường tròn b/ Chứng minh: AI. BK = AC. CB c/ Giả sử A, B, I cố định hãy xác định vị trí điểm C sao cho diện tích hình thang vuông ABKI lớn nhất. Bài tập 13: Cho tam giác đều ABC nội tiếp đường tròn (O). M là điểm di động trên cung nhỏ BC. Trên đoạn thẳng MA lấy điểm D sao cho MD = MC. a/ Chứng minh: VDMC đều b/ Chứng minh: MB + MC = MA c/ Chứng minh tứ giác ADOC nội tiếp được. d/ Khi M di động trên cung nhỏ BC thì D di động trên đường cố định nào?. Bài tập 14: 62